You are on page 1of 257

LECTURE NOTES

ON
CONSTRUCTION
PROJECT MANAGEMENT

Emad Elbeltagi, Ph.D., P.Eng.,


Professor of Construction Management
Structural Engineering Department,
Faculty of Engineering,
Mansoura University
Construction Project Management
2009

Copyright © 2009 by the author. All rights reserved. No part of this book may be
reproduced or distributed in any form or by any means, or stored in a data base or
retrieval system, without the prior written permissions of the author.
PREFACE

In the Name of ALLAH the Most Merciful, the Most Compassionate

All praise is due to ALLAH and blessings and peace be upon His messenger and servant,
Muhammad, and upon his family and companions and whoever follows his guidance
until the Day of Resurrection.

Construction project management is a relatively young field. However, its impact has
been quite remarkable. It has become an important practice for improving the efficiency
of construction operations around the world. This book deals with some topics and tools
of the large field of project management.

This book is dedicated mainly to undergraduate engineering students, especially Civil


Engineering students where most of the applications are presented in the civil engineering
field. It provides the reader with the main knowledge to manage a construction project
from preliminary stages to handover. It includes eight chapters: Chapter 1 provides a
general introduction to construction projects in terms of their types, project life cycle and
the main players involved. Chapter 2 is dedicated for the contract strategy. The planning
stages of a construction project are presented in chapter 3. Chapter 4 is dedicated for
presenting different scheduling techniques along with the schedule representation.
Chapter 5 is dedicated to discuss the scheduling methods on non-deterministic activity
durations. The scheduling of linear projects is presented in chapter 6. Chapter 7 is dealing
with both the resource scheduling and smoothing problems. The schedule compression is,
also, presented in chapter 8. Chapter 9 is dedicated for the project finance and cash flow
analysis. Finally, chapter 01 is dedicated for project control. Many solved examples have
been added to enable the students to understand the material presented in this book. Also,
each chapter is followed by exercises for training purposes.

Finally, May ALLAH accepts this humble work and I hope it will be beneficial to its
readers.

i
TABLE OF CONTENTS

CHAPTER 1: INTRODUCTION
1.1 The Need for Project Management 1
1.2 The Construction Project 2
1.3 The Project Scope and Goals 3
1.4 The Project Life-Cycle 6
1.4.1 Preconstruction phase 9
1.4.2 Procurement phase (Bidding and award phase) 10
1.4.3 Construction Phase 10
1.4.4 Closeout Phase 11
1.5 Major Types of Construction Projects 11
1.5.1 Residential Housing Construction 11
1.5.2 Institutional and Commercial Building Construction 12
1.5.3 Specialized Industrial Construction 13
1.5.4 Infrastructure and Heavy Construction 13
1.6 Construction Projects Participants 14
1.6.1 The Owner (Client) 14
1.6.2 The Design Professionals 15
1.6.3 The Construction Professionals 15
1.6.4 The Project Manager 16
1.7 Exercises 17

CHAPTER 2: CONTRACT STRATEGY


2.1 What is a Contract 19
2.2 Selection of Contract Type 20
2.2.1 Project Objectives 21
2.2.2 Project Constraints 22
2.3 Project Delivery Methods 23

ii
2.3.1 Traditional Approach 23
2.3.2 Direct Labor 24
2.3.3 Design-Build 24
2.3.4 Turnkey 25
2.3.5 Build-Operate-Transfer (BOT) 25
2.3.6 Professional Construction Management (PCM) 26
2.3.7 Contractual Relationships 26
2.4 Types of Contracts 28
2.4.1 Lump-sum Contract 28
2.4.2 Admeasurement Contract 29
2.4.3 Cost-reimbursable Contract (cost-plus contract) 30
2.4.4 Target Cost Contract 30
2.4.5 Time and Material (T&M) Contract 31
2.5 Contract Administration 31
2.5.1 Contract Documents 32
2.5.2 Conditions of Contract 33
2.5.3 The Standard (general) Forms of Conditions of Contract 34
2.5.4 Special Conditions of Contract 36
2.5.4 Construction claims Contract 37
2.6 Selecting the Contractor 38
2.7 Sub-Contracting 38
2.8 Exercises 39

CHAPTER 3: PROJECT PLANNING


3.1 Introduction 42
3.2 Project Planning Steps 43
3.2.1 Work Breakdown Structure (WBS) 44
WBS and organizational breakdown structure (OBS) 47
WBS coding 47

iii
3.2.2 Project Activities 48
3.2.3 Activities Relationships 52
Logical relationship considering resource constraints 54
Overlap or lag 55
Types of activities relationships 58
3.2.4 Drawing Project Network 58
Activity on arrow network (AOA) 59
Activity on node network (AON) 60
Comparison between AOA and AON 61
3.3 Estimating Activity Duration and Direct Cost 65
3.4 Exercises 68

CHAPTER 4: PROJECT SCHEDULING


4.1 The Critical Path Method 75
4.2 Calculations for the Critical Path Method 76
4.2.1 Activity-On-Arrow Networks Calculations 76
Forward path 77
Backward path 79
Float calculations 81
Identifying the Critical Activities 83
4.2.2 Precedence Diagram Method (PDM) 83
4.3 Time-Scaled Diagrams 84
4.4 Schedule Presentation 88
4.5 Criticisms to Network Techniques 89
4.6 Solved Examples 90
4.6.1 Example 1 90
4.6.2 Example 2 91
4.6.3 Example 3 92
4.6.4 Example 4 93
4.7 Exercises 94

iv
CHAPTER 5: STOCHASTIC SCHEDULING
5.1 Scheduling with Uncertain Durations 100
5.1.1 Program Evaluation and Review Technique 102
5.1.2 Criticism to Program Evaluation and Review Technique 109
5.2 Monte Carlo Simulation 110
5.2.1 Monte Carlo Simulation Characteristics 110
5.2.2 Monte Carlo Simulation Process 110
5.2.3 Criticality Index 113
5.3 Exercises 113

CHAPTER 6: SCHEDULING OF LINEAR PROJECTS


6.1 Linear Projects 116
6.2 Resource-Driven Scheduling 117
6.3 Summary Diagrams 117
6.3.1 Summary Diagrams Using One Relationship 117
6.3.2 Summary Diagrams Using Two Relationships 120
6.4 Line of Balance (LOB) 123
6.4.1 Basic Representation 123
6.4.2 LOB Calculations 125
Crew synchronization 126
Meeting a deadline duration 127
Calculating resource needs 128
Drawing the LOB Schedule 130
6.5 Exercises 134

CHAPTER 7: RESOURCES MANAGEMENT


7.1 Resource Definition 136
7.2 Resource Management 137
Resource leveling (smoothing) 138
Resource scheduling 138

v
7.3 Resource Allocation 129
7.4 Resource Aggregation (Loading) 129
7.5 Resource Leveling (Smoothing) 141
7.5.1 Method of Moments for Resource Smoothing 142
7.5.2 Heuristic Procedure for Resource Smoothing 143
7.6 Scheduling with Limited Resource 152
7.7 Case Study 154
7.8 Exercises 161

CHAPTER 8: PROJECT TIME-COST TRADE-OFF


8.1 Time-Cost Trade-Off 164
8.2 Activity Time-Cost Relationship 165
8.3 Project Time-Cost Relationship 169
8.4 Shortening Project Duration 170
8.5 Exercises 180

CHAPTER 9: PROJECT FINANCE AND CONTRACT PRICING


9.1 Contract Cash Flow 182
9.1.1 Construction Project Costs 183
Project direct costs 183
Project indirect costs 184
9.1.2 The S-Curve 186
9.1.3 Project Income (Cash-in) 188
9.1.4 Calculating Contract Cash Flow 190
9.1.5 Minimizing Contractor Negative Cash Flow 195
9.1.6 Cost of Borrowing (Return on Investment) 197
9.2 Project Cash Flow 202
9.2.1 Project Profitability Indicators 203
9.3 Discounted Cash Flow 205
9.3.1 Present Value 205
9.3.2 Net Present Value (NPV) 206

vi
9.3.3 Internal Rate of Return (IRR) 207
9.4 Finalizing a Tender Price 208
9.4.1 Estimating Profit Margin 209
9.4.2 Risk Management 209
Risk Identification 210
Response to Risk and Uncertainties 213
Risk Analysis 214
9.5 Pricing Policy 217
9.5.1 Balanced bid (straight forward method) 217
9.5.2 Unbalanced bid (Loading of Rates) 218
9.5.3 Method Related Charge 222
9.6 Exercises 225

CHAPTER 10: PROJECT CONTROL


10.1 Problems that may Arise During Construction 229
10.2 Schedule Updating 230
10.3 Delays Analysis 234
10.3.1 Types of Delays 234
10.3.2 The As-Built Schedule 236
10.3.3 Analysis of Concurrent Delays 237
10.4 Earned Value Management 240
Budgeted Cost of Work Scheduled (BCWS) 241
Budgeted Cost of Work Performed (BCWP) 241
Actual Cost of Work Performed (ACWP) 241
10.5 Exercises 245

REFERENCES 248

vii
CHAPTER 1

INTRODUCTION

1.1 The Need for Project Management

The construction industry is the largest industry in the world. It is more of a service than
a manufacturing industry. Growth in this industry in fact is an indicator of the economic
conditions of a country. This is because the construction industry consumes a wide
employment circle of labor. While the manufacturing industry exhibit high-quality
products, timelines of service delivery, reasonable cost of service, and low failure rates,
the construction industry, on the other hand, is generally the opposite. Most projects
exhibit cost overruns, time extensions, and conflicts among parties. Figure 1.1 is an
example of a complicated project. Table 1.1, also, exhibits some magnificent projects that
suffered from huge cost overruns.

Figure 1.1: Example of a complicated project

Construction Management 1 Dr. Emad Elbeltagi


Table 1.1: Magnificent projects with huge cost overruns
Project Cost overruns (%)
Suez Canal 1,900
Sydney Opera House 1,400
Concorde Supersonic Aeroplane 1,100
Panama Canal 200
Brooklyn Bridge 100
(Source: Mette K. Skamris, 'Economic Appraisal of Large-Scale Transport
Infrastructure Investments', Ph.D dissertation, Aalborg University, 2000).

In general, the construction industry is more challenging than other industries due to: its
unique nature; every project is one-of a kind; many conflicting parties are involved;
projects are constrained by time, money and quality; and high risk.

1.2 The Construction Project

A project is defined, whether it is in construction or not, by the following characteristics:

- A defined goal or objective.


- Specific tasks to be performed.
- A defined beginning and end.
- Resources being consumed.

The goal of construction project is to build something. What differentiate the construction
industry from other industries is that its projects are large, built on-site, and generally
unique. Time, money, labor, equipment, and, materials are all examples of the kinds of
resources that are consumed by the project.

Projects begin with a stated goal established by the owner and accomplished by the
project team. As the team begins to design, estimate, and plan out the project, the
members learn more about the project than was known when the goal was first
established. This often leads to a redefinition of the stated project goals.

Construction Management 2 Dr. Emad Elbeltagi


1.3 The Project Scope and Goals

Project Goal Setting


You can’t hit a target if you don’t know what it looks like. Similarly, you can’t possibly
reach your project’s goal if you don’t know what it is. When you understand how your
project fits in with the broader company direction, it’s time to really pin down your goal.
“But,” you say, “I know exactly what my goal is, because my boss told me.” However, a
set of deliverables isn’t necessarily a goal. On first consideration, you might say the goal
of expanding the railroad westward in the United States was to enable a train to go from
coast to coast. But was it? Perhaps the goal of those railroad barons was not to get a train
to go cross country, but to open up opportunities for commerce in the West. It’s time to
put the same kind of thought to your project’s goal.

Getting your goal straight


Say your project involves training new employees in a new software system. Your goal in
training them on this software could be to:
- Make employees more productive in their jobs.
- Enable employees to better serve customers.
- Create a prototype program with reduced training costs that can be used to reduce
overall training costs across the company.
- Increase employee retention by providing useful on-the-job skills. These goals
suggest different priorities as well as different measurements for gauging the
degree to which your project has succeeded in meeting its goal.

How, exactly, do you go about determining your goal? First, go back to the person who
asked you to take on the project and grill him or her about what’s expected of this project.
Should the training have a measurable impact on job performance, customer satisfaction,
employee retention, or cost of delivery? The answer you get might be that the project
should do all of these things. But think about whether that’s realistic and whether one of
these goals should be paramount in guiding you and your project team. If overall training
program cost reduction is the biggest goal factor, for example, employee productivity

Construction Management 3 Dr. Emad Elbeltagi


might have to take a back seat when you’re making choices along the way. If customer
satisfaction is the real goal, training costs might have to be adjusted accordingly.

Writing a goal statement


After you get more specifics about your project’s goal, it’s a good idea to put your goal in
writing in a goal statement. A goal statement outlines why you’re doing this project and
what you hope to accomplish at the end. You don’t get down to specific deliverables and
parameters in a goal statement. For now focus on the why and the desired result.
Here are a few sample goal statements:
- The goal of the project is to upgrade the shopping cart feature on our Web site to
be easier to use so we can increase online sales by 25 percent.
- Our goal is to reduce human resource workload by 10 percent by offering self-
service information on job benefits on the company intranet.
Using the example of a training project, consider for a moment what such a project might
involve. Are you supposed to write new training materials, hire staff to deliver classes,
analyze the training’s effectiveness, update training materials as needed, and promote the
training internally? In that case, the scope of your project involves managing the entire
creation, delivery, and maintenance of a training program. Or is your goal to simply
create the training materials? Or should your focus be limited to launching the new
training, including promoting it to management and staff? These goals indicate very
different projects, each with its own set of deliverables, tasks, resources, schedule, and
costs. Writing a goal statement helps you focus on such a project from the outset.

Project scope
When you understand your goal, you can begin to define the specific parameters of the
project. This is often referred to as a project’s scope. It is necessary to know that a scope
is not a goal. Take a look again at this goal statement from the previous section: The goal
of the project is to upgrade the shopping cart feature Web site to be easier to use to
increase online sales by 25 percent. A scope statement for this project might read: This
project will involve all the steps to design and implement a new shopping cart feature
(but does not include maintaining or refining it once launched). The cost of the project

Construction Management 4 Dr. Emad Elbeltagi


will not exceed $25,000 and implementation must be completed before October 1 to
accommodate holiday sales traffic. The new shopping cart feature should help to increase
sales by allowing customers more options to review their orders, give them more frequent
opportunities to shop for more items after they have added a product to the cart, and
allow them to save their cart contents and come back to complete the sale at a future date.
The new feature must function on our existing Web technology infrastructure.

Writing a scope statement


Scope statements define both what a project will involve and what it will not involve. In
our example, the scope statement specifies that the maintenance of the shopping cart,
once launched, will be handled by other project team. You typically get into specifics
about the project budget, timeframe, and deliverables in a scope statement. You shouldn’t
include every single detail, but you should have enough information that a project team
can understand the most important parameters of the project. Together, a goal statement
and a scope statement are two valuable tools for focusing yourself and your team and
keeping you on track as you proceed. If you take the next logical step in pinning down
your project at the outset, at this point you would create what’s called a project charter.
This would include specifying a project name, getting authorization in writing to begin
the project as of a certain date and to draw on a specified budget, creating a list of
responsibilities, and having those with an interest in the project (called stakeholders) sign
off giving you authority to run the project. You can use your goal and scope statements to
help you obtain the various pieces of your project charter.

Breaking Your Project into Phases


How does all this goal and scope analysis relate to Project? When you start a new project
schedule, one of the first things you will do is to enter individual tasks. Knowing your
goal and scope helps you to identify the steps you should be performing to accomplish
them. Before you create your first task, you should probably begin to think beyond the
scope of your project to more detailed project parameters. These parameters help you
determine what tasks to include in your project. For example, you might consider:

Construction Management 5 Dr. Emad Elbeltagi


- Deliverables: These are tangible products, services, or results that you’ll produce
during your project. Somewhere in your project should be tasks that reflect the
delivery of each deliverable.
- Key Dates: In addition to the project end date, do you have to meet other key
dates along the way?
- Completion Criteria: How will you know when you’re done? Do you start up
the new service and that’s it, or do you have to test it for a week before your job is
done? Knowing your completion criteria gives your team something specific to
aim for and helps you create the last phase of your project.
- Expectations: Knowing what you expect from your team, management, and
yourself can help you identify some tasks. If you expect your team to hold a
quarterly debriefing meeting and submit a progress report, you might include such
a task in your project. If you expect management to sign off on a prototype, a task
such as Prototype Approval is logical.
- Potential Risks: Identifying potential problem areas can help you build in some
checks and balances to help avoid or minimize them. For example, you may
create tasks that contain terms such as Q&A, Testing, Review, Debrief, and
Revise to monitor or fix problems along the way.

1.4 The Project Life-Cycle

The acquisition of a constructed facility usually represents a major capital investment,


whether its owner happens to be an individual, a private corporation or a public agency.
Since the commitment of resources for such an investment is motivated by market
demands or perceived needs, the facility is expected to satisfy certain objectives within
the constraints specified by the owner and relevant regulations.

From the perspective of an owner, the project life cycle for a constructed facility may be
illustrated schematically in Figure 1.2. A project is expected to meet market demands or
needs in a timely fashion. Various possibilities may be considered in the conceptual
planning stage, and the technological and economic feasibility of each alternative will be

Construction Management 6 Dr. Emad Elbeltagi


assessed and compared in order to select the best possible project. The financing schemes
for the proposed alternatives must also be examined, and the project will be programmed
with respect to the timing for its completion and for available cash flows. After the scope
of the project is clearly defined, detailed engineering design will provide the blueprint for
construction, and the definitive cost estimate will serve as the baseline for cost control. In
the procurement and construction stage, the delivery of materials and the erection of the
project on site must be carefully planned and controlled. After the construction is
completed, there is usually a brief period of start-up of the constructed facility when it is
first occupied. Finally, the management of the facility is turned over to the owner for full
occupancy until the facility lives out its useful life and is designated for demolition or
conversion.

Of course, the stages of development in Figure 1.2 may not be strictly sequential. Some
of the stages require iteration, and others may be carried out in parallel or with
overlapping time frames, depending on the nature, size and urgency of the project.
Furthermore, an owner may have in-house capacities to handle the work in every stage of
the entire process. By examining the project life cycle from an owner's perspective we
can focus on the proper roles of various activities and participants in all stages regardless
of the contractual arrangements for different types of work.

The project life cycle may be viewed as a process through which a project is
implemented from beginning to end. This process is often very complex; however, it can
be decomposed into several stages as indicated by the general outline in Figure 1.2. The
solutions at various stages are then integrated to obtain the final outcome. Although each
stage requires different expertise, it usually includes both technical and managerial
activities in the knowledge domain of the specialist. The owner may choose to
decompose the entire process into more or less stages based on the size and nature of the
project. Very often, the owner retains direct control of work in the planning stages, but
increasingly outside planners and financial experts are used as consultants because of the
complexities of projects. Since operation and maintenance of a facility will go on long
after the completion and acceptance of a project, it is usually treated as a separate

Construction Management 7 Dr. Emad Elbeltagi


problem except in the consideration of the life cycle cost of a facility. All stages from
conceptual planning and feasibility studies to the acceptance of a facility for occupancy
may be broadly lumped together and referred to as the Design/Construct process, while
the procurement and construction alone are traditionally regarded as the province of the
construction industry.

Figure 1.2: Project life cycle

There is no single best approach in organizing project management throughout a project's


life cycle. All organizational approaches have advantages and disadvantages, depending
on the knowledge of the owner in construction management as well as the type, size and
location of the project. It is important for the owner to be aware of the approach which is
most appropriate and beneficial for a particular project. In making choices, owners should

Construction Management 8 Dr. Emad Elbeltagi


be concerned with the life cycle costs of constructed facilities rather than simply the
initial construction costs. Saving small amounts of money during construction may not be
worthwhile if the result is much larger operating costs or not meeting the functional
requirements for the new facility satisfactorily. Thus, owners must be very concerned
with the quality of the finished product as well as the cost of construction itself. Since
facility operation and maintenance is a part of the project life cycle, the owners'
expectation to satisfy investment objectives during the project life cycle will require
consideration of the cost of operation and maintenance. Therefore, the facility's operating
management should also be considered as early as possible, just as the construction
process should be kept in mind at the early stages of planning and programming. In
summary the project phases can be summarized as follows:

1.4.1 Preconstruction phase

The preconstruction phase of a project can be broken into conceptual planning, schematic
design, design development, and contract documents.
Conceptual design:
- Very important for the owner.
- During this stage the owner hires key consultants including the designer and
project manager, selects the project site, and establish a conceptual estimate,
schedule, and program.
- The owner must gather as much information as possible about the project.
- The most important decision is to proceed with the project or not.

Schematic design:
- During this phase, the project team investigates alternate design solutions,
materials and systems.
- Completion of this stage represents about 30% of the design completion for
the project.

Design development:
- Designing the main systems and components of the project.

Construction Management 9 Dr. Emad Elbeltagi


- Good communication between owner, designer, and construction manager is
critical during this stage because selections during this design stage affect
project appearance, construction and cost.
- This stage takes the project from 30% design to 60% design.

Contract documents:
- Final preparation of the documents necessary for the bid package such as the
drawings, specifications, general conditions, and bill of quantities.
- All documents need to be closely reviewed by the construction manager and
appropriate owner personnel to decrease conflicts, and changes.
- With the contract documents are almost complete; a detailed and complete
cost estimate for the project can be done.

1.4.2 Procurement phase (Bidding and award phase)

- The project formally transits from design into construction.


- This stage begins with a public advertisement for all interested bidders or an
invitation for specific bidders.
- In fast-track projects, this phase overlaps with the design phase.
- If the project is phased, each work package will be advertised and bid out
individually.
- It is very important stage to select highly qualified contractors. It is not wise
to select the under-bid contractors.

1.4.3 Construction phase

- The actual physical construction of the project stage.


- This stage takes the project from procurement through the final completion.
- It is the time where the bulk of the owner’s funds will be spent.
- It is the outcome of all previous stages (i.e., good preparation means smooth
construction).

Construction Management 10 Dr. Emad Elbeltagi


- The consultant will be deployed for contract administration and construction
supervision.
- Changes during construction may hinder the progress of the project.

1.4.4 Closeout phase

- Transition from design and construction to the actual use of the constructed
facility.
- In this stage, the management team must provide documentation, shop
drawings, as-built drawings, and operation manuals to the owner organization.
- The as-built drawings are the original contract drawings adjusted to reflect all
the changes that occurred.
- Assessment of the project team’s performance is crucial in this stage for
avoiding mistakes in the future.
- Actual activity costs and durations should be recorded and compared with that
was planned. This updated costs and durations will serve as the basis for the
estimating and scheduling of future projects.

Figure 1.3 shows the increasing cumulative cost as the projects progresses while the
influence in the project cost and scope decreases.

1.5 Major Types of Construction Projects

In planning for various types of construction, the methods of procuring professional


services, awarding construction contracts, and financing the constructed facility can be
quite different. The broad spectrum of constructed facilities may be classified into four
major categories, each with its own characteristics.

1.5.1 Residential Housing Construction

Residential housing construction includes houses and high-rise apartments. During the
development and construction of such projects, the developers usually serve as surrogate
owners and take charge, making necessary contractual agreements for design and

Construction Management 11 Dr. Emad Elbeltagi


construction, and arranging the financing and sale of the completed structures.
Residential housing designs are usually performed by architects and engineers, and the
construction executed by builders who hire subcontractors for the structural, mechanical,
electrical and other specialty work.

Figure 1.3: Level of influence vs. project duration

The residential housing market is heavily affected by general economic conditions.


Often, a slight increase in total demand will cause a substantial investment in
construction, since many housing projects can be started at different locations by
different individuals and developers at the same time. Because of the relative ease of
entry, many new builders are attracted to the residential housing construction. Hence, this
market is highly competitive, with potentially high risks as well as high rewards.

1.5.2 Institutional and Commercial Building Construction

Institutional and commercial building encompasses a great variety of project types and
sizes, such as schools and universities, medical centers and hospitals, sports facilities,
shopping centers, warehouses and light manufacturing plants, and skyscrapers for offices

Construction Management 12 Dr. Emad Elbeltagi


and hotels. The owners of such buildings may or may not be familiar with construction
industry practices, but they usually are able to select competent professional consultants
and arrange the financing of the constructed facilities themselves. Specialty architects and
engineers are often engaged for designing a specific type of building, while the builders
or general contractors undertaking such projects may also be specialized in only that type
of building.

Because of the higher costs and greater sophistication of institutional and commercial
buildings in comparison with residential housing, this market segment is shared by fewer
competitors. Since the construction of some of these buildings is a long process which
once started will take some time to proceed until completion, the demand is less sensitive
to general economic conditions than that for housing construction.

1.5.3 Specialized Industrial Construction

Specialized industrial construction usually involves very large scale projects with a high
degree of technological complexity, such as oil refineries, steel mills, chemical
processing plants and coal-fired or nuclear power plants. The owners usually are deeply
involved in the development of a project, and prefer to work with designers-builders such
that the total time for the completion of the project can be shortened. They also want to
pick a team of designers and builders with whom the owner has developed good working
relations over the years.

Although the initiation of such projects is also affected by the state of the economy, long
range demand forecasting is the most important factor since such projects are capital
intensive and require considerable amount of planning and construction time.
Governmental regulation such as environmental protection can also influence decisions
on these projects.

1.5.4 Infrastructure and Heavy Construction

Infrastructure and heavy construction includes projects such as highways, tunnels,


bridges, pipelines, drainage systems and sewage treatment plants. Most of these projects

Construction Management 13 Dr. Emad Elbeltagi


are publicly owned and therefore financed either through bonds or taxes. This category of
construction is characterized by a high degree of mechanization, which has gradually
replaced some labor intensive operations.

The engineers and builders engaged in infrastructure construction are usually highly
specialized since each segment of the market requires different types of skills. However,
demands for different segments of infrastructure and heavy construction may shift with
saturation in some segments. For example, as the available highway construction projects
are declining, some heavy construction contractors quickly move their work force and
equipment into the field of mining where jobs are available.

1.6 Construction Projects Participants

1.6.1 The Owner (The Client)

The owner is the individual or organization for whom a project is to be built under a
contract. The owner owns and finances the project. Depending on the owners’
capabilities, they may handle all or portions of planning, project management, design,
engineering, procurement, and construction. The owner engages architects, engineering
firms, and contractors as necessary to accomplish the desired work.

Public owners are public bodies of some kind ranging from agencies from the country
level to the municipal level. Most public projects or facilities are built for public use and
not sold to others. Private owners may be individuals, partnerships, corporations. Most
private owners have facilities or projects built for their own use or to be sold, operated,
leased, or rented to others.

In order to achieve success on a project, owners need to define accurately the projects
objectives. They need to establish a reasonable and balanced scope, budget, and schedule.
They need to select qualified designers, consultants, and contractors.

Construction Management 14 Dr. Emad Elbeltagi


1.6.2 The Design Professionals

Examples of design professionals are architects, engineers, and design consultants. The
major role of the design professional is to interpret or assist the owner in developing the
project’s scope, budget, and schedule and to prepare construction documents. Depending
on the size and sophistication of the owner, the design professional can be part of the
owner’s group or an independent, hired for the project. In some cases design professional
and construction contractor together form a design-build company.

Architect: An architect is an individual who plans and design buildings and their
associated landscaping. Architects mostly rely on consulting engineers for structural,
electrical, and mechanical work.

Engineer: The term engineer usually refers to an individual or a firm engaged in the
design or other work associated with the design or construction. Design engineers are
usually classified as civil, electrical, mechanical depending upon their specialty. There
are also scheduling, estimating, cost, and construction engineers.
Engineering-Construction Firm: An engineering-construction firm is a type of
organization the combines both architect/engineering and construction contracting. This
type of company has the ability of executing a complete design-build sequence.

1.6.3 The Construction Professionals

The constructions Professional are the parties that responsible for constructing the
project. In traditional management where the owner, design professional, and contractors
are separate companies, the contractor would be termed a prime contractor. The prime
contractor is responsible for delivering a complete project in accordance with the contract
documents. In most cases, the prime contractor divides the work among many specialty
contractors called subcontractors as shown in Figure 1.4.

Construction Management 15 Dr. Emad Elbeltagi


Owner

Contract

Prime contractor
First sub-
contractor Contract

Civil Mechanical Electrical


Contract
Second sub-
contractor Plumbing Elevators

Figure 1.4: Contractor hierarchy

1.6.4 The Project Manager

The project manager is the individual charged with the overall coordination of the entire
construction program for the owner. These include planning, design, procurement, and
construction. Among his/her duties:

- Clear definitions of the goals of the project.


- Investigate alternative solutions for the problems.
- Develop a detailed plan to make the selected program reality.
- Implement the plan and control the project.

Construction Manager: The construction manager is a specialized firm or organization


which administrates the on-site erection activities and the consulting services required by
the owner from planning through design and construction to commissioning. The
construction manager is responsible for design coordination, proper selection of materials
and methods of construction, contracts preparation for award, cost and scheduling
information and control.

Construction Management 16 Dr. Emad Elbeltagi


1.7 Exercises

1. State if True (T) or False (F):


a. The amount of information that a project manager must consider increases as a
project moves towards completion.
b. Dams, bridges, and highways would be classified as commercial building
projects.
c. In the construction phase of the project, the owner needs to be heavily involved.
d. The construction project must have a defined goal or objective.
e. The construction project must have a defined beginning and end.
f. The main objective of the Owner is to win the job, finish it in a reasonable time,
with maximum profit and reasonable quality.

2. Select the right answer:


I. Site selection and financing would be the responsibility of which project
member.
a. Owner b. Designer
c. Construction project manager d. Subcontractor
II. This Category of projects is often funded by public pounds and is termed
“infrastructure”.
a. Residential b. Commercial building
c. Heavy engineering d. Industrial
III. Which of the following is not a characteristic of a project?
a. Having a specific goal b. Having a defined beginning and end
c. Resources being consumed d. usually being performed only once
e. Never being found outside the construction field
IV. The advertising for contractors and review of contractors’ bids occurs during
which project phase.
a. Procurement b. Design
c. Construction d. Conceptual planning

Construction Management 17 Dr. Emad Elbeltagi


V. As-built drawings, warranties, and operation manuals are all provided to the
owner during which project phase.
a. Design b. Conceptual planning
c. Construction d. Project closeout

VI. As project moves on in time, the ability to change the project


becomes…………difficult and…………expensive.
a. more, less b. less, less
c. more, more d. less, more

3. What are the main types of construction?


4. Briefly describe the project life cycle.
5. “As the project progress, the ability of the change decreases while the cost of the
change increases”, comment on this statement and show your answer using a
schematic diagram.
6. List 10 subcontractors that can be engaged in building project.

Construction Management 18 Dr. Emad Elbeltagi


CHAPTER 2

CONTRACT STRATEGY

At the early stage of a project and once a project manager is selected, the main issue that
faces the owner is to decide on the contract strategy that best suits the project objectives.
Contract strategy means selecting organizational and contractual policies required for the
execution of a specific project. The development of the contract strategy comprises a
complete assessment of the choices available for the management of design and
construction to maximize the likelihood of achieving project objectives. The scope of
such contracts is very wide, from a simple purchase of standard article to multi-million-
pound projects. The size and complexity of the contract matter vary accordingly.

A proper contract strategy for a project involves five key decisions:


- Setting the project objectives and constraints
- Selecting a proper project delivery method
- Selecting a proper contract form / type
- Contract administration practices

2.1 What is a Contract

A contract is defined as: "an agreement made between two or more parties which is
enforceable by law to provide something in return for something else from a second
party". Contracts can be very simple or they may be very long and complicated legal
documents. When a contract is properly set-up it is legally binding upon. The two parties
are expected to perform the various obligations they have undertaken, as expressed in a
mutually agreed set of contract documents. A contract therefore, is necessary to protect
both client and contractor. According to its simple definition, a contract is an agreement

Construction Management 19 Dr. Emad Elbeltagi


enforceable at law, but not all agreements are contracts. Some elements must be present
before an agreement becomes a contract. These elements are:

- Competent Parties: For an agreement to be a contract, there must be two or more


competent parties. In order to be considered competent, a part must have a certain
legal standing.

- Proper Subject Matter: For the subject matter of a contract to be proper, the first
requirement is that it was be clearly defined as to the rights and obligations of
each party. Second, the purpose of the contract must not violate the law.

- Consideration: There must be a lawful and valuable consideration given b both


parties. A consideration often called "Something for Something." A consideration
must, also, be possible.

- Agreement: For valid contract, there must be a mutual agreement. An agreement


is considered to have been reached when an offer made by one party is accepted
by the second party. Both parties must wish and intend their bargain to be
enforceable by law.

- Proper Form: The terms of a contract must be written so that both parties are
very sure of what their rights and responsibilities are.

- Consent of the Parties: The agreement must be free from: Misrepresentation,


Duress Undue influence, etc.

The main steps to be taken before placing contract are presented in Fig. 2.1.

2.2 Selection of Contract Type

The selection of contract type to be used for a construction project is made by the owner,
acting upon the advice of his Engineer and his legal advisor. The selection must meet the
owner Objectives and takes into account the constraints that might relate to the project.
Consultants and contractors should be fully informed by the project objectives and
constraints. The scope and the nature of the project will primarily affect the selection of
type of contract.

Construction Management 20 Dr. Emad Elbeltagi


Identify Project

Prepare Scope

Select Contract

Prepare Tender Documents

Invite Tenderes

Tender Meetings

Revise and Evaluation

Place Contract

Fig. 2.1: Steps of contracting process

2.2.1 Project objectives

The client will have a number of overall objectives. These objectives may be of primary
and/or secondary importance. Primary objectives include functional performance, time
objectives, and cost objectives.

a. Project Scope (performance): The project scope defines the extent or the area that the
contract covers. Any additions or omissions during the life of the project will increase
or decrease the quantity of work involved. Likewise, any changes in design must be
discussed carefully to establish whether or not they are likely to affect the scope of the
project.

b. Time: The scope and time are closely interrelated. Decisions must often be made on
the effect of increasing or decreasing scope on time. If the completion date of a project
is critical, then increasing scope will call for an accelerated program. The extra cost
associated with this acceleration must be quantified.

Construction Management 21 Dr. Emad Elbeltagi


c. Price: The cost of a project is closely related to scope and time. The effect of the
contract on price, and the various incentives and penalties that can help to keep price
steady must be discussed and clearly defined.

On the other hand, secondary objectives could arise on a construction project and would
exert a major influence over contract strategy decisions Examples of secondary
objectives are:

• Allocation and payment for risk.

• Training of the client's staff.

• Transfer of technology.

• Involvement of contractor in design.

• Involvement of client in contract management.

• Choice of labor-incentive construction.

• Use of local material and resources.

• Protection of the environment.

2.2.2 Project constraints

All construction projects have constraints that influence the achievement of the project
objectives. These constraints should therefore, be considered when choosing an
appropriate contract strategy. There are a variety of constraints and these are examples:
• Availability of funds.

• Availability of contractual incentives.

• Method of tendering.

• Project location.

• Target dates of the project.

• Possibility of design changes.

• Availability of resources.

• Seasonal working.

• Number of contractors willing or able to tender.

• Inflation.

Construction Management 22 Dr. Emad Elbeltagi


Factors influencing contract choice

Three main factors influence the choice of a given contract including: the incentive, risk
sharing and the flexibility.

2.3 Project Delivery Methods

The project delivery method translates what project parties are involved in the project and
how they interact with each other and called also project organizational structure. The
choice of an organizational structure should be related to project objectives and
constraints. It can be facilitated considering the following factors:
• Size and nature of the work packages within the project.

• Selection of the design team form in-house resources external consultants or

contractors.
• Process of supervision of construction.

• Restrictions upon using combination of organizational structures within the

project.
• Expertise which the client wishes to commit to the project.

When plans are completed and the owner is interested in securing the low price, the use
of competitive bids is suggested. The competitive bidding results in the type of contract
that many are familiar with.

A negotiated contract should be used when construction should start before plans are
completed or when the many unknown factors of the project make an accurate estimate
impossible. When many changes are expected and when inspection and supervision
cannot be done efficiently, the negotiated type of contract should be used. The various
project delivery methods are summarized as follows:

2.3.1 Traditional approach

This is the most common approach in civil engineering projects in which the design has
to be completed before construction can start. Design and construction are usually

Construction Management 23 Dr. Emad Elbeltagi


performed by two different parties who interact directly and separately with the owner.
The pros and cons of this approach are summarized as follow:
Advantages:
- Price competition
- Total cost is known before construction starts
- Well documented approach used in most government projects.
Disadvantages
- Long time
- Design does not benefit from construction expertise
- Conflict between owner, contractor and A/E

Therefore, this method is fine in many cases where the project is clearly definable, design
is completed, time need not be shortened, and changes are unlikely to occur during
construction.

2.3.2 Direct labor

In this approach, owner organization performs both the design and construction using its
in-house labor force.
- Used by large authorities
- The owner performs both the design and the construction
- May use consultants for some specialized designs
- Most suitable for small projects
- Can be used when expertise are available
- Low risk projects
- Inadequate scope definition

2.3.3 Design-build

In this approach, a single organization is responsible for performing both design and
construction and, in some cases, providing certain “know-how” for the project. The pros
and cons of this approach are summarized as follow:
Advantages:

Construction Management 24 Dr. Emad Elbeltagi


- One contract that may include know-how
- Minimum owner involvement
- Used for fast-track projects in order to reduce time
- Co-ordination between design and construction and easier in implementing
the changes
Disadvantages
- Cost may not be known until end of the construction
- High risk to contractor and more cost to owner
- Design-build company may reduce quality to save cost

The use of this approach, therefore, should be considered when contractors offer
specialized design/construction/know-how expertise or when design is strongly
influenced by the method of construction.

2.3.4 Turnkey

This approach is similar to the design-build approach but with the organization being
responsible for performing both design, construction, know-how (if any), and project
financing. Owner payment is then made at the completion (when the contractor turns over
the “key”). An example is franchise projects in which a new branch of a restaurant chain
needs to maintain the same design, construction quality, and food service quality.

2.3.5 Build-operate-transfer (BOT)

In this approach, a business entity is responsible for performing the design, construction,
long-term financing, and temporary operation of the project. At the end of the operation
period, which can be many years, operation of the project is transferred to the owner.
This approach has been extensively used in recent years and is expected to continue. An
example of its use is in express routes and turnpikes. A consortium of companies shares
the cost (design, construction, financing, operation, and maintenance) and the profits
gained from user fees, for a stipulated number of years. Afterwards, the project returns to
the government to become publicly owned. This approach has also been used extensively

Construction Management 25 Dr. Emad Elbeltagi


in large infrastructure projects financed by the World Bank in parts of the world that
cannot afford the high investment cost of such projects.

2.3.6 Professional construction management (PCM)

In this approach, the owner appoints a PCM organization (also known as Construction
Management organization) to manage and coordinate the design and construction phases
of a project using a Teamwork approach. The design may be provided by specialist
design firms and in some cases by the PCM organization. With high level of
coordination between the participants, innovative approaches of overlapping design and
construction (i.e., fast tracking) can be adopted. The PCM organization aims at holding a
friendly position similar to that of the consultants in the traditional approach.

The services offered by the PCM organization overlap those traditionally performed by
the architect, the engineer, and the contractor. This may include: management and
programming of design; cost forecasting and financial arrangements; preparation of
tender documents; tender analysis and selection of contractors; selection of methods of
construction; recommendations on construction economics; planning and scheduling
construction works; materials procurement and delivery expedition; provision for site
security, cleanup, and temporary utilities; supervision of control of construction
contractors; construction quality assurance; cost control; costing of variations and
assessment of claims; and certification of interim and final payments to contractors. The
use of PCM approach, therefore, should be considered when there is a need for time
saving, flexibility for design changes is required, and owner has insufficient management
resources.

2.3.7 Contractual relationships

Within each project delivery method, the contractual relationships among the project
participants can take various arrangements and the owner needs to make a decision
regarding the proper arrangement that suits the project and the parties involved. The
different contractual relationships associated with various project delivery methods are
illustrated in Figs. 2.2 (A represents services and $ contractual relationships).

Construction Management 26 Dr. Emad Elbeltagi


Direct force Traditional Design-build (turn-key)

Fig. 2.2: Contractual relationships

Construction Management 27 Dr. Emad Elbeltagi


2.4 Types of Contracts

There are many types of contracts that may be used in the construction industry.
Construction contracts are classified according to different aspects. They may be
classified according to the method of payment to the contractor. When payment is based
on prices which submitted by the contractor in his tender, they are called cost-based
contracts. Examples are cost-reimbursable and target cost contracts. Contracts may be
classified in the point of view of the risk involved. The range of risk runs from a fixed-
price contract to a totally non-risk cost-reimbursable contract at the other end.

2.4.1 Lump-sum contract

A single tendered price is given for the completion of specified work to the satisfaction of
the client by a certain date. Payment may be staged at intervals on the completion. The
contract has a very limited flexibility for design changes. The tendered price may include
high level of financing and high risk contingency. Where considerable risk has been
places with the contractor, this contract may lead to cost cutting, trivia claims, or
bankruptcy. Contract final price is known at tender. A lump-sum contract would seem to

Construction Management 28 Dr. Emad Elbeltagi


prevent risks for the client where in fact it just changes them. An important risk t the
client is that of not receiving competitive bids from desirable contractors who may avoid
a high-risk lump-sum contract.

This contract may be used for a turnkey construction. It is appropriate when work is
defined in detail, limited variations are expected, level of risk is low and quantifiable, and
client does not wish to be involved in the management of his project.

2.4.2 Admeasurement contract

In this type of contracting, items of work are specified in Bills of Quantities or Schedule
of Rates. The contractor then specifies rates against each item. The rates include risk
contingency. Payment is paid monthly for all work completed during the month. The
contract offers a facility for the client to introduce changes in the work defined in the
tender documents. The contractor can claim additional payment for any changes in the
work content of the contract. Claims resolution is very difficult because the client has no
knowledge of actual cost or hidden contingency. Tender price is usually increased by
variations and claims. Two forms of admeasurement contract are usually used: bill of
quantities and schedule of rates.

Bill of Quantities Contract: Tenderers enter rates against each item of the estimated
quantities of work. The quantities are re-measured during the course of the contract,
valued at the tendered rates and the contract price adjusted accordingly.
Schedule of Rates Contract: It contains inaccurate quantities of work, possibly
with upper and lower probable limits. Therefore, it is common for separate rates to
be quoted for labour, plant, and materials. The contract price is derived by
measuring the man-hours, plant-hours and the quantities of materials actually
consumed, and then pricing them at the tendered price. This contract is best
suitable for repetitive works.

Construction Management 29 Dr. Emad Elbeltagi


The admeasurement contract is well understood and widely used. It can be used when
little or no changes are expected, level of risk is low and quantifiable, and when design
and construction need to be overlapped.

2.4.3 Cost-reimbursable contract (cost-plus contract)

The contractor is reimbursed for actual cost plus a special fee for head office overheads
and profit, no special payment for risk. Payment may be made monthly in advance. The
contract involves a high level of flexibility for design changes. Final price depends on
changes and extent to which risks materialize. The contractor must make all his records
and accounts available for inspection by the client or by some agreed third party. The fee
may be a fixed amount or a percentage of actual costs. This contract has no direct
financial incentives for the contractor to perform efficiently. It may be used when it is
desirable for design to proceed concurrently with construction and when the client wishes
to be involved in contract management.

2.4.4 Target cost contract

Cost targets may be introduced into cost-reimbursable contracts. In addition to the


reimbursement of actual cost plus percentage fee, the contractor will be paid a share for
any saving between target and actual cost, while the fee will be reduced if actual cost
exceeds the target. The target figure should be realistic and the incentive must be
sufficient to generate the desired motivation. Specified risk' can be excluded from the
tendered target cost. When these occur, the target cost is adjusted accordingly and the
client pays the actual cost incurred by the contractor. The target may also b' adjusted for
major changes in work and cost inflation. This contract can be used in the same
circumstances as the cost-plus contract.

A brief summary of the level of risk exposed by each of the discussed contract forms is
illustrated in Fig. 2.3. As shown in the figure, competitive bidding contracts (Lump Sum
and Unit Price) are among the top risky contracts to contracts and thus present a

Construction Management 30 Dr. Emad Elbeltagi


challenge in estimating their cost and schedule at the bidding stage and before a
commitment is made.

Fig. 2.3: Level of risk associated with various contracts

2.4.5 Time and material (T&M) contracts

T&M contracts are a hybrid type of contractual arrangement that contains aspects of both
cost-reimbursable and fixed-price-type arrangements. T&M contracts resemble cost-type
arrangements in that they are open ended, because the full value of the arrangement is not
defined at the time of the award. Thus, T&M contracts can grow in contract value as if
they were cost-reimbursable-type arrangements. Conversely, T&M arrangements can also
resemble fixed-unit arrangements when, for example, the unit rates are preset by the
buyer and seller, as when both parties agree on the rates for the category of "senior
engineers."

Most appropriate when the buyer wants to be more in control. It is also used in an
emergency to begin work immediately when a scope of work has not yet been completed.
Not possible at time of placing contract to estimate extent or duration of the work, or
anticipated cost, with any degree of confidence.

2.5 Contract Administration

As it was discussed in the previous sections, there is variety of types of contracts used in
civil engineering projects. Each type has its specific characteristics. Contracts may be
prepared under the heading of one type but could include characteristics of more than a

Construction Management 31 Dr. Emad Elbeltagi


single type. Many professional societies and government agencies have done a great deal
toward the standardization of construction contracts such that the general form and
content are well established for the various types of construction that may arise.

2.5.1 Contract documents

The contract is defined by the contract documents, which are developed from the tender
documents. In a logical order, these documents refer to the following subjects:

• Input from the client (task description).

• Output of the contract (specifications, results to be achieved).

• Prices for the contractor's contribution.

• Responsibilities and procedures (liability, resources provided, time schedule,

payment conditions, change procedures, etc).

Contract documents are usually arranged according to the following sequence:


• General (for any project).

• Special (for a specialty area of the project).

• Supplementary (unique to a given project).

• Additional (during bidding or negotiation).

• Agreement form (for singing very important and particular clauses).

• Modifications (during contract fulfillment).

The complete contract agreement usually consists of the following documents:


• Conditions (general, special, supplementary).

• Drawing and specifications.

• Addenda.

• Agreement form.

• Modifications.

The most important document from the legal point of view is the agreement. It is
sometimes called the contract. Since so many documents are included as contract

Construction Management 32 Dr. Emad Elbeltagi


documents, the agreement is the better term for this particular one. The form of the
agreement can be standardized and used for many projects, or a unique document can be
prepared for each project. The standard form of agreement prescribed by the American
Institute of Architects has proved to be satisfactory and has been used on many building
projects with good results. The form followed for non-building projects is often more
varied. Man: agencies have own standard forms, which are used on all their projects.

Information usually included in the agreement are of three parts. The first part is a short
introductory paragraph which defines the parties, gives the date of the agreement, and
state that each party agrees to what follows. The second part contains the elements of
contract and defines the work to be undertaken. The final paragraph confirms the
agreement and provides space for signatures of the parties. Thus, the agreement usually
composed of the following articles:
1. A short introductory paragraph.
2. Scope of the work.
3. Time of completion.
4. Contract documents.
5. Performance bond.
6. Contractor's insurance.
7. Owner's insurance.
8. Laws, regulations and permits.
9. Payments.
10. Extensions of time.
11. Changes in the work.
12. Owner's right to terminate the work.
13. Contractor's right to terminate the work.
14. Confirmation and signatures.

2.5.2 Conditions of contract

The conditions of a contract are rules by which the execution of the contract is to be
governed. They set-out the responsibilities, rights, and liabilities of the two parties. They

Construction Management 33 Dr. Emad Elbeltagi


also set-out the actions to be taken by the parties if and when certain eventualities should
arise. No two civil engineering contracts are similar. Probably, no two construction
contracts are truly the same. Therefore identical conditions of contracts are not likely to
be required. However, for work of a similar type, certain conditions will apply for the
vast majority of cases. It follows that a standard form of conditions for a given type of
work will remove the necessity of thinking out and drafting new sets of conditions for
every new contract. By taking the standard form and modifying it to suit the requirements
of a particular contract, time and effort will be saved. The terms of a contract legally fall
into two categories:

Conditions: They are terms expressing matters basic to the contract. A failure to perform
the requirements of a condition is a fundamental breach of an essential obligation giving
the aggrieved party the right to:
1. End the contract and claim damages, or
2. Continue the contract and claim damages.

Warranties: They deal with matters not of the essence of the contract, being subsidiary
to the main purposes for which the parties contracted. An example of a warranty is where
a nominated sub-contractor warrants that the work will be carried-out to specific
standards. The conditions of a contract usually comprise the following:
1. A standard form of general conditions of contract appropriate to the natures of the
work involved.
2. A series of amendments to the forging in order to adjust them to the
circumstances of the actual contract concerned.
3. A number of special conditions, which deal with matters peculiar to the contract
and not dealt with by the standard.

2.5.3 The standard (general) forms of conditions of contract

Standard forms are prepared jointly by professional bodies and organizations representing
contractors or by large organizations and public bodies to suit their own circumstances.
The intention is that a common approach by the parties to all contracts will be achieved
and standard interpretations of risks and responsibilities involved. There are a number of

Construction Management 34 Dr. Emad Elbeltagi


standard forms of conditions of contract used in civil engineering. The most commonly
used are:
Institute of Civil Engineering (ICE) Conditions of Contract: This document includes the
forms of Tender, Agreement and Bond. It is applicable to all civil engineering
construction works. It is particularly suitable for general civil engineering work which is
predominantly either in the ground or in, or adjacent to, water, and carets for the
attendant risks and claims situations. It is also used, sometimes, for building works and
for mechanical and electrical works where such works are included in a civil engineering
or building contract.

Federation Internationale Des Ingenieure-Conseils (FIDIC): These conditions are, in


effect, international versions of the ICE conditions to which they are closely related. The
fourth edition (1987) of the conditions for works of civil engineering construction
comprises:
Part I: General conditions with forms of tender and agreement.
Part II: Conditions of particular application with guidelines for preparation of Part
II clauses.
It is intended for general use for works where tenderers are invited on an international
basis, but it also suitable to domestic contracts. The objective is to provide a standardized
document which is well-known, internationally recognized and accepted, an adequately
reflects the interests of the parties concerned. The standard (general) forms of conditions
of contract usually cover the general aspects in arranged clauses. For example, FIDIC
general conditions of contract comprise clauses that cover the following items:
• Definitions and interpretations.

• Engineer and Engineer's representative.

• Assignment and sub-contracting.

• Contract documents.

• General obligations.

• Labor, Materials, plant, and workmanship.

• Suspension.

• Commencement and delays.

Construction Management 35 Dr. Emad Elbeltagi


• Defects liability.

• Alternations, Additions and Omissions.

• Procedure of claims.

• Contractors' equipment, temporary works and materials.

• Provisional sums.

• Certificates and payments.

• Remedies.

• Special risks.

• Release for performance.

• Settlement of disputes.

• Notices.

• Default of Employer.

• Changes in cost and legislation.

• Currency and rates of exchange.

Most of the standard forms of conditions of contract contain one or more clauses, which
require completion by the Client/Engineer before the conditions are issued.

2.5.4 Special conditions of contract

Special conditions are new clauses to augment the general conditions of a standard form.
Usually they deal with subjects not touched on by the standard form. It is often simpler to
introduce a special condition than to amend a standard form condition. After a new clause
is written, it must be ensure that no conflict or ambiguity is being introduced. The range
of possible subjects for special conditions is large. They normally deal with the
peculiarities of one contract and one site. There are a variety of choices in which a new
subject may be written:
• In the specification (if it is of technical matter).As special condition of contract.

• In the site regulations (if it deals with site administration or discipline).

• In separate correspondence (if it is not essential to incorporate it into the formal

contract).
The decision to consider a special condition does not depend on importance. The
following are typical examples of subjects for special conditions:

Construction Management 36 Dr. Emad Elbeltagi


• Special terms of payments.

• Patents and licenses.

• Applicable law.

• Official Language.

• Obligatory use of local labor, plant and material.

• Co-operation with contractor on site.

• Fair wages to be paid.

• Union membership of work force.

• Protection and disposal of historic, valuable, archaeological, etc. finds on site.

• Prohibition of access to named places.

• Restrictions on noise levels, dust, fire hazards, etc.

• Control of demolition work, explosives, etc.

• Responsibility for damage to public services.

• Responsibility for payment of local taxes.

2.5.5 Construction claims

A construction claim is a request for payment or time extension to which the contractor
considers him/herself entitled. There are three types under which claims are required:
- Extension of time only.
- Additional cost.
- Both extensions of time and additional cost.

The main reasons for construction claims may include:


• Late possession of site or late provision of working drawings.

• Change of contract start or activities schedule.

• Design change and variation.

• Delays in approval and examining work.

• Work acceleration by the client.

• Late delivery of materials supplied by the client.

• Different ground and/or site conditions.

• Unforeseen events and disasters.

Construction Management 37 Dr. Emad Elbeltagi


2.6 Selecting the Contractor

Selecting key personnel and organizations that will participate in a project is a major step
for the owner and can mean the success or failure of a project. By large, the competitive
bidding process has been the main vehicle for contractors to obtain jobs. The process is
required by law for public projects, which has been the largest percentage of all projects,
except in emergencies such as war or natural disasters. Under this process, a simple
quantitative criterion is used to award the bid to the “lowest responsible bidder”, thus
potentially obtaining the lowest construction cost. The process, however, has its
drawbacks, including: (1) overlooking important criteria such as contractor’s experience
and strength; (2) potentially causing construction delays and problems if the contractor
bids below cost to win the job; and (3) contributing to adverse relationships between the
owner and the contractor. The competitive bidding process encompasses three main
steps: announcement, bid preparation and bid evaluation.

To announce for a project, the owner should have the design completed and a bid
package prepared with all design information. The owner then announces a general call
for bidders or sends a limited invitation to a list of pre-qualified contractors. Through the
limited invitation, the owner organization can reduce potential construction problems by
avoiding unknown contractors who intentionally reduce their bids to win jobs,
particularly if the project requires a certain experience. Owners, therefore, need to
maintain a list of qualified contractors with whom they had successful experience or by
advertising a call for pre-qualification.

2.7 Sub-Contracting

On almost all construction projects, some of the work is sub-contracted to specialty


contractors, known as sub-contractors. The greatest part of the work is sub-contracted on
building projects, with a lesser amount usually sub-contracted on heavy construction
projects. Construction contracts generally have clauses pertaining to sub-contracting.
Such clauses often limit the amount of work to be sub-contracted and generally provide
that the client retains the right to approve sub-contractors. The contractor who employs

Construction Management 38 Dr. Emad Elbeltagi


sub-contractors to carryout part of the works must be totally responsible for their
workmanship, performance, and general behavior on the contract. Any communication
on these aspects should be made between the main contractor and the client. It is the
former's responsibility. Any restrictions on sub-contracting should be made in the tender
documents. If the client wishes a particular sub-contractor to carryout part of the work, he
may then make such a nomination. Nominating sub-contractors in such way is useful
when work of a specialized nature dictates such action. It is common for clients to place
other restrictions on sub-contracting, through confirming the main contractor to a list of
approved contractor. This occurs at the tender stage.

2.8 Exercises

1. State if True (T) or False (F):


a. Contract changes are more likely to occur on a single fixed price contract than
on a cost plus a fee contract.
b. The delivery method that an owner chooses should be in response to the
amount and type of risk that an owner sees in a project.
c. In lump sum contracts, it is allowed to change in the quantity of work
performed within a limit of 25%.
d. In the admeasurement contracts, the item description, quantity, unit of measure,
unit cost and the total cost in the B.O.Q should be cleared.
e. The owner has the ability to know the contractor profit in the unit price
contracts.
f. The direct costs are the summation of the cost of the labor, equipment,
materials, and subcontractors.
g. Overheads include the cost of items which cannot be directly charged to a
specific work element.
h. Loading of rates may be risky to both the contractor and the owner.
i. Contract type has not effect on the project deadline.

Construction Management 39 Dr. Emad Elbeltagi


2. Select the right answer:
I. The advantage(s) of a traditional delivery method is (are):
a. Reduced project time b. Known project cost before construction
c. Non-adversarial relationships between participants d. All of the above
II. The developer of a 40 story high-rise office building desires the shortest
possible construction time. What delivery method would be the best?
a. Traditional b. Design-build
c. B and C only d. Construction project management
d. All of the above
III. Which type of contracted arrangement would be best used when the quantities
of work are difficult to determine in advance?
a. Single fixed price b. Unit price
c. Cost plus a fee d. None of the above
IV. If the contractor considers that the quantity of an item in the BOQ has been
underestimated:
a. He should phone the Client to declare this item
b. He has to use this quantity. c. He can raise the rate of this item.
IV. The contractor may decide to subcontract an item of work in order to:
a. give a chance to another contractor
b. cover lack of specialized resources. c. reduce project cost.

3. Briefly differentiate among the various project delivery approaches (various


organizational structures).
4. Explain what is meant by loading of rates in B.O.Q.
5. What are the main types of construction contracts?
6. Specify the suitable contract type for each of the following construction project:
- A pumping station of standard design.
- Sewer and water lines projects.
- Highways
- Hospitals
- Construction of irrigation canals.
- Bridges.

Construction Management 40 Dr. Emad Elbeltagi


- Repetitive and similar units.
- Military projects.

7. What are the construction contract documents?


8. Explain what is meant by the two terms: “Price-based Contracts” and “Cost-based
Contracts”.
9. Compare the following types of contracts from the point of view of flexibility for
design changes and variations:
- Lump Sum.
- Admeasurement.

- Target cost.

10. Compare the lump sum, admeasurements, and cost plus contracts from the
following point of view:
- Early start to construction.
- Risk sharing.

11. Give three examples of secondary objectives which could affect the selection of
the contract strategy decision.

12. Give two examples of project organizational structure which can be used to
achieve fast-track construction.

Construction Management 41 Dr. Emad Elbeltagi


CHAPTER 3

PROJECT PLANNING

This chapter deals with preparing projects plans in terms of defining: work breakdown
structure, activities, logical relations, durations and activities direct cost. Terminology of
project planning will be presented and discussed. Project network representation using
different graphical methods including: activity on arrow and activity on node are
presented.

3.1 Introduction

Planning is a general term that sets a clear road map that should be followed to reach a
destination. The term, therefore, has been used at different levels to mean different
things. Planning involves the breakdown of the project into definable, measurable, and
identifiable tasks/activities, and then establishes the logical interdependences among
them. Generally, planning answers three main questions:

What is to be done?
How to do it?
Who does it?

In construction, for example, plans may exist at several levels: corporate strategic plans,
pre-tender plans, pre-contract plans, short-term construction plans, and long-term
construction plans. These plans are different from each other; however, all these plans
involve four main steps:

Construction Management 42 Dr. Emad Elbeltagi


- Performing breakdown of work items involved in the project into activities.
- Identifying the proper sequence by which the activities should be executed.
- Activities representation.
- Estimating the resources, time, and cost of individual activities.

Detailed planning for tendering purposes and the preparation of construction needs to be
conducted through brainstorming sessions among the planning team. The inputs and
outputs of the planning process are shown in Figure 3.1.

Contract information Activities


Drawings Relationships among activities
Specifications Method statement

OUTPUTS
INPUTS

Available resources Responsibility


PLANNING
Bills of quantities Reporting levels
Site reports Project network diagram
Organizational data Activities duration
Construction methods Activities cost

Figure 3.1: Planning inputs and outputs

Planning requires a rigorous effort by the planning team. A planner should know the
different categories of work and be familiar with the terminology and knowledge used in
general practice. Also, the planning tem should seek the opinion of experts including
actual construction experience. This helps produce a realistic plan and avoids problems
later on site.

3.2 Project Planning Steps

The following steps may be used as a guideline, or checklist to develop a project plan:
1. Define the scope of work, method statement, and sequence of work.
2. Generate the work breakdown structure (WBS) to produce a complete list of
activities.
3. Develop the organization breakdown structure (OBS) and link it with work
breakdown structure o identify responsibilities.

Construction Management 43 Dr. Emad Elbeltagi


4. Determine the relationship between activities.
5. Estimate activities time duration, cost expenditure, and resource requirement.
6. Develop the project network.

3.2.1 Work Breakdown Structure (WBS)

The WBS is described as a hierarchical structure which is designed to logically sub-


divide all the work-elements of the project into a graphical presentation. The full scope of
work for the project is placed at the top of the diagram, and then sub-divided smaller
elements of work at each lower level of the breakdown. At the lowest level of the WBS
the elements of work is called a work package. A list of project’s activities is developed
from the work packages.

Effective use of the WBS will outline the scope of the project and the responsibility for
each work package. There is not necessarily a right or wrong structure because what may
be an excellent fit for one discipline may be an awkward burden for another. To visualize
the WBS, consider Figure 3.2 which shows a house construction project.

House

Civil Plumping Electrical

Foundations Walls/Roof Piping H/C Water Wiring Fittings

Figure 3.2: WBS and their description

As shown in Figure 3.2, level 1 represents the full scope of work for the house. In level 2,
the project is sub-divided into its three main trades, and in level 3 each trade is sub-
divided to specific work packages. Figure 3.3 shows another example for more detailed
WBS, in which the project WBS is divided into five levels:

Construction Management 44 Dr. Emad Elbeltagi


Level 1 Gas development project

Level 2 Recovery unit 300 Process unit 400

Level 3 Train 2 Train 1 Gas treating Separation and stabilization

Level 4 Instrumentation Structural steel Civil Piping

Piping
Level 5 fabrication

Figure 3.3: Five levels WBS

Level 1: The entire project.


Level 2: Independent areas.
Level 3: Physically identifiable sections fully contained in a level 2 area, reflect
construction strategy.
Level 4: Disciplines set up schedule.
Level 5: Master schedule activities, quantity, duration.

Example 3.1:

The WBS for a warehouse is as follow:

For more details, another two levels (third and fourth levels) can be added as shown
below:

Construction Management 45 Dr. Emad Elbeltagi


Accordingly, a complete WBS for the warehouse project can be shown as follow
(Figure 3.4):

Figure 3.4: Warehouse project WBS

Construction Management 46 Dr. Emad Elbeltagi


WBS and organizational breakdown structure (OBS)

The WBS elements at various levels can be related to the contractor’s


organizational breakdown structure (OBS), which defines the different
responsibility levels and their appropriate reporting needs as shown in Figure 3.5.
The figure, also, shows that work packages are tied to the company unified code of
accounts. The unified code of accounts allows cataloging, sorting, and summarizing
of all information. As such, the activity of installing columns formwork of area 2,
for example, which is the responsibility of the general contractor’s formwork
foreman, has a unique code that represents all its data.

WBS coding

A project code system provides the framework for project planning and control in
which each work package in a WBS is given a unique code that is used in project
planning and control. The coding system provides a comprehensive checklist of all
items of work that can be found in a specific type of construction. Also, it provides
uniformity, transfer & comparison of information among projects. An example of
this coding system is the MasterFormat (Figure 3.6) which was developed through a
joint effort of 8 industry & professional associations including: Construction
Specifications Institute (CSI); and Construction Specifications Canada (CSC).
Figure 3.7 shows an example of the coding system using a standardize system as the
MasterFormat. The Master format is divided into 16 divisions as follows:
1) General Requirements.
2) Site work.
3) Concrete.
4) Masonry.
5) Metals.
6) Woods & Plastics.
7) Thermal & Moisture Protection.
8) Doors & Windows.
9) Finishes.

Construction Management 47 Dr. Emad Elbeltagi


10) Specialties.
11) Equipment
12) Furnishings.
13) Special Construction.
14) Conveying Systems.
15) Mechanical.
16) Electrical.

WBS (Work elements)

Project

Area 1 Area 2 Area 3 ……

Beams Columns Slabs ……


superintendent

Formwork Reinforcement Concreting ……


Electrical
Subcontractor
OBS (Responsibility & reporting)

Concrete
foreman
B
Project manager

superintendent superintendent

Formwork
contractor

Control account
foreman
General

Civil
Subcontractor

foreman
Rebar
Mechanical
A

Figure 3.5: WBS linked to the OBS

3.2.2 Project Activities

The building block (the smallest unit) of a WBS is the activity, which is a unique unit of
the project that has a specified duration. An activity is defined as any function or decision
in the project that: consumes time, resources, and cost. Activities are classified to three
types:

Construction Management 48 Dr. Emad Elbeltagi


Figure 3.6: MasterFormat coding system

Construction Management 49 Dr. Emad Elbeltagi


Production activities: activities that involve the use of resources such as labor,
equipment, material, or subcontractor. This type of activities can be easily
identified by reading the project’s drawings and specifications. Examples are:
excavation, formwork, reinforcement, concreting, etc. each production activity
can have a certain quantity of work, resource needs, costs, and duration.

Procurement activities: activities that specify the time for procuring materials or
equipment that are needed for a production activity. Examples are: brick
procurement, boiler manufacturing and delivery, etc.

Management activities: activities that are related to management decisions such


as approvals, vacations, etc.

An activity can be as small as “steel fixing of first floor columns” or as large as


“construct first floor columns”. This level of details depends on the purpose of preparing
the project plan. In the pre construction stages, less detailed activities can be utilized,
however, in the construction stages, detailed activities are required. Accordingly, level of
details depends on: planning stage, size of the project, complexity of the work,
management expertise.

Figure 3.7: An example of an activity coding system

Construction Management 50 Dr. Emad Elbeltagi


Example 3.2:

Figure 3.8 shows a double-span bridge. Break the construction works of the bridge into
activities. The plan will be used for bidding purposes.

Hand rail

Road base right


Road base left
Deck slab
Precast beams

Figure 3.8: Double span bridge

A list of the double-span bridge activities is shown in Table 3.1

Table 3.1: Activities of the double-span bridge


Activity Description
10 Set-up site
14 Procure reinforcement
16 Procure precast beams
20 Excavate left abutment
30 Excavate right abutment
40 Excavate central pier
50 Foundation left abutment
60 Foundation right abutment
70 Foundation central pier
80 Construct left abutment
90 Construct right abutment
100 Construct central pier
110 Erect left precast beams
120 Erect right precast beams
140 Fill left embankment
150 Fill right embankment
155 Construct deck slab
160 Left road base
170 Right road base
180 Road surface
190 Bridge railing
200 Clear site

Construction Management 51 Dr. Emad Elbeltagi


3.2.3 Activities Relationships

In order to identify the relationships among activities, the planning team needs to answer
the following questions for each activity in the project:

- Which activities must be finished before the current one can start?
- What activity(ies) may be constructed concurrently with the current one?
- What activity(ies) must follow the current one?

A circle of activity precedence will result in an impossible plan. For example, if activity
A precedes activity B, activity B precedes activity C, and activity C precedes activity A,
then the project can never be started or completed. Figure 3.9 illustrates the resulting
activity network.

Figure 3.9: Example of a circle of activity precedence

Example 3.3:

Suppose that a site preparation and concrete slab foundation construction project consists
of nine different activities:
A. Site clearing (of brush and minor debris),
B. Removal of trees,
C. General excavation,
D. Grading general area,
E. Excavation for utility trenches,
F. Placing formwork and reinforcement for concrete,
G. Installing sewer lines,
H. Installing other utilities,
I. Pouring concrete.

Construction Management 52 Dr. Emad Elbeltagi


Activities A (site clearing) and B (tree removal) do not have preceding activities since
they depend on none of the other activities. We assume that activities C (general
excavation) and D (general grading) are preceded by activity A (site clearing). It might
also be the case that the planner wished to delay any excavation until trees were removed,
so that B (tree removal) would be a precedent activity to C (general excavation) and D
(general grading). Activities E (trench excavation) and F (concrete preparation) cannot
begin until the completion of general excavation and grading, since they involve
subsequent excavation and trench preparation. Activities G (install lines) and H (install
utilities) represent installation in the utility trenches and cannot be attempted until the
trenches are prepared, so that activity E (trench excavation) is a preceding activity. We
also assume that the utilities should not be installed until grading is completed to avoid
equipment conflicts, so activity D (general grading) is also preceding activities G (install
sewers) and H (install utilities). Finally, activity I (pour concrete) cannot begin until the
sewer line is installed and formwork and reinforcement are ready, so activities F and G
are preceding. Other utilities may be routed over the slab foundation, so activity H (install
utilities) is not necessarily a preceding activity for activity I (pour concrete). The result of
our planning is the immediate precedence shown in Table 3.2.

Table 3.2: Precedence relations for Example 3.3

Activity Description Predecessors


A Site clearing ---
B Removal of trees ---
C General excavation A
D Grading general area A
E Excavation for utility trenches B,C
F Placing formwork and reinforcement for concrete B,C
G Installing sewer lines D,E
H Installing other utilities D,E
I Pouring concrete F,G

Construction Management 53 Dr. Emad Elbeltagi


Example 3.4:

Determine the relationships between activities of the project studied in Example 3.2.

Table 3.3: Solution of Example 3.4

Activity Description Predecessors


10 Set-up site ---
14 Procure RFT ---
16 Procure P.C. Beams ---
20 Excavate left abutment 10
30 Excavate right abutment 10
40 Excavate central pier 10
50 Foundation left abutment 14, 20
60 Foundation right abutment 14, 30
70 Foundation central pier 14, 40
80 Construct left abutment 50
90 Construct right abutment 60
100 Construct central pier 70
110 Erect left P.C. Beams 16, 80, 100
120 Erect right P.C. Beams 16, 90, 100
140 Fill left embankment 80
150 Fill right embankment 90
155 Construct deck slab 110, 120
160 Left road base 140
170 Right road base 150
180 Road surface 155, 160, 170
190 Bridge railing 155
200 Clear site 180, 190

Logical relationship considering resource constraints

For efficient use of resources or in case of constrained resources, it might be beneficial to


consider the resources when determining the logical relationship among the activities that
use the same resources. For example, consider the case of construction a simple project
consists of three units and each unit has three sequential activities (logical relationship).

Construction Management 54 Dr. Emad Elbeltagi


Table 3.4 shows the logical relationship among these activities assuming unconstrained
(resources are available with any quantities) and constrained resources (only one resource
unit is available from each resource type).

Table 3.4: Logical relationships considering constrained and unconstrained


resources
Predecessors Predecessors
Activity description
(unconstrained resources) (constrained resources)
A1 Excavate unit 1 - -
B1 Concreting unit 1 A1 A1
C1 Brickwork unit 1 B1 B1

A2 Excavate unit 2 - A1
B2 Concreting unit 2 A2 B1, A2
C2 Brickwork unit 2 B2 C1, B2

A3 Excavate unit 3 - A2
B3 Concreting unit 3 A3 B2, A3
C3 Brickwork unit 3 B3 C2, B3

Overlap or lag

Overlap between activities (negative lag) is defined as how much a particular activity
must be completed before a succeeding activity may start. The absence of overlap means
that the first activity must finish before the second may start. A negative overlap (lag)
means a delay is required between the two activities (Figure 3.10)

+ve overlap (-ve lag) -ve overlap (+ve lag)

Figure 3.10: Overlap among activities

Construction Management 55 Dr. Emad Elbeltagi


Example 3.5:

This case study is for a small 3 houses project. The main segments of a single house, the
responsibilities, and the logical relationship are identified as follows:

- 11 work packages are involved: A and B (civil work, substructure), C, D, E, and F


(civil work, superstructure), G (electrical, interior), H (electrical, exterior), I
(mechanical, HVAC), J (mechanical, elevator), and K (mechanical, plumbing).
- Substructure is supervised by Ahmed (activity A), and Ali (activity B).
- Superstructure is supervised by Hossam (activities C and F) and Mona (activities D
and E).
- All electrical work is supervised by George.
- HVAC and plumbing are supervised by Adam; elevator work is supervised by
Samy.
- Activities E and F follow activity B.
- Activity C precedes activity G.
- Activity I follows the completion of activity E.
- The predecessors to activity K are activities H and I.
- Activity D follows activity A and precedes activity H.
- Activity J is preceded by activities F and G.

It is required to create a WBS and OBS chart.

Solution

From the available information, the relationship table, the network diagrams, and the
WBS linked to an OBS are formed as shown below (Table 3.5 and Figure 3.11).

Construction Management 56 Dr. Emad Elbeltagi


Table 3.5: Logical relationships of Example 3.5
Activity Predecessors
Start -
A Start
B Start
C Start
D A
E B
F B
G C
H D
I E
J F, G
K H, I
Finish J, K

WBS (Work elements) Project

Civil Elec. Mech

House1 House1 House1

Sub Super

Ahmed A
OBS (Responsibility & reporting)

Ali B G
George
H
Project manager

C
Hossam
F
D
Mona
E I
Adam
K
Samy J

Figure 3.11: WBS and OBS of Example 3.5

Construction Management 57 Dr. Emad Elbeltagi


Types of activities relationships

Four types of relationships among activities can be defined as described and illustrated
below (Figure 3.12). Typically, relationships are defined from the predecessor to the
successor activity.

a) Finish to start (FS). The successor activity can begin only when the current
activity completes.
b) Finish to finish (FF). The finish of the successor activity depends on the finish of
the current activity.
c) Start to start (SS). The start of the successor activity depends on the start of the
current activity.
d) Start to finish (SF). The successor activity cannot finish until the current activity
starts.

a b

c d

Figure 3.12: Types of relationships

3.2.4 Drawing Project Network

A network is a graphical representation of the project activities and their relationships. A


project network is a set of arrows and nodes. Before drawing the network, it is necessary
to ensure that the project has a unified starting and ending point. The need for this start
activity arises when there is more than one activity in the project that has no predecessors
and the end activity is needed when there is more than one activity that has no successors.
Also, networks should be continuous (i.e., each activity except the first and the last has
both preceding and succeeding activities).

Construction Management 58 Dr. Emad Elbeltagi


There are two ways that are commonly used to draw a network diagram for a project:

1. Activity on Arrow (AOA) representation.


2. Activity on Node (AON) representation

Activity on arrow network (AOA)

In this method, the arrows represent activities while the nodes represent the start and the
end of an activity (usually named as events) (Figure 3.13). The length of the arrow
connecting the nodes has no significance and may be straight, curved, or bent. When one
activity depends upon another, both appear on the diagram as two arrows having a
common node.

Activity A Activity B
i j j>i 5 10

A B
5 10 15 B depends on A

A C
5 10 15 C depends on A and B
B
5

15
B B depends on A
C depends on A
A C
5 10 15

5 15
A C B depends on A and B
D depends on A and B
B 10 D
5 15

Figure 3.13: Basic patterns of AOA diagrams

The following are some rules that need to be followed when constructing an AOA
network diagram:

Construction Management 59 Dr. Emad Elbeltagi


- Each activity must have a unique i – j numbers, where i (the number at the tail of
the arrow) is smaller than j (the number at the head of the arrow).
- It is recommended to have a gap between numbers (i.e., 5, 10, 15, etc.). This will
allow for accommodation of missed activities.
- Avoid back arrows.

In some situations, when more than one arrow leave the same node and arrive at another
node, dummy activities must be used. The dummy activity is an activity with zero
duration, consumes no resources, drawn as dashed lines, and used to adjust the network
diagram. A dummy activity is also used when one activity depends upon two preceding
activities and another activity depends only upon one of these two preceding activities as
shown in Figure 3.14.

Activity on node network (AON)

This method is also called the precedence diagram method. In this method, the nodes
represent activities and the arrows represent logical relationships among the activities. If
the arrow starts from the end side of an activity (activity A) and ends at the start side of
another activity (activity B), then A is a predecessor of B (Figure 3.15). AON
representation allows the overlap or lag representation on the relationship arrows
connecting activities.
A C A C
5 15 20 5 20 25
C depends on A and B
B D D depends on B only Dummy
25 B D
10 10 15 30

A A
5 15 5 15

B
Dummy
B
10

Incorrect representation Correct representation

Figure 3.14: Use of dummy activity

Construction Management 60 Dr. Emad Elbeltagi


10 Activity number 20
A B
Activity name

10 20
B depends on A
A B

10 30 40 C depends on A and B
A C D D depends on C

20
B
30
C
10 20 B depends on A
A B C depends on B
D depends on B
40
D

Figure 3.15: Basic patterns of AON diagrams

Comparison between AOA and AON

While both networks can be used to represent a project network, there are some
differences between them:
- There is no need for the use of dummy activities in AON representation.
- AON are more easily to draw and to read.
- In AOA, an activity can only start when all its predecessors have finished.
- AON allows for overlap/lag representation.
- AON allows for the representation of the four types of relationships while AOA
allows only for the finish to start relationship.

Example 3.6:

Construct an AOA and AON networks for the activities listed in Table 3.6.

Construction Management 61 Dr. Emad Elbeltagi


Table 3.6: Data for Example 3.6
Activity Predecessors
A -
B -
C A, B
D C
E C
F D
G D, E

Forming an AOA network for this set of activities might begin be drawing activities A, B
and C as shown in Figure 3.16 (a). At this point, we note that two activities (A and B) lie
between the same two event nodes; for clarity, we insert a dummy activity X and
continue to place other activities as in Figure 3.16 (b). Placing activity G in the figure
presents a problem, however, since we wish both activity D and activity E to be
predecessors. Inserting an additional dummy activity Y along with activity G completes
the activity network, as shown in Figure 3.16 (c).

Figure 3.16: AOA Network for Example 3.6

Construction Management 62 Dr. Emad Elbeltagi


To understand the drawing of the AON, some ordering for the activities may be
necessary. This is done by placing the activities in a sequence step order. A sequence step
may be defined as the earliest logical position in the network that an activity can occupy
while maintaining the logical relationships. In this example, as there are two activities
(activities A and B) has no predecessor, then a start activity is added to have one unified
start activity (Start) for the project. Also, a finish activity (Finish) is added as there are
two activities without successors (activities F and G).

Considering the data given in Table 3.6, sequence step 1 is assigned to the Start activity.
Then, we take all activities on the list one by one and look at their immediate
predecessors and then assign a sequence step that equals the highest sequence step of all
immediate predecessors plus one as given in Table 3.7. After all sequence step numbers
have been assigned, the AON diagram can be drawn.

Table 3.7: Determining the sequence steps

Activity Predecessors Sequence step (SS)


Start - SS(Start)=1
A Start 2=SS(Start)+1
B Start 2=SS(Start)+1
C A, B 3=Highest of [SS(B), SS(A)]
D C 4=SS(C)+1
E C 4=SS(C)+1
F D 5=SS(D)+1
G D, E 5=Highest of [SS(D), SS(E)]
Finish F, G 6= Highest of [SS(F), SS(G)]

AON representation is shown in Figure 3.17, including project start and finish nodes.
Note that dummy activities are not required for expressing precedence relationships in
activity-on-node networks.

Construction Management 63 Dr. Emad Elbeltagi


Sequence step 1 2 3 4 5 6

Figure 3.17: An AON Network

Example 3.7

Draw the AOA and AON networks for the project given in Example 3.5.

Solution

The AOA is given in Figure 3.18 and the AON is given in Figure 3.19 as shown below.

D
10 25
A H

B E I K
5 15 30 40 45

F
C J

G
20 35

Figure 3.18: AOA network

Construction Management 64 Dr. Emad Elbeltagi


A D H K

Start B E I Finish

F
J

C G

Figure 3.19: AON network

3.3 Estimating Activity Duration and Direct Cost

Having defined the work activities, each activity has associated time duration. These
durations are used in preparing a schedule. For example, suppose that the durations
shown in Table 3.8 were estimated for a project. The entire set of activities would then
require at least 3 days, since the activities follow one another directly and require a total
of 1.0 + 0.5 + 0.5 + 1.0 = 3 days.

Table 3.8: Durations and predecessors for a four-activity project

Activity Predecessor Duration (Days)


Excavate trench --- 1.0
Place formwork Excavate trench 0.5
Place reinforcing Place formwork 0.5
Pour concrete Place reinforcing 1.0

All scheduling procedures rely upon estimates of the durations of the various project
activities as well as the definitions of the predecessor relationships among activities. A
straightforward approach to the estimation of activity durations is to keep historical
records of particular activities and rely on the average durations from this experience in
making new duration estimates. Since the scope of activities is unlikely to be identical

Construction Management 65 Dr. Emad Elbeltagi


between different projects, unit productivity rates are typically employed for this purpose.
The duration of an activity may be estimated as:

Activity duration = quantity of work / number of crews x resource output

Typically, the quantity of work is determined from engineering drawings of a specific


project. The number of crews working is decided by the planner. In many cases, the
number or amount of resources applied to particular activities may be modified in light of
the resulting project plan and schedule. Some estimate of the expected work productivity
must be provided. Historical records in a firm can also provide data for estimation of
productivities.

Having defined an activity duration, it means that the planner have already defined the
number of resources that will be employed in a particular activity. Knowing activity
duration and resources employed, it is simple to estimate the activity direct cost. Then,
the three elements of an activity: duration, cost, and resources form what is called
construction method. Some activities can be performed using different construction
methods. Where, its method will have its own resources, cost and duration.

Example 3.8:

If the daily production rate for a crew that works in an activity is 175 units/day and the
total crew cost per day is LE 1800. The material needed for daily work is 4.5 units at LE
100/unit.

a. Calculate the time and cost it takes the crew to finish 1400 units
b. Calculate the total unit cost. Consider an eight hour work day.

Solution

a. Duration (units of time) = Quantity / Production per unit of time x number of crews
= 1400 / 175 x 1 = 8 days
Cost (labor cost) = Duration (units of time) x crew cost per unit of time

Construction Management 66 Dr. Emad Elbeltagi


= 8 days x LE 1800 / day = LE 14400
Total direct cost = Le 14400 + 4.5 units of material x LE 100 / day x 8 days
= LE 18000

b. Unit cost = total cost / quantity


= LE 18000 / 1400 = LE 12.86 / unit

Sometimes the productivity of a specific crew expressed in man-hours/unit not units/day.


For example, if the productivity is said to be 0.5 Man-hour/cubic meters, this means how
long it will take one labor to construct one unit. This way applied to any crew formation
and work hours.

Example 3.9:

What is the duration in days to install 6000 square feet of walls shuttering if:

a. Crew of 2 carpenters is used with output of 200 square feet/day


b. Productivity is measured as 0.008 man-hour/square feet. Number of
carpenters =3, and number of working hours/day = 8 hours

Solution

a. Duration = 6000 / 200 = 3 days


b. Total man-hours needed = 6000 x 0.008 = 48 man-hours (if one man used)
Duration = 48 / 8 = 6 days (if one man used)
Duration using 3 men = 6 / 3 = 2 days

Example 3.10: (use of several resources)

The construction of a reinforced concrete wall involves placing 660 m3 concrete, fixing
50 ton of steel, and 790 m2 of formwork. The following information belongs to the jobs
involved in this activity:
- A 6 man concrete crew can place 16 m3 of concrete/day.
- A steel-fixer and assistant can fix 0.5 ton of reinforcement/day.
- A carpenter and assistant can fix and remove 16 m2 of shuttering/day.

Construction Management 67 Dr. Emad Elbeltagi


Calculate the duration of the activity considering the steel-fixer as the critical resource.

Solution

- using one steel-fixer: duration = 50 / 0.5 = 100 days


- using one carpenter: duration = 790 / 16 = 49.4 days
- using one concreting crew: duration = 660 / 16 = 41.25 days.

Then, for a balanced mix of resources, use 2 steel-fixer crews, one carpenter crew,
and cone concreting crew. Accordingly, the activity duration = 50 / 0.5 x 2 = 50 days.

3.4 Exercises

1. Select the right answer:


I. The elements of construction project planning are:
a. Time b. Resources
c. Cost d. All
II. Which of the following is not a typical activity category?
a. Production b. Procurement
c. Administrative d. None of the above

2. In developing the WBS for a project, level of details depends on: …..,……,……..

3. List four main differences between AOA and AON networks.

4. A small single-story commercial building is to be constructed on the site of an


existing old structure. The exterior and interior walls are of concrete blocks. The
roof is erected from steel members covered with rigid insulation and build-up
roofing. The ceiling is of suspended tile. The floor is a concrete slab on grade
with an asphalt tile finish. Interior finish on all walls is paint. The project has been
broken down into 18 steps with construction time estimate has been made for
each step. These steps are not given in any particular order. Specify the
predecessors of each activity.

Construction Management 68 Dr. Emad Elbeltagi


- Under ground services (water and sewage services), 1 day.
- Exterior walls, 6 days. - Foundations, 3 days.
- Demolition, 2 days. - Roof steel, 2 days.
- Interior walls, 3 days. - Roof finishing, 2 days.
- Floor slab, 3 days. - Floor finishing, 2 days.
- Rough plumbing, 3 days. - Finish plumbing, 4 days.
- Rough electrical, 3 days. - Finish electrical, 3 days.
- Rough carpentry, 2 days. - Finish carpentry, 4 days.
- Ceiling, 3 days. - Windows, 1day.
- Painting, 1day.

5. Prepare a complete plan for the project described below. This project calls for the
contractor to construct a temporary two-span, Bypass Bridge for use while a
permanent bridge is being replaced. The following figure contains a sketch of the
project.

Scope of Work: The Bridge’s substructure will include two abutments and a
midstream pier. The abutments will be constructed by driving a row of timber

Construction Management 69 Dr. Emad Elbeltagi


piles. Heavy planks will be spiked to the shore side of these piles to act as a
retaining wall. A heavy timber will be placed on top of the row of piles as an
abutment cap. The pier will be constructed by driving two rows of timber piles.
Heavy timbers will be fastened on top of these piles to serve as the pier cap. The
superstructure will consist of steel beams supported by the abutment and pier
caps. Timber decking will be secured to the steel beams to serve as the roadway.
Miscellaneous bracing, curbs, and guard rails will be installed to complete the
bridge. It will be necessary to construct an asphalt concrete access road at both
ends of the temporary bridge and to demolish that access road once the bypass
bridge is removed. The scope of this project does not include removal of the
bypass bridge or its access roads.

Planned Work Sequence: Only one pile-driving rig is available. It is not possible
to drive this rig across the existing bridge, and it is not feasible to detour around
the bridge; therefore, all piles must be driven from one side of the stream. All
other equipment needed for construction can cross over the existing bridge. Work
will start with the construction of the access road to the east bank pier. This access
road will not be paved until pile driving is completed. Next, the piles will be
driven for the east bank abutment, and the east bank abutment will be completed.
Then, the midstream piles will be driven from the east bank. When the east bank
abutment is completed and when the caps have been installed on the midstream
pier, the steel beams will be placed for the east span. After the east span decking
is installed, the pile driver can be moved onto the east span and the west bank
piles can be driven. Equipment other than the pile driver can be driven across the
existing bridge to the west bank; therefore, construction of the west bank access
road can be started as soon as the equipment is released from the same task on the
east bank.

Task Definition: The tasks shown in the following table have been defined. Task
durations were estimated on the basis of an eight-hour workday.

Construction Management 70 Dr. Emad Elbeltagi


Estimated Duration
Task No. Description
(days)
1 Survey and layout 1
2 Rough-grade access road on east bank 2
3 Drive timber piles for east abutment 1
4 Construct east abutment 5
5 Drive timber piles for midstream pier 2
6 Complete midstream pier 5
7 Place steel girders east span 2
8 Place timber decking east span 2
9 Drive timber piles west abutment 1
10 Construct west abutment 5
11 Place steel girders west span 2
12 Place timber decking west span 2
13 Rough-grade access road on west bank 2
14 Finish grading access road east bank 1
15 Finish grading access road west bank 1
16 Pave access roads both banks 3
17 Install curbs and gutters on bridge 5
18 Stripe access road 1
19 Erect barricades to site of permanent bridge 1

6. Draw a PDM network for a project with the following activities. Show all steps
including removing redundant relations; and sequence steps.

- Activity B depends on A;
- Activity G follows E, F & D;
- Activity E depends on B and A;
- Activity F can start when D & B are completed;
- Activity C is followed by F and follows A;
- Activity D is dependent upon A and B.

Construction Management 71 Dr. Emad Elbeltagi


7. Consider the following set of activities:

Code Description
A Layout foundation
B Excavation
C Obtain concrete materials
D Place concrete
E Obtain steel reinforcement
F1 Cut and bend reinforcement (part 1)
F2 Cut and bend reinforcement (part 2)
G1 Place reinforcement (part 1)
G2 Place reinforcement (part 2)
H Obtain formwork
I Erect formwork
J Remove formwork
K Clean up

A gang of steelfixers is used to cut and bend reinforcement and another gang is
used for placing reinforcement. The first part of reinforcement can be placed
during formwork erection while the second part should wait for completion of
formwork erection. Tabulate the predecessors of each activity and draw AON
network.

8. For the network below, prepare a table showing a list of immediate predecessors
and immediate successors for each of the activities. Use the i - j node notation for
activities.

5 6

1 3 4 7

Construction Management 72 Dr. Emad Elbeltagi


9. The construction of RC wall involves placing 660 m3 concrete, 50 t of steel, and
790 m2 of formwork. Calculate the duration of the activity using a balanced mix
of the resources if:

- A 6 man concrete gang can place 16 m3 of concrete / day.


- One steelfixer and one assistant can fix 0.5 t of steel / day.
- One carpenter and one assistant can fix and strike 16 m2 / day.

10. Estimate the labor cost for the formwork of a continuous wall footing that has a
quantity of 500 SF. The activity is constructed by crew that has a daily output of
485 SF/day, and consists of: 3 carpenters at rate LE 21.60/hr & 1 building labor at
rate LE 17.15/hr.

11. A construction project has the following activities along with their relationships.
Develop an Activity on Arrow (AOA) network.

I. A is the first activity.


II. B, C and D follow A and can be done concurrently.
III. E and G cannot begin until C is completed, and can be done
concurrently.
IV. F is the immediate successor to activities B and E.
V. H and K run in parallel, and both succeed G.
VI. L succeeds F and H.
VII. J and I are immediate successor activities to D.
VIII. M and N are immediate successor to K and I. However, both M
and N can be performed concurrently.
IX. Q is the last activity and follows O & P. O is the immediate
successor to N and L. Activity P is the immediate successor to M
and J.

Construction Management 73 Dr. Emad Elbeltagi


CHAPTER 4

PROJECT SCHEDULING

In chapter 3, the AOA and AON networks were presented, also the time and cost of
individual activities based were calculated. Yet, however, we do not know how long is
the total project duration. Also, we need to evaluate the early and late times at which
activities start and finish. In addition, since real-life projects involve hundreds of
activities, it is important to identify the group of critical activities so that special care is
taken to make sure they are not delayed. All these statements are the basic objectives of
the scheduling process, which adds a time dimension to the planning process. In other
words, we can briefly state that: Scheduling = Planning + Time.

Scheduling is the determination of the timing of the activities comprising the project to
enable managers to execute the project in a timely manner. The project scheduling id
sued for:
- Knowing the activities timing and the project completion time.
- Having resources available on site in the correct time.
- Making correction actions if schedule shows that the plan will result in late
completion.
- Assessing the value of penalties on project late completion.
- Determining the project cash flow.
- Evaluating the effect of change orders on the project completion time.
- Determining the value pf project delay and the responsible parties.

Construction Management 74 Dr. Emad Elbeltagi


4.1 The Critical Path Method

The most widely used scheduling technique is the critical path method (CPM) for
scheduling. This method calculates the minimum completion time for a project along
with the possible start and finish times for the project activities. Many texts and managers
regard critical path scheduling as the only usable and practical scheduling procedure.
Computer programs and algorithms for critical path scheduling are widely available and
can efficiently handle projects with thousands of activities.

The critical path itself represents the set or sequence of activities which will take the
longest time to complete. The duration of the critical path is the sum of the activities'
durations along the path. Thus, the critical path can be defined as the longest possible
path through the "network" of project activities. The duration of the critical path
represents the minimum time required to complete a project. Any delays along the critical
path would imply that additional time would be required to complete the project.

There may be more than one critical path among all the project activities, so completion
of the entire project could be delayed by delaying activities along any one of the critical
paths. For example, a project consisting of two activities performed in parallel that each
requires three days would have each activity critical for a completion in three days.
Formally, critical path scheduling assumes that a project has been divided into activities
of fixed duration and well defined predecessor relationships. A predecessor relationship
implies that one activity must come before another in the schedule.

The CPM is a systematic scheduling method for a project network and involves four main
steps:

- A forward path to determine activities early-start times;


- A backward path to determine activities late-finish times;
- Float calculations; and
- Identifying critical activities.

Construction Management 75 Dr. Emad Elbeltagi


4.2 Calculations for the Critical Path Method

The inputs to network scheduling of any project are simply the AOA or the AON
networks with the individual activity duration defined. The network scheduling process
for AOA and AON networks, however, is different. To demonstrate these two techniques,
let’s consider a simple 5-activity project, with activity A at the start, followed by three
parallel activities B, C, and D; which are then succeeded by activity E. The AOA or the
AON networks of this example are presented in Figure 4.1. Detailed analysis of theses
AOA or the AON networks are presented in the following subsections. It is noted that the
example at hand involves only simple finish-to-start relationships among activities.

5
B (3) d1

A (3) C (4) E (5)


1 3 9 11
D (6) d2

7
Activity (duration)
i j (a - AOA)

B
(3)

A C E
(3) (4) (5)

Activity
(Duration) D (b - AON)
(6)
Figure 4.1: Network example

4.2.1 Activity-On-Arrow Networks Calculations

The objective of arrow network analysis is to compute for each event in the network its
early and late timings. These times are defined as:

- Early event time (ET) is the earliest time at which an event can occur, considering
the duration of preceding activities.

Construction Management 76 Dr. Emad Elbeltagi


- Late event time (LT) is the latest time at which an event can occur if the project is
to be completed on schedule.

Forward Path

The forward path determines the early-start times of activities. The forward path proceeds
from the most left node in the network (node 1 – Figure 4.2) and moves to the right,
putting the calculations inside the shaded boxes to the left.

Each node in the network, in fact, is a point at which some activities end (head arrows
coming into the node), as shown in Figure 4.3. That node is also a point at which some
activities start (tail arrows of successor activities). Certainly, all successor activities can
start only after the latest predecessor is finished. Therefore, for the forward path to
determine the early-start (ES) time of an activity, we have to look at the head arrows
coming into the start node of the activity. We then have to set the activity ES time as the
latest finish time of all predecessors.

5
B (3) d1

1 A (3) 3 C (4) 9 E (5) 11


D (6) d2

Figure 4.2: Preparation for the forward path

Predecessor 1
Successor 1

Predecessor 2 no.

Predecessor 3 Successor 2

Figure 4.3: A node in an AOA network

Construction Management 77 Dr. Emad Elbeltagi


In this example, the forward path calculations are as follows:
- Start at node 1, the first node of the project, and assign it an early-start time of zero.
Here, all activity times use an end-of-day notation. Therefore, the ES of activity A is
zero means that activity starts at end of day zero, or the beginning of day 1 in the
project.

- Then, move to node 3. This node receives one head arrow, and as such, it has one
predecessor, activity A. Since the predecessor started on time zero and has 3 days
duration, then, it ends early at time 3 (Early-Finish (EF) = Early-Start (ES) + d).
Accordingly, the ES time of all successor activities to node 3 (activities B, C, and D)
is time 3. This value is therefore, put in the shaded box on top of node 3, as shown in
Figure 4.4.

6
3+3=6

Project ES+d=EF 5
start=0 0+3=3
B d1 14
0 3 3 9+5=14
A C E
1 3 9 11
d=3 4 5
D d2 9
6 6+0=6
3+4=7 or
7 9+0=9
9
3+6=9

Figure 4.4: Forward path calculations in AOA networks

- Now, move forward to successor nodes 5, 7, and 9. However, since node 9 is linked
to nodes 5 and 9 by dummy activities, we begin with nodes 5 and 7. Node 5 receives
one head arrow from its predecessor activity B, we evaluate the EF time of B as 6 (ES
(3) + d (3)). Successor activities to node 5, therefore, can have an ES time of 6.
Similarly, the ES time at node 7 is calculated as time 9.

- Moving to node 9, the EF times of its 3 predecessors (d1, C, and d2) are time 6, 7,
and 9, respectively. Accordingly, the ES time of successor activities is the largest

Construction Management 78 Dr. Emad Elbeltagi


value 9. Notice that only the largest EF value of predecessor activities is used to
calculate the ES of successor activities and all other values not used. As such, only
ES values can be directly read from the calculations in Figure 4.4. EF values, on the
other hand can be calculated as EF = ES + d.

- The last node (11) receives one head arrow, activity E which has an ES value of 9.
The EF time of activity E, therefore =9 + 5 = time 14. Since node 11 is the last node,
the EF of this node becomes the end of the project, reaching total project duration of
14 days.

Generally, for any activity x connecting between nodes i and j as shown in Figure 4.5, the
calculations as follows:

ETi LTi ETj LTj


` x
i j
dx

Figure 4.5: Activity times

ETj = ETi + dx
(4.1)
In case of more than one arrow terminating at node j, then consider the largest value.

Accordingly, ESx = ETi (4.2)

EFx = ESx +dx (4.3)

Backward Path

The backward path determines the late-finish (LF) times of activities by proceeding
backward from the end node to the starting node of the AOA network. We put the LF
values in the right side boxes adjacent to the nodes, as shown in Figure 4.6. For the
example at hand, we do the following:

Construction Management 79 Dr. Emad Elbeltagi


6 9
9-0=9
9-3=6,
9-4=5, or 5
3-3=0 9-6=3 B d1 14 14
0 0 3 3 3
A C E
1 3 9 11
3 4 5
D d2 9 9
6 14-5=9
LF-d=LS
7
9 9
9-0=9

Figure 4.6: Backward path calculations in AOA networks

- Start from the last node of the network (node 11) and we transfer the early-finish
value from the left box to be the late-finish (LF) value at the right side box.

- Then, move backward to node 9 which has only one tail arrow of activity E. With the
LF time of E being time 14, its LS time becomes LS = LF - d = 14 – 5 = time 9. At
node 9, therefore, time 9 becomes the LF time of the predecessor activities of this
node.

- Moving backward to predecessor nodes 5, and 7. Node 5 has one tail arrow of the
dummy activity d1, and as such, the LF time value to be used at node 5 becomes 9.
Similarly, the LF time value of node 7 becomes 9.

- Moving to node 3, we evaluate the LS time of its 3 successor activities B, C, and D as


6, 5, and 3, respectively. The LF time at node 3, therefore, becomes the smallest value
3. With other LS values not used, the values in the calculation boxes, as such, directly
show the LF times of activities. LS times can be calculated as LS = LF – d.

- Now, proceed to the first node in the network (node 1). It connects to one tail arrow
of activity A. The LS time of A, therefore, is LS = LF – d = 3 – 3 = 0, a necessary
check to ensure the correctness of the calculation.

Construction Management 80 Dr. Emad Elbeltagi


Having Figure 4.5 again in mind and to generalize the calculations, for any activity x
connecting between nodes i and j, the calculations as follows:

LTi = LTj - dx (4.4)

In case of more than one arrow leaving node i, then consider the smallest value.

Accordingly, LFx = LTj (4.5)

LSx = LFx -dx (4.6)

Float Calculations

Once forward path and backward path calculations are complete, it is possible to analyze
the activity times. First, let's tabulate the information we have as shown in Table 4.1. One
important aspect is Total-Float (TF) calculations, which determine the flexibility of an
activity to be delayed. Notice in Table 4.1 that some activities such as activity A has ES
time = LS time, and its EF time = LF time, indicating no slack time for the activity. Other
activities such as B can start early at time 3 and late at time 6, indicating a 3-day of total
float. Float calculations can be illustrated as shown in Figure 4.7 for any activity.

Table 4.1: CPM results

Early Start Late Finish Late Start Early Finish Total Float Critical
Activity Duration
(ES) (LF) (LS) (EF) (TF) Activity
A 3 0 3 0 3 0 Yes
B 3 3 9 6 6 3 No
C 4 3 9 5 7 2 No
D 6 3 9 3 9 0 Yes
E 5 9 14 9 14 0 Yes

Figure 4.7 shows two ways of scheduling each activity using its activity times. One way
is to schedule it as early as possible (using its ES time). The other way is as late as

Construction Management 81 Dr. Emad Elbeltagi


possible (using its LS time). The activity float can, therefore, be represented by the
following relationships:

Total Float (TF) = LF – EF (4.7)

= LS – ES (4.8)

Name
i duration = d j

ET LT ET LT
ES = ETi ETj LF = LTj
ES EF=ES+d Total Float
a) Activity is early d LF

ES Total Float LS=LF-d


b) Activity is late d LF
d Free Float (FF)
Total time available for the activity = LF - ES

Figure 4.7: Float calculations

Also, with the ES and LF times directly read from the boxes used in forward and
backward path calculations, the total float can also be calculated as; TF = LF – ES – d.
Using these relationships, activities total floats are calculated as shown in Table 4.1.

Another type of float often used in network analysis is the Free Float, which can be
calculated as:

Free Float (FF) = ETj – ETi – d (4.9)

or FF = smallest ES (of succeeding activities) – EF (of current activity) (4.10)

The free float defines the amount of time that an activity can be delayed without affecting
any succeeding activity. With free float available for an activity, a project manager

Construction Management 82 Dr. Emad Elbeltagi


knows that the float can be used without changes the status of any non-critical activity to
become critical.

Identifying the Critical Activities

Activities with zero total floats mean that they have to be constructed right at their
schedule times, without delays. These activities are considered to be critical. They
deserve the special attention of the project manager because any delay in critical
activities causes a delay in the project duration.

One interesting observation in the results of CPM analysis is that critical activities form a
continuous path of the critical activities that spans from the beginning to the end of the
network. In our example, activities A, D, and E (excluding dummy activities) are critical
and the critical path is indicated by bold lines on Figure 4.6. Notice that among the 3
paths in this example (A-B-E; A-C-E; and A-D-e), the critical path is the longest one, an
important characteristic of the critical path. In real-life projects with many activities, it is
possible that more than one critical path are formed. By definition, the length of these
critical paths is the same.

4.2.2 Precedence Diagram Method (PDM)

Precedence Diagram Method (PDM) is the CPM scheduling method used for AON
networks and it follows the same four steps of the CPM for AOA method.

Forward Path

Forward path can proceed from one activity to the other; the process is as follow (Figure
4.8):

- At activity A. It is the first activity in the network. We give it an early-start (ES) of 0


in the left top box. Adding the activity duration, we determine the EF time of the
activity and we put it in the top right box.

Construction Management 83 Dr. Emad Elbeltagi


3 6
B (3)

6, 7, or 9
0 3 3 7 9 14
A (3) C (4) E (5)

Early start Ealry Finish 3 9


Name (duration) D (6)
Late start Late finish

Figure 4.8: Forward Path in PDM Analysis

- Then, move forward to the succeeding activities B, C, and D. These three activities
have only A as a predecessor with time 3 as its EF. As such, all the three activities
can start as early as time 3 (ES = 3). Each activity, accordingly, has its own EF time
based on its duration.

- Moving forward to activity E. This activity has 3 predecessors (3 head arrows) of


activities B, C, and D with their largest EF time being 9. The ES of activity E, thus,
with becomes time 9. Adding its duration, the EF becomes time 14.

To generalize the calculations consider Figure 4.9, of two activities i and j with
relationship finish to start and overlap between them. Overlaps will have a positive sign,
while lags will have a negative sign. The forward path calculations are as follows:

ESi EFi ESj EFj


overlapij
i (di) j (dj)
LSi LFi LSj LFj

Figure 4.9: Activities times in PDM Analysis

ESj = EFi - overlapij (4.11)


In case of more than one activity precedes activity j then consider the maximum. Then,
apply Equation 4.3 to calculate the early finish times.

Construction Management 84 Dr. Emad Elbeltagi


Backward Path

Once the forward path is finished, the backward path can start, moving from the last
activity to the first, putting the calculations in the bottom two boxes of each activity, as
shown in Figure 4.10. The process is as follows:

3 6
B (3)
6 9

0 3 3 7 9 14
A (3) C (4) E (5)
0 3 5 9 9 14
6, 5, or 3

Early start Early Finish 3 9


Name (duration) D (6)
Late start Late finish 3 9

Figure 4.10: Backward path in PDM analysis

- Start at the last activity E and we transfer the early-finish value to become the
activity's late-finish (LF) time. Then, subtracting the activity's own duration, the late-
start (LS) time is calculated as time 9 and put in the bottom left box of the activity.

- Moving backward to activities B, C, and D all have one successor (activity E) with
LS time of 9. The LF of all these activities becomes time 9. Each activity then has its
own LS time, as shown in Figure 4.10.

- Moving to activity A. The activity is linked to 3 tail arrows (i.e., has 3 successors) of
activities B, C, and D. The LF of activity A, thus, is the smallest of its successors' LS
times, or time 3. Activity A then has LS equals zero.
Considering Figure 3.9 again, the backward path calculations are as follows:

LFi = LSj + overlapij (4.12)

Construction Management 85 Dr. Emad Elbeltagi


In case of more than one activity succeeds activity j then consider the minimum. Then,
apply Equation 4.6 to calculate the late start times.

Notice that by the end of the backward path, all activity times can be read directly from
the boxes of information on the activity, without additional calculations. This also, makes
it simple to calculate the total float of each activity using the same relationships used in
the AOA analysis.

Identifying Critical Activities

Critical activities can also be easily determined as the ones having zero float times,
activities A, D, and E. The critical path is then shown in bold as Figure 4.10. The PDM
analysis, as explained, is a straight forward process in which each activity is considered
as an entity that stores its own information.

4.3 Time-Scaled Diagrams

Time-scaled diagrams are used extensively in the construction industry. Such diagrams
enable one to determine immediately which activities are scheduled to proceed at any
point in time and to monitor field progress. Also, it can be used to determine resources
need. The time scale used in time-scaled diagrams can be either the calendar dates or the
working periods (ordinary dates), or using both at the same time.

The activities are represented as arrows that drawn to scale to reflect the activity duration
it represents. The horizontal dashed lines represent total float for groups of activities and
free float for the immediate activity to the left of the dashed line. The precedence of an
activity is the immediate activities before it or that linked to it through vertical dashed
lines. The name and the duration of an activity are written above and below the arrow
representing it respectively (Figure 4.11). The ES, EF, and FF times of the activities can
be easily read directly from the diagram. The TF for an activity is the smallest sum of
succeeding FF on all paths. Accordingly, the LS and LF times can be easily calculated as
follows:

Construction Management 86 Dr. Emad Elbeltagi


LSi = ESi + TFi (4.13)

LFi = LSi + Di (3.14)

The critical path can be easily determined as the continuous lines from the beginning to
the end of the network with any dashed lines. The main advantage of this diagram is its
simple representation and it can be sued directly for determining resources need.
However, its disadvantage is that it needs a great effort to be modified or updated. Also,
it cannot be used to represent overlapping activities. Figure 4.11 shows the time-scaled
diagram for the same 5-activities project solved previously using AOA and AON
networks.

1 2 3 4 5 6 7 8 9 10 11 12 13 14

B
A C
D

Figure 4.11: Time-scaled diagram

The TF for activity A equals the smallest of the sum of the floats along all paths from the
end of activity A to the end of the project. The float on path ABE = 3, path ACE = 2 and
path ADE = 0, then the TF of activity A = 0. The calculations are shown in Table 3.2.

Table 4.2 Time-scaled diagram calculations

Activity ES EF FF TF LF=EF+TF LS=LF-d


A 0 3 0 0 3 0
B 3 6 3 3 9 6
C 3 7 2 2 9 5
D 3 9 0 0 9 3
E 9 14 0 0 14 9

Construction Management 87 Dr. Emad Elbeltagi


4.4 Schedule Presentation

After the AOA and AON calculations are made, it is important to present their results in a
format that is clear and understandable to all the parties involved in the project. The
simplest form is the Bar chart or Gantt chart, named after the person who first used it. A
bar chart is a time versus activity chart in which activities are plotted using their early or
late times, as shown in Figures 4.12 a and b. Early bar chart is drawn using the ES times
of activities, while the late bar chart is drawn using the LS times.

Activity
d=3
A
ES = 0 d=3 TF=3
B ES=3
d=4 TF=2
a) C ES=3
d=6
D ES=3
d=5
E ES=9

0 1 2 3 4 5 6 7 8 9 10 11 12 13 14 Time

Activity
d=3
A LF=3
TF=3 d=3
B LF=9
TF=2 d=4
b)
C LF=9
d=6
D LF=9
d=5
E LF=14

0 1 2 3 4 5 6 7 8 9 10 11 12 13 14 Time

Figure 4.12: a) Early bar chat b) Late bar chart

The bar chart representation, in fact, shows various details. Float times of activities,
critical activities can be shown in a different color, or bold borders, as shown in Figure
4.12. The bar chart can also be used for accumulating total daily resources and / or costs,
as shown at the bottom part of Figure 4.13. In this figure, the numbers on each activity
represent the number of labors needed.

Construction Management 88 Dr. Emad Elbeltagi


Activity 0 1 2 3 4 5 6 7 8 9 10 11 12 13 14 Time

2 2 2
A
2 2 2
B
1 1 1 1
C
3 3 3 3 3 3
D
1 1 1 1 1
E

6
6
Profile of the labor
resource demand 5
4
4
3 3
2
2
1
1

2 2 2 6 6 6 4 3 3 1 1 1 1 1 Total labors

Figure 4.13: Using bar chart to accumulate resources

One additional benefit of the bar chart is its use on site to plot and compare the actual
progress in the various activities to their scheduled times. An example is shown on Figure
4.13, showing actual bars plotted at the bottom of the original bars of the schedule.

4.5 Criticisms to Network Techniques

The CPM and PDM analyses for network scheduling provide very important information
that can be used to bring the project to success. Both methods, however, share some
drawbacks that require special attention from the project manager. These drawbacks are:

- Assume all required resources are available: The CPM calculations do not incorporate
resources into their formulation. Also, as they deal with activity durations only, it can
result in large resource fluctuations. Dealing with limited resources and resource
leveling, therefore, has to be done separately after the analysis;

Construction Management 89 Dr. Emad Elbeltagi


- Ignore project deadline: The formulations of CPM and PDM methods do not
incorporate a deadline duration to constrain project duration;

- Ignore project costs: Since CPM and PDM methods deal mainly with activities
durations, they do not deal with any aspects related to minimize project cost;

- Use deterministic durations: The basic assumption in CPM and PDM formulations is
that activity durations are deterministic. In reality, however, activity durations take
certain probability distribution that reflect the effect of project conditions on resource
productivity and the level of uncertainty involved in the project.

4.6 Solved Examples

4.6.1 Example 1

For the project data in Table 4.3, answer the following questions:
a) Draw an AOA network of the project?
b) Perform forward path and backward path calculations?
c) What is the effect of delaying activity D by 3 days?

Table 4.3: Data for Example 1

Immediate
Activity Duration
predecessor
A 2 -
B 6 A
C 3 A
D 1 B
E 6 B
F 3 C, D
G 2 E, F

Construction Management 90 Dr. Emad Elbeltagi


Solution

a, b) 8 or
8 8 10

2 or 3 14 or
8 B E 12
0 0 2 2 6 6 14 14 16 16
A D 1 G
1 2 5 6
2 2
C F 3
3
4
9 or
9 11
Critical 5

c) Total float of activity D = LF – ES – d = 11 – 8 – 1 = 2.

Then delaying activity D by 1 day more than its total float will cause a net delay in
the whole project by 1 day to become 17 days.

4.6.2 Example 2

Perform PDM calculations for the small project below and determine activity times.
Durations are shown on the activities.

I
(2)

B D G
(4) (1) (1)

A J L
(1) (7) (2)

C E H
(1) (2) (1)

F K
(2) (4)

Construction Management 91 Dr. Emad Elbeltagi


Solution
7 9
I (2)
12 14

1 5 5 6 6 7
B (4) D (1) G (1)
1 5 5 6 6 7 9 or
9 or
12 or
14
7
0 1 7 14 14 16
A (1) J (7) L (2)
0 1 7 14 14 16
1 or 6
1 2 2 4 4 5
C (1) E (2) H (1)
6 7 7 9 9 10
7 or 8 5 or 4

ES EF 2 4 5 9
Name (duration) F (2) K (4)
LS LF 8 10 10 14

Critical path

4.6.3 Example 3

For the activities listed in the table below, draw the time-scaled diagram and mark the
critical path. Determine the completion time for the project. Tabulate activities times and
floats.

Activity Duration Predecessor


A 4 -
B 4 A
C 8 B
D 3 C
E 5 A
F 2 B, E
G 8 C, F
H 5 D, G
I 17 -
J 10 G, I

Construction Management 92 Dr. Emad Elbeltagi


Solution

1 2 3 4 5 6 7 8 9 0 1 2 3 4 5 6 7 8 9 0 1 2 3 4 5 6 7 8 9 0 1 2 3 4
0 1 2 3
A B C D H
4 4 8 3 5 5 5

F G
1 2 5 8
I J
17 7 10

Activity ES EF TF FF LS LF
A 0 4 0 0 0 4
B 4 8 0 0 4 8
C 8 16 0 0 8 16
D 16 19 10 5 26 29
E 4 9 5 0 9 14
F 9 11 5 5 14 16
G 16 24 0 0 16 24
H 24 29 5 5 29 34
I 0 17 7 7 7 24
J 24 34 0 0 24 34

4.6.4 Example 4

Perform PDM calculations for the small AoN network shown here. Pay special attention
to the different relationships and the lag times shown on them.

B
SS=2 (3)

A C E
(3) (4) (5)

D FF=2
(6)

Construction Management 93 Dr. Emad Elbeltagi


Solution

0+2=2
2 5
B (3)
SS=2 4 7
5, 7 or
(9+2-5)
0 3 3 7 7 12
A (3) C (4) E (5)
0 3 3 7 7 12
4 or
3 or
(4-2+3) 3 9
D (6) FF=2

ES EF 4 10
Name (duration) 12-2=10
LS LF

4.7 Exercises

1. The free float is defined as:


a. The amount of time an activity can be delayed without affecting the following
activity.
b. The amount of time an activity can be delayed without affecting total project
duration.

2. Total float equals:


a. Late finish minus early finish c. Late start minus early start
b. Late finish minus (early start + duration) d. All of the above

3. State True (T) or False (F):


a. The critical activities can be determined easily when using the bar chart.
b. The network must have definite points of beginning and end.
c. The network must be continuous from start to end.
d. There’s no dummy activities in the arrow networks,
e. A forward pass is used to determine late start and late finish times.

Construction Management 94 Dr. Emad Elbeltagi


f. The time for completing a project is equal to the sum of the individual activity
times.

4. For the Following project data, answer the following questions:

Activity Duration (days) Predecessor


A 2 --
B 6 A
C 3 A
D 1 B
E 6 B
F 3 C, D
G 2 E, F

a. Draw an AOA network and perform forward and backward pass calculations?
b. Draw an AON network and perform forward and backward pass calculations?
c. Draw a time-scaled diagram of the project?
d. Tabulate activities ES, EF, LS, LF, TF, and FF.
e. What is the effect of delaying activity D by 3 days?

5. For the following AOA network, determine the following:


a. Calculate ES, LF, & TF for all activities. Identify critical ones.
b. Draw an early Bar Chart for the project.
c. What is the effect of delaying activity “H” by two days on the total project
duration?
3 9

1 5 11 15 17
E (10)

7 13

Construction Management 95 Dr. Emad Elbeltagi


6. Perform PDM calculations for the project below and determine activity times.
Durations are shown on the activities

I
(2)

B D G
(4) (1) (1)

A J L
(1) (7) (2)

C E H
(1) (2) (1)

F K
(2) (4)

7. A gas station is proposed to be built on an already developed site. It will consist


essentially of a sales outlet and an office block. The sales outlet comprises of cash
office and gas pumps. The manager’s office building, which also houses public
washrooms and an air compressor, is called the office block. Adjacent to pumps
will be a concrete pit that will house the gas tanks.

The entire area, excluding the office and pumps site, is covered with a concrete
slab, and there is a low perimeter wall in the rear. The utility company has
undertaken to install an electric meter on the site and connect it to the mains.
Gasoline pumps must be obtained from the manufacturers, and after being
installed, they are to be connected to the gasoline tanks and the power supply.
Before use the local authority to ensure safety and compliance with regulations
must inspect them. Gasoline tanks are housed in concrete pits and covered by
concrete slabs. Before they are covered, however, the tanks and the associated
pipe work have to be inspected by the local authority. The sales outlet base is
excavated first, the pipe work and tanks second, the office block third, and the

Construction Management 96 Dr. Emad Elbeltagi


trench for underground services last. After the excavation for the tanks and pipe
work is completed, work can proceed on the construction of the perimeter wall
and air points.

No Activity Description Duration


A Excavate for sales outlet 1
B Construct sales outlet base 1
C Construct cash office 2
D Obtain pumps 16
E Install pumps 2
F Connect pumps 1
G Inspector approves pumps installation 2
H Paint & furnish office & washrooms 2
I Connect office & toilet lighting 1
J Excavate for office block 1
K Construct office block 1
L Build office & washrooms + services 15
M Install electric meter 14
N Connect main cable to meter 1
O Install area lighting 4
P Mobilize site 1
Q Set out and level site 1
R Excavate & lay underground services 1
S Excavate for pipes and tanks 1
T Construct concrete pit 2
U Obtain pipes and tanks 2
V Install pipes and tanks 3
W Obtain compressor 10
X Install compressor 1
Y Connect power to compressor 1
Z Inspection of compressor 2
AA Backfill and cover tanks 1
BB Pour concrete slab 2
CC Construct perimeter wall + air points 2
DD Connect air points 1
EE Demobilize and clean site 1
FF Inspection of pipes and tanks 2

Construction Management 97 Dr. Emad Elbeltagi


Compressed air for inflated tires will be supplied by an electrically driven
compressor, which must be inspected by a competent person before the
compressor is put into use. The air lines to the “free air” points are installed with
the general underground services, and the points themselves are mounted on the
perimeter wall. The air pints can be hooked up after the concrete slab has been
poured. Mobilization to start work comprises, among other preparations, the
moving of a trailer to the site to store tools, furnishings, and any weather prone
parts and to serve as the site office. Similarly, when work at the site is completed,
the trailer will be removed, and all scaffolding and construction equipment taken
away. This is known as “demobilization and clean up of site.

You are required to determine the project duration, critical path(s), and tabulate
activity times (ES, EF, LS, LF, TF, and FF).

8. For the following list of activities, draw a time-scaled diagram and mark the
critical path. Determine activities ES, EF, LS, LF, FF, and TF.

Activity Duration (days) Predecessor


A 4 --
B 10 A
C 2 A
D 6 C
E 15 B, D
F 4 B, D
G 3 F
H 2 B, D
I 1 E, G, H
J 3 I
K 2 E
L 1 J
M 2 K, L

Construction Management 98 Dr. Emad Elbeltagi


9. For the following PDM, perform the forward pass and backward pass
calculations. Determine the project duration and critical path. Tabulate the ES,
EF, LS, LF, TF, and FF information for each activity.

E
FF7
7

A SS3, FF4
B SS2
D F SS8
G
10 8 12 FF5 9 12

SS8

J
C 0
SF5
8

H I
FS6 FF3
10 8

Construction Management 99 Dr. Emad Elbeltagi


CHAPTER 5

STOCHASTIC SCHEDULING

In some situations, estimating activity duration becomes a difficult task due to ambiguity
inherited in and the risks associated with some work. In such cases, the duration of an
activity is estimated as a range of time values rather than being a single value. This
chapter deals with the scheduling of the project under uncertain activity duration. The
program evaluation and review technique is presented as one of the methods that can be
used to schedule projects with uncertain activities durations. Also, an introduction to
Monte Carlo simulation technique is presented.

5.1 Scheduling with Uncertain Durations

Some scheduling procedures explicitly consider the uncertainty in activity duration


estimates by using the probabilistic distribution of activity durations. That is, the
duration of a particular activity is assumed to be a random variable that is distributed in a
particular fashion. For example, an activity duration might be assumed to be distributed
as a normal or a beta distributed random variable as illustrated in Figure 5.1. This figure
shows the probability or chance of experiencing a particular activity duration based on a
probabilistic distribution. The beta distribution is often used to characterize activity
durations, since it can have an absolute minimum and an absolute maximum of possible
duration times. The normal distribution is a good approximation to the beta distribution
in the center of the distribution and is easy to work with, so it is often used as an
approximation.

Construction Management 100 Dr. Emad Elbeltagi


Figure 5.1: Beta and normally distributed activity durations

If a standard random variable is used to characterize the distribution of activity


durations, then only a few parameters are required to calculate the probability of any
particular duration. Still, the estimation problem is increased considerably since more
than one parameter is required to characterize most of the probabilistic distribution used
to represent activity durations. For the beta distribution, three or four parameters are
required depending on its generality, whereas the normal distribution requires two
parameters.

The most common formal approach to incorporate uncertainty in the scheduling process
is to apply the critical path scheduling process and then analyze the results from a
probabilistic perspective. This process is usually referred to as the Program Evaluation
and Review Technique (PERT) method. As noted earlier, the duration of the critical path
represents the minimum time required to complete the project. Using expected activity
durations and critical path scheduling, a critical path of activities can be identified. This
critical path is then used to analyze the duration of the project incorporating the
uncertainty of the activity durations along the critical path. The expected project duration
is equal to the sum of the expected durations of the activities along the critical path.

Construction Management 101 Dr. Emad Elbeltagi


Assuming that activity durations are independent random variables, the variance or
variation in the duration of this critical path is calculated as the sum of the variances
along the critical path. With the mean and variance of the identified critical path known,
the distribution of activity durations can also be computed.

5.1.1 Program Evaluation and Review Technique

Both CPM and PERT were introduced at approximately the same time and, despite their
separate origins, they were very similar. The PERT method shares many similarities with
CPM. Both require that a project be broken down into activities that could be presented
in the form of a network diagram showing their sequential relationships to one another.
Both require time estimates for each activity, which are used in routine calculations to
determine project duration and scheduling data for each activity.

CPM requires a reasonably accurate knowledge of time and cost for each activity. In
many situations, however, the duration of an activity can not be accurately forecasted,
and a degree of uncertainty exists. Contrary to CPM, PERT introduces uncertainty into
the estimates for activity and project durations. It is well suited for those situations where
there is either insufficient background information to specify accurately time and cost or
where project activities require research and development.

In the original development of PERT approach, AOA notations are used. However, AON
diagramming can be easily used alternatively. The method is based on the well-known
“central limit theorem”. The theorem states that: “Where a series of sequential
independent activities lie on the critical path of a network, the sum of the individual
activity durations will be distributed in approximately normal fashion, regardless of the
distribution of the individual activities themselves. The mean of the distribution of the
sum of the activity durations will be the sum of the means of the individual activities and
its variance will be the sum of the activities’ variances”. The primary assumptions of
PERT can be summarized as follows:

Construction Management 102 Dr. Emad Elbeltagi


1. Any PERT path must have enough activities to make central limit theorem
valid.
2. Any PERT path must have enough activities to make central limit theorem
valid.
3. The mean of the distribution of the path with the greatest duration, from the
initial node to a given node, is given by the maximum mean of the duration
distribution of the paths entering the node.
4. PERT critical path is longer enough than any other path in the network.

PERT, unlike CPM, uses three time estimates for each activity. These estimates of the
activity duration enable the expected mean time, as well as the standard deviation and
variance, to be derived mathematically. These duration estimates are:

- Optimistic duration (a); an estimate of the minimum time required for an activity
if exceptionally good luck is experienced.
- Most likely or modal time (m); the time required if the activity is repeated a
number of times under essentially the same conditions.
- Pessimistic duration (b); an estimate of the maximum time required if unusually
bad luck is experienced.

These three time estimates become the framework on which the probability distribution
curve for the activity is erected. Many authors argue that beta distribution is mostly fit
construction activities.

The use of these optimistic, most likely, and pessimistic estimates stems from the fact
that these are thought to be easier for managers to estimate subjectively. The formulas
for calculating the mean and variance are derived by assuming that the activity durations
follow a probabilistic beta distribution under a restrictive condition. The probability
density function of beta distributions for a random variable x is given by:

f(x) = k(x - a)α (x - b) β , a ≤ x ≤ b, α, β > −1 (5.1)

Construction Management 103 Dr. Emad Elbeltagi


where k is a constant which can be expressed in terms of α and β. Several beta
distributions for different sets of values of α and β are shown in Figure 5.2.

Figure 5.2: Illustration of several Beta distributions

Using beta distribution, simple approximations are made for the activities’ mean time

and its standard deviation. Using the three times estimates, the expected mean time (te) is

derived using Eq. 5.2. Then, te is used as the best available time approximation for the
activity in question. The standard deviation is given by Eq. 5.3, and hence the variance
(ν) can be determined as ν = σ2.

a + 4m + b
te = (5.2)
6

b−a
σ= (5.3)
6

By adopting activity expected mean time, the critical path calculations proceed as CPM.
Associated with each duration in PERT, however, is its standard deviation or its

Construction Management 104 Dr. Emad Elbeltagi


variance. The project duration is determined by summing up the activity expected mean
time along the critical path and thus will be an expected mean duration. Since the
activities on the critical path are independent of each other, central limit theory gives the
variance of the project duration as the sum of the individual variances of these critical
path activities.

Once the expected mean time for project duration (TX) and its standard deviation (σX)
are determined, it is possible to calculate the chance of meeting specific project duration
(TS). Then normal probability tables are used to determine such chance using Equation
5.4.

TS − TX
Z= (5.4)
σX

TS = TX + Z * σ X is an equivalent form of Equation 5.4, which enables the scheduled


time for an event to be determined based on a given risk level. The procedure for hand
probability computations using PERT can be summarized in the following steps:

1. Make the usual forward and backward pass computations based on a single
estimate (mean) for each activity.
2. Obtain estimates for a, m, and b for only critical activities. If necessary, adjust

the length of the critical path as dictated by the new te values based on a, m,
and b.
3. Compute the variance for event x (νX) by summing the variances for the
critical activities leading to event x.
4. Compute Z using Equation 5.4 and find the corresponding normal probability.

Consider the nine activity example project shown in Table 5.1. Suppose that the project
have very uncertain activity time durations. As a result, project scheduling considering
this uncertainty is desired.

Construction Management 105 Dr. Emad Elbeltagi


Table 5.1: Precedence relations and durations for a 9-activity project example

Activity Description Predecessors Duration


A Site clearing --- 4
B Removal of trees --- 3
C General excavation A 8
D Grading general area A 7
E Excavation for trenches B, C 9
F Placing formwork and RFT for concrete B, C 12
G Installing sewer lines D, E 2
H Installing other utilities D, E 5
I Pouring concrete F, G 6

Table 5.2 shows the estimated optimistic, most likely and pessimistic durations for the
nine activities. From these estimates, the mean, variance and standard deviation are
calculated. In Figure 5.3, PERT calculations are performed very similar to that of CPM,
considering the mean duration of each activity.

Table 5.2: Activity duration estimates for the 9-activity project


Duration Standard
Activity Mean
a m b Deviation
A 3 4 5 4.0 0.33
B 2 3 5 3.2 0.50
C 6 8 10 8.0 0.67
D 5 7 8 6.8 0.50
E 6 9 14 9.3 1.33
F 10 12 14 12.0 0.67
G 2 2 4 2.3 0.33
H 4 5 8 5.3 0.67
I 4 6 8 6.0 0.67

Construction Management 106 Dr. Emad Elbeltagi


4.0 D 10.8 21.3 H 26.6
14.9 6.8 21.7 24.7 5.3 30.0

0.0 A 4.0 4.0 C 12 12.0 E 21.3 21.3 G 23.3 24.0 I 30.0


0.0 4.0 4.0 4.0 8.0 12.0 12.4 9.3 21.7 21.7 2.3 24.0 24.0 6.0 30.0

0.0 B 3.2 12.0 F 24.0


8.8 3.2 12.0 12.0 12 24.0

Figure 5.3: PERT calculations for 9-activity example

The critical path for this project ignoring uncertainty in activity durations consists of
activities A, C, F and I. Applying the PERT analysis procedure suggests that the duration
of the project would be approximately normally distributed. The sum of the means for
the critical activities is 4.0 + 8.0 + 12.0 + 6.0 = 30.0 days, and the sum of the variances is
(0.33)2 + (0.67)2 + (0.67)2 + (0.67)2 = 1.44 leading to a standard deviation of 1.2 days.

With normally distributed project duration, the probability of meeting a project deadline
can be computed using Equation (4.4). For example, the probability of project
completion within 35 days is:

35 − 30
Z= = 4.167
1.2

where z is the standard normal distribution tabulated value of the cumulative standard
distribution, which can be determined form standard tables of normal distribution. From
Table 4.3, the probability of completing the project in 35 days is 100%.

Construction Management 107 Dr. Emad Elbeltagi


-3σ -2σ -σ 0 σ 2σ 3σ

Figure 5.4: Normal distribution curve

Table 5.3: Area under the normal curve measured from the center

SD Area % from the center SD Area % from the center


0.1σ 4.0 1.6 44.5
0.2 7.9 1.7 45.5
0.3 11.8 1.8 46.4
0.4 15.5 1.9 47.1
0.5 19.2 2.0 47.7
0.6 22.6 2.1 48.2
0.7 25.8 2.2 48.6
0.8 28.8 2.3 48.9
0.9 31.6 2.4 49.2
1.0 34.1 2.5 49.4
1.1 36.4 2.6 49.5
1.2 38.5 2.7 49.6
1.3 40.3 2.8 49.7
1.4 41.9 2.9 49.98
1.5 43.3 3.0 49.99

Example 5.1

Suppose that a network has been developed for a particular project with non-
deterministic durations for the activities and the completion time for that network is 320
days and the sum of the standard deviation for the activities on the critical path is 2130.
Find the probability that the project will be completed in 300 days.

Construction Management 108 Dr. Emad Elbeltagi


Solution

First, convert the normal random variable to the standard normal random
variable.

TS − TX
Z= = (300 – 320) / 46.2 = - 0.43
σX

From Table 3, the corresponding probability = 16.5%


Then, the probability to complete the project in 300 days equals = 50 – 16.5 =
33.5%.

Example 5.2

Given the information from the previous example, what is the duration that you can give
with 90 percent assurance?

Solution

From tables find the value of z corresponding to probability of 40%, thud yields
z = 1.28
then, apply z into equation 4: 1.28 = (t – 320) / 46.2
or t = 46.2 x 1.28 + 320 = 380 days.

5.1.2 Criticism to Program Evaluation and Review Technique

While the PERT method has been made widely available, it suffers from three major
problems. First, the procedure focuses upon a single critical path, when many paths
might become critical due to random fluctuations. For example, suppose that the critical
path with longest expected time happened to be completed early. Unfortunately, this
does not necessarily mean that the project is completed early since another path or
sequence of activities might take longer. Similarly, a longer than expected duration for
an activity not on the critical path might result in that activity suddenly becoming

Construction Management 109 Dr. Emad Elbeltagi


critical. As a result of the focus on only a single path, the PERT method typically
underestimates the actual project duration.

As a second problem with the PERT procedure, it is incorrect to assume that most
construction activity durations are independent random variables. In practice, durations
are correlated with one another. For example, if problems are encountered in the
delivery of concrete for a project, this problem is likely to influence the expected
duration of numerous activities involving concrete pours on a project. Positive
correlations of this type between activity durations imply that the PERT method
underestimates the variance of the critical path and thereby produces over-optimistic
expectations of the probability of meeting a particular project completion deadline.

Finally, the PERT method requires three duration estimates for each activity rather than
the single estimate developed for critical path scheduling. Thus, the difficulty and labor
of estimating activity characteristics is multiplied threefold.

5.2 Monte Carlo Simulation

5.2.1 Monte Carlo Simulation Characteristics

- Replaces analytic solution with raw computing power.


- Avoids need to simplify to get analytic solution
- No need to assume functional form of activity/project distributions.
- Allows determining the criticality index of an activity (Proportion of runs in
which the activity was in the critical path).
- Hundreds to thousands of simulations needed.

5.2.2 Monte Carlo Simulation Process

- Set the duration distribution for each activity.


• No functional form of distribution assumed.
• Could be joint distribution for multiple activities.
- Iterate: for each “trial” (“realization”)
• Sample random duration from each distributions

Construction Management 110 Dr. Emad Elbeltagi


• Find critical path & durations with standard CPM; Record these results
- Report recorded results Report recorded results.
• Duration distribution
• Per--node criticality index (% runs where critical)

Example 5.3

Table 5.4: Data for Example 5.3

Optimistic Most likely Pessimistic Expected Standard


Activity
time, a time, m time, b value, d deviation, s
A 2 5 8 5 1
B 1 3 5 3 0.66
C 7 8 9 8 0.33
D 4 7 10 7 1
E 6 7 8 4 0.33
F 2 4 6 5 0.66
G 4 5 6 5 0.33

Construction Management 111 Dr. Emad Elbeltagi


Table 5.5: Summary of simulation runs for Example 5.3

Run Activity Duration Critical Completion


Number A B C D E F G Path Time
1 6.3 2.2 8.8 6.6 7.6 5.7 4.6 A-C-F-G 25.4
2 2.1 1.8 7.4 8.0 6.6 2.7 4.6 A-D-F-G 17.4
3 7.8 4.9 8.8 7.0 6.7 5.0 4.9 A-C-F-G 26.5
4 5.3 2.3 8.9 9.5 6.2 4.8 5.4 A-D-F-G 25.0
5 4.5 2.6 7.6 7.2 7.2 5.3 5.6 A-C-F-G 23.0
6 7.1 0.4 7.2 5.8 6.1 2.8 5.2 A-C-F-G 22.3
7 5.2 4.7 8.9 6.6 7.3 4.6 5.5 A-C-F-G 24.2
8 6.2 4.4 8.9 4.0 6.7 3.0 4.0 A-C-F-G 22.1
9 2.7 1.1 7.4 5.9 7.9 2.9 5.9 A-C-F-G 18.9
10 4.0 3.6 8.3 4.3 7.1 3.1 3.1 A-C-F-G 19.7

Figure 5.5: Project duration distribution

Construction Management 112 Dr. Emad Elbeltagi


Then the probability that a project ends in a specific time (t) equals number of times the
project finished in less than or Equal to t divided by the total number of replications. For
example, the probability that the project ends in 20 weeks or less is:

P( ≤ 20 ) = 13 / 50 = 26%.

5.2.3 Criticality Index

Criticality index is defined as the proportion of runs in which the activity was in the
critical path. PERT assumes binary (either 100% or 0%).

5.3 Exercises

1. Recently, you were assigned to manage a project for your company. You have
constructed a network diagram depicting various activities in the project. In
addition, you asked various managers and subordinates to estimate the amount of
time they would expect each activity to take. Their responses (in days) were as
follows:

Duration (days)
Activity Optimistic Most likely Pessimistic
(a) (m) (b)
A 4 7 10
B 8 10 14
C 3 6 7
D 2 4 6
E 7 8 9

10
A C

D 40
5
B E
35

Construction Management 113 Dr. Emad Elbeltagi


a. Compute the mean and variance in time for each activity.
b. Determine the critical path and the expected length of the critical path.
c. Assume that the time required to complete a path is normally distributed. What
is the probability of completing the critical path in less than 17 days?
d. If you wanted to be at least 95 percent sure of completing the project on time,
what schedule durations would you quote?

2. Consider the project given in the next table. Find the probability that the project
will be completed within 70 and 80 days. What is the project expected duration
corresponding to 70% assurance.

Duration (days)
Activity Optimistic Most likely Pessimistic Dependencies
(a) (m) (b)
A 10 16 22 -
B 24 32 42 -
C 22 32 40 A
D 12 16 21 B
E 20 25 35 C, D
F 13 16 19 A, B

3. You have just decided to open your own geotechnical consulting firm. To secure
financing from the bank, you must present a plan for when you expect some
income from your firm. You sit down with your banker and develop this plan of
things that must be accomplished prior to starting your fist job. Estimated
duration and precedence relationships for these activities are shown in the table
below. Durations are given in days.
a. Draw a PERT diagram and determine the expected duration and the critical
path.

Construction Management 114 Dr. Emad Elbeltagi


b. Assume that the time required to complete a path is normally distributed.
Determine the probability of being able to start your fist job within 180 days.
c. To provide an allowance for unforeseen problems, you want to present your
banker with a time before you start operations based upon a 95 percent
confidence level. How many days from receipt of the loan will you tell banker
to expect you to start your first job?

Duration (days)
Most Immediate
Activity Description Optimistic Pessimistic
likely predecessor
(a) (b)
(m)
A Purchase land 30 60 90 -
B Hire staff 9 25 32 A
C Obtain permits 2 10 18 A
D Obtain business license 20 45 52 A
E Site preparation 3 4 11 C, D
F Construct office 21 25 41 E
G Paving & landscaping 9 12 15 F
H Stock soil testing 25 30 41 B, G
equipment
I Test equipment 11 12 16 H

Construction Management 115 Dr. Emad Elbeltagi


CHAPTER 6

SCHEDULING OF REPETITIVE PROJECTS

This chapter introduces new techniques for scheduling of multiple and linear projects
that involve a number of repetitive activities. These techniques include: the summary
diagrams and the line of balance (LOB). Examples of these projects are highways,
pipelines, and high-rise buildings. The objective of the LOB technique is to determine a
balanced mix of resources and synchronize their work so that they are fully employed
and non-interrupted. As such, it is possible to benefit from repetition, and the crews will
likely be able to spend less time and money on later units once they develop a learning
momentum. Another benefit of the LOB technique is its interesting representation of the
schedule, given the large amount of data for the repetitive units. This chapter introduces
the summary diagrams calculations presented on AON networks and integrated CPM-
LOB calculations that combine the benefits of CPM network analysis of a single unit and
the LOB analysis and representation.

6.1 Linear Projects

Linear projects are projects involving repetitive activities. They take their name from
either: (a) involving several uniform units of work such as multiple houses or typical
floors in a building; or (b) being geometrically linear such as highway, pipeline, and
utility projects. In both categories, however, some non-typical units could be involved
such as a non-typical floor in a high-rise building or a non-standard station in a highway
project. The activities in these non-typical units may certainly involve higher or lower
quantity of work than their counterparts in the typical units. To simplify the scheduling
task in these situations, we can assume that the project is comprised of (n) typical units,
with the activities in each unit having average quantity of the work in all units. As the

Construction Management 116 Dr. Emad Elbeltagi


number of units in a project increases, eventually the project becomes more complex and
more challenging.

6.2 Resource-Driven Scheduling

As we have seen in network scheduling, the basic inputs to critical-path analysis are the
individual project activities, their durations, and their dependency relationships.
Accordingly, the forward-path and backward-path calculations determine the start and
finish times of the activities. The CPM algorithm, therefore, is duration-driven.
Activities’ durations here are function of the resources that are required (rather than
available) to complete each activity. The CPM formulation, therefore, assumes that
resources are in abundance and cannot be used to determine what resources are needed in
order to meet known project deadline duration.

Resource-driven scheduling, on the other hand, is different and is more focused on


resources. Its objective is to schedule the activities (determine their start and finish times)
so that a project deadline is met using predefined resource availability limits. The line of
balance technique dealt with in this chapter is a resource-driven schedule.

6.3 Summary Diagrams

6.3.1 Summary Diagrams Using Two Relationships

One of the methods used for preparing schedules for linear and repetitive projects is the
summary diagrams. The diagram can be constructed as follows:

1. The repetitive units of one work type are represented by one activity. The duration
of this activity is the summation of the durations for individual units within the
same activity.

2. In order to maintain the logic of the network of such a project, relationships


between the start of each activity and the start of its successor and between the
finish of each activity and the finish of its successor are introduced as follows:

Construction Management 117 Dr. Emad Elbeltagi


Start to start relationship (LagSS) = duration of one unit of predecessor
Finish to finish relationship (LagFF) = duration of one unit of successor

3. The timings of each activity can be calculated as follows:

For any tow activities A and B (where A is a predecessor of B)

LagSS

ESA EFA ESB EFB


DA DB
LSA LFA LSB LFB

LagFF

ESB = ESA + LagSS

EFB = ESB + DB
Whichever is longer
EFB = EFA + LagFF

LFA = LFB – LagFF

LSA = LFA – DA Whichever is smaller


LSA = LSB – LagSS

4. The early and late start timings of each activity represent timings of the first
individual unit of this activity. On the other hand, the early and late finish timings
of each activity represent timings of the last individual unit of this activity.
Therefore, contract completion time equals finish time of the last activity in the
summary diagram.

Example 6.1

Activity Duration (one unit)


A 3
B 2
C 2

Construction Management 118 Dr. Emad Elbeltagi


Solution

The complete precedence diagram is given below:

0 3 3 5 5 7
A1 (3) B1 (2) C1 (2)
0 3 6 8 8 10

3 6 6 8 8 10
A2 (3) B2 (2) C2 (2)
3 6 8 10 10 12

6 9 9 11 11 13
A3 (3) B3 (2) C3 (2)
6 9 10 12 12 14

9 12 12 14 14 16
A4 (3) B4 (2) C4 (2)
9 12 12 14 14 16

Figure 6.1: PDM for Example 6.1

The summary diagram is given below:

0 12
Lag 2
A
0 12 12
3 14
Lag 3 Lag 2
B
6 8 14 5 16
C
Lag 2
8 8 16

Figure 6.2: The summary diagram for Example 6.1

From the summary diagram it is obvious that the critical activities are:

All units of activity A


The last unit of activity B
The last unit of activity C

Construction Management 119 Dr. Emad Elbeltagi


The first unit of both activities B and C has a total float of 3 units. The results of the
summary diagram are compatible with the results of the precedence diagram on the
assumption that duration of the units of an activity will not be changed.

6.3.2 Summary Diagrams Using One Relationship

One of the most important aspects of this procedure is the ability to specify one
relationship type among different activities also, it maintain the resource continuity
usage. To specify such relationships between two consecutive activities, the production
rate of each activity is compared with that of its successors. If di and ri denote unit
duration and production rate of activity i, respectively, then:

ri = 1/di (6.1)

The activity under consideration will be referred to as current activity. If rc and rs denote
production rates of current and succeeding activities, respectively, the possible cases that
may be encountered are:

rs < rc: This implies that the start of the 1st unit of the succeeding activity is
controlled by the finish of the 1st unit of the current activity. Then a Start-to-Start
(SS) relationship is to be specified. The lag associated with SS relationship (LagSS)
equals the unit duration of the current activity, as follows:

LagSS = dc (unit duration of current activity) (6.2)

Referring to Figure 6.3 and considering, for example, activities A and B, where B is
a succeeding activity to A. In this case, dC (activity A) = 1 and dS (activity B) = 2.5,
and consequently rC (activity A) = 1 and rS (activity B) = 0.4 (1/2.5). Therefore, SS
relationship exists between activities A and B and the corresponding lag = dC = 1.

rs > rc: In this case, the start of the last unit of the succeeding activity is controlled
by the finish of the last unit of the current activity. Then a Finish to-Finish (FF)
relationship exists. The lag associated with FF relationship (LagFF) equals the unit
duration of the succeeding activity, or:

Construction Management 120 Dr. Emad Elbeltagi


LagFF = ds (unit duration of succeeding activity) (6.3)

For example, relation between activities C and D in Figure 6.3 depicts a FF


relationship. In this case, rC (activity C) = 0.67 (1/1.5) and rS (activity D) = 2 (1/0.5),
and the corresponding LagFF(C-D) = dS = 0.5.

rs = rc: In this case, a SS or FF relationships can be specified, with lags as described


above.

After specifying the relationship type between consecutive activities, the duration of an
activity i; (Di), is calculated as the sum of unit duration of all repetitive units, i.e.:

Di = n × di (6.4)
Having the relationship type among activities determined with their associated lags,
network calculations similar to that of PDM are done. Forward path calculations are done
to determine the early times of each activity, while the backward path determines the late
times. Also, the critical activities are specified.

Example 6.2
A small project consists of 12 activities and each activity contains 5 repetitive units. The
data is presented in Table 6.1.

Solution
The example is solved through applying the following steps:

• Determine activities’ production rates ri. For example, the production rate for
activity A (using Equation 6.1), rA = 1/unit duration = 1/12. The activities’
production rates are shown underneath the activities in Figure 6.3.
• Calculate activities’ durations by multiplying activity unit duration by the number
of units using Equation 6.4. Also, the activities’ durations are shown in Figure
6.3.

Construction Management 121 Dr. Emad Elbeltagi


Table 6.1: Data for Example 6.2

Activity Duration (days) Predecessors

A 12 -
B 10 A
C 4 B
D 8 B
E 6 B
F 11 B
G 5 C, D, E, F
H 5 G
I 4 C, H
J 8 H
K 5 H
L 10 I, J

• Draw the precedence network and specify the relationship type between different
consecutive activities. Relationship type and the corresponding lag are shown in
Figure 6.3 along with the link between different activities.
• Perform the forward and backward paths calculations to determine the activities’
timing and mark the critical activities.
54 74 79 99
SS-4
C (20) I (20)
FF-4 61 81 SS-4 79 99
1/4 FF-4 SS-4
1/4
0 60 20 70 38 78 FF 65 90 70 95 75 115 83 133 133 133
FF SS SS
A (60) B (50) FF D (40) 5 G (25) 5 H (25) J (40) SS L (50) End (0)
10 8 5 8
0 60 20 70 45 85 65 90 70 95 75 115 83 133 133 133
1/12 1/8 1/5 1/5 1/8 1/10
1/10 FF-6 FF-5
46 76
E (30) SS-5
SS-10 55 85
1/6 FF-5

30 85 75 100
F (55) K (25)
30 85 108 133
1/11 1/5

Legend:
Early Start Early Finish SS: Start to Start relationship
Activity (Duration)
FF: Finish to Finish relationship
Critical Path
Late Start Late Finish
Activity Duration = unit duration x number of units
Figure 6.3: The summary diagram for Example 6.2

Construction Management 122 Dr. Emad Elbeltagi


6.4 Line of Balance (LOB)

6.4.1 Basic Representation

Let’s consider a medium-sized high-rise building of 40 typical floors. The construction of


each typical floor involves various inter-related activities. If a CPM network is to be
developed for the whole project, certainly it will be so complex and will be composed of
copies of the activities in a single floor. A Bar Chart of the project will still be so
complex and will not serve the purpose of a good communication tool between planners
and execution personnel.

A schedule representation that suits projects with repetitive activities is shown in Figure
6.4 between time on the horizontal axis and units on the vertical axis. This representation
shows the following information:

- Each sloping bar represents one activity (A, B, C, or D) in the project and the width
of the bar is the activity duration of one unit, which is uniform along all units;
- A horizontal line at any unit intersects with the activity bars at the planned start and
finish times of the work in that unit;
- A vertical line at any date (time) shows the planned work that should be
completed/started before and on that date;
- The slope of each activity represents its planned rate of progress and this is direct
function of the number of crews involved in the activity. The slope of the last activity
is the rate of delivery of the various units; and
- The finish time of the last unit in last activity represents the end date of the project.

It is possible also to add more details to the basic LOB schedule as shown in Figure 6.5.
The modified figure shows interesting information, as follows:

- The number of crews employed in each task is graphically represented with each
crew indicated by a different pattern. As such, the movement of the crews from one
unit to the other is shown;

Construction Management 123 Dr. Emad Elbeltagi


Project
End Date
11
Units

.
. A B C D
.
.
Start Finish
4 7 11 13 14½ 16½ 19 22
5

3
Delivery rate
1
Time
0 2 4 6 8 10 12 14 16 18 20 22 24 26 28 30

Figure 6.4: LOB representation

Buffer time
11
Units

.
. A B C D
.
. 3 crews 4 crews 3 crews 3 crews

Crew 3
3
Crew 2

Crew 1 Time
1
Buffer time
0 2 4 6 8 10 12 14 16 18 20 22 24 26 28 30

Figure 6.5: LOB schedule with crew details

- The three crews employed in activity (A) have different work assignments. Crew 1
works in four units (numbers 1, 4, 7, and 10) and leaves site on day 12. Similarly,
Crew 2 works on four units (numbers 2, 5, 8, and 11) then leaves site on day 13. Crew
3, on the other hand, works on three units only (numbers 3, 6, and 9) and leaves site
on day 11;

Construction Management 124 Dr. Emad Elbeltagi


- Each crew moves to a new unit as soon as it finishes with the previous one, without
interruption. As such, work continuity is maintained and the learning phenomenon
can lead to some savings in cost and time;
- To prevent interference among the sequential tasks of the LOB schedule in case an
activity is delayed, a buffer time may be introduced as shown, to act as a float time;
- When a slower activity is to follow a faster activity (e.g., C follows B), the activity C
can be scheduled starting from unit 1, immediately following the predecessor B.
Since interference can happen at unit 1, buffer time can added to the start of unit 1;
- When a faster activity is to follow a slower activity (e.g., B follows A), the activity B
needs to be scheduled starting at the top unit. If buffer time is to be added, it will be
added at top. Notice that the start of unit 1 in activity B has been delayed to allow the
task the proceed without interruption;
- Changing the production rate (slope) of any activity changes the project duration.
Even speeding one task may prove to be harmful to the project when the conflict
point changes from bottom to top; and
- If speeding an activity or relaxing it may result in a delay in the project, a good
scheduling strategy is to schedule the activities as parallel as possible to each other
and also parallel to a desired project delivery.

6.4.2 LOB Calculations

The objective of using LOB is to achieve a resource-balanced schedule by determining


the suitable crew size and number of crews to employ in each repetitive activity. This is
done such that: (1) the units are delivered with a rate that meets a pre-specified deadline;
(2) the logical CPM network of each unit is respected; and (3) crews’ work continuity is
maintained. The analysis also involves determining the start and finish times of all
activities in all units and the crews’ assignments.

The CPM-LOB formulation that achieves the above objective involves four main issues,
which are discussed in the next sections:

Construction Management 125 Dr. Emad Elbeltagi


- Crew synchronization and work continuity equation;
- Computation of a project delivery rate that meets a given deadline duration;
- Calculating resource needs for critical and non-critical activities; and
- Drawing the LOB schedule.

Crew synchronization

A simple relationship between the duration taken by a crew in one unit (D) and the
number of crews (C) to employ in a repetitive activity can be derived from the illustration
in Figure 6.6. In this figure, we have a 5-unit activity and 3 crews to use. Only one crew
is assumed to work in a single unit and the crew spends time (D) on the unit before
moving to another unit.

Crew 2
5
Units

Crew 1
4
Crew 3
3
Crew 2
2 No. of Crews
(C) = 3
Crew 1 R
1 Time
0 1 2 3 4 5 6 7
D = 3 days

Figure 6.6: Crew synchronization

Having 3 crews available for this activity, it is possible to schedule their movements in
and out of each unit, as shown in the figure, so that they are not interrupted and the work
progresses at a rate (R). For that work synchronization to happen, the following simple
relationship applies:

Number of Crews (C) = D x R (6.5)

In the example shown, C = 3; D = 3 days; then, R becomes 1 unit/day, according to


Equation 6.5. Therefore, it is possible to achieve work continuity given any change in the
number of crews (C) or crew formation (affects D) by adjusting the rate of progress (R).

Construction Management 126 Dr. Emad Elbeltagi


For example, if 4 crews become available, we can apply the same Equation 6.5 to
determine a faster progress rate of 1.25 units/day.

Crew 3

3
Units

Crew 2
2

1
R Crew 1
R
1
0 1 2 3 Time
D/C D/C D/C

Figure 6.7: Deriving Equation 6.5

Driving the relationship of Equation 6.5 is simple. By enlarging part of Figure 6.6 and
dividing the duration (D) among the (C) crews, the slope of the shaded triangle in Figure
6.6 becomes:
R = 1 / (D / C) (6.6)
and the time D/C becomes:
D/C=1/R (6.7)
Both equations lead to our formulation of C = D x R. Equation 6.6 also means that work
continuity is achieved by shifting the start of each unit from its previous one by a time
D/C or 1/R. This shift also has another practical meaning. Since each crew has part of its
duration non-shared with other crews, the chance of work delay is reduced when two
crews need the same equipment, or other resource, such as a crane on site.

Meeting a deadline duration

A basic objective in CPM-LOB calculation is to meet a given deadline for finishing a


number of (n) repetitive units; each has its own CPM network of component activities.
Using the illustration in Figure 6.8, it is possible to formulate a strategy for meeting the
deadline by calculating a desired rate of delivery (Rd) for the units, as follows:

Construction Management 127 Dr. Emad Elbeltagi


n

Units
.
.
.
n-1
2

R
1 Time

T1 = CPM Duration of Unit 1

TL = Project Deadline Duration

Figure 6.8: Calculating a desired rate of delivery

Rd = (n – 1) / (TL - T1) (6.8)

where, TL is the deadline duration of the project and T1 is the CPM duration of the first
unit. The delivery rate determined from Equation 6.8 is the minimum rate required to
meet the desired deadline. Any higher rate can expectedly produce shorter project
duration, however, more crews may need to be used and the schedule can be more costly.

Calculating resource needs

Once a minimum delivery rate (Rd) is calculated, it is desirable to enforce this rate on the
schedule of the repetitive activities to determine the resources needed to complete the
project on time. Equation 6.5, therefore, needs to be applied particularly to the critical
activities, which are the sequential tasks that take the longest path in the CPM network of
each unit. Non-critical activities, on the other hand, have float (TF) times and as such, we
can afford to relax them according to their float times to reduce cost. It is, therefore,
possible to modify Equation 6.8 and generalize it to determine a desired rate (Ri) for any
repetitive task (i), as follows:

Ri = (n – 1) / (TL - T1) + TFi (6.9)

Construction Management 128 Dr. Emad Elbeltagi


The physical meaning of Equation 6.9 is illustrated in Figure 6.9. In this figure, a 5-unit
project is shown with each unit consisting of a simple four-activity network. Three of the
four activities A, B, and C are sequential and each has 5-days duration. The fourth
activity D runs parallel to B and has a duration of 2 days only. Accordingly, A, B, and C
are critical activities while activity D is non-critical with Total Float (TF) of 3 days. As
shown in Figure 6.9, the slopes of activities A, B, and C are the same and are steep up.
The slope of activity (D), on the other hand, has been relaxed by simply starting unit 1 of
task D as early as possible while starting the last unit as late as possible (notice the
difference in the CPM networks of the first and the last units). In this manner, simple
analysis of the slope of activity D in the figure leads us to the formulation of Equation
6.9. Using this approach, the relaxation of non-critical activities can be performed
without violating any logical relationships or crew work continuity requirements.

TF = 3 D (2)
A (5) B (5) C (5)
Unit n

A B D C
n-1

(TL - T1 ) + TFD
TL - T1
Unit 1

A (5) B (5) C (5)


D (2) TF = 3

Figure 6.9: Utilization of float in LOB calculations

With the desired rates calculated for the individual activities, a generalized form of
Equation 6.5 can be used to determine the necessary number of crews (Ci) to use in each
activity (i), as follows:

Ci = Di x Ri (6.10)

Construction Management 129 Dr. Emad Elbeltagi


Another important consideration is that, in most cases, the number of crews calculated
using Equation 6.10 is not an integer value. Since a fraction of a crew is not possible, the
number of crews (Ci)’s has to be rounded up to determine the actual number of crews

(Cai)’s. As a consequence to that, the actual rates of progress in the activities (Rai)’s need
to be adjusted, as follows:

Cai = Round Up (Ci) (6.11)

Rai = Cai / Di (6.12)

Equations 6.9 to 6.12, therefore, become the basis of integrated CPM-LOB calculations.

Drawing the LOB Schedule

A LOB schedule becomes simple to draw when all activities run with an exactly similar
rate (i.e., activities run parallel to each other). However, due to the rounding of number of
crews in Equation 6.8, the activities’ actual rates (Rai)s calculated using Equation 6.9 will
not be parallel. Drawing the LOB schedule as such requires extra care as conflict points,
either at the top unit or at the first unit, will be introduced due to the difference in
progress rates from one activity to the other. As explained earlier, sometimes speeding an
activity will cause a net delay in the whole project, if work continuity is to be maintained.
Therefore, some non-critical activities may end up being delayed even in some situations
violating the logical relationships or becomes critical themselves. Also, in some
situations, the end schedule may slightly extend beyond the deadline. In this case, a
simple approach to use is to re-schedule the project with a deadline duration that is
slightly (one or two days) shorter than originally desired.

To draw the LOB schedule using the activities actual rates (Rai)s, we need to proceed in a
forward path, following the logical relationships in the CPM network. When an activity is
considered, its predecessors are first examined to identify their largest finish times, which
are then considered as a boundary on the start of the current activity. Drawing the

Construction Management 130 Dr. Emad Elbeltagi


schedule by hand is simple when the network is small and can be done with varying
levels of detail as shown in Figures 6.4 and 6.5.

5
Unit

A B C D
3
Critical Activities
2

Crew 1 Time
1
0 2 4 6 8 10 12 14 16 18 20
2
Earliest start
3 line of all
Latest finish
line of all
E successors

4 Predecessors

Non-Critical Activity
5 with its Boundary Lines

Figure 6.10: Alternative LOB representation

In terms of presentation, showing all the activities on the same grid results in a crowded
schedule and can be confusing even for a small network. Two interesting approaches can
be used to circumvent this problem. One approach is to draw the critical path on one grid
and draw the other paths, each on a different grid. The benefit of drawing these paths is to
help visualize the successor/predecessor relations for any given task, and accordingly
facilitate any desired changes to rates or crews. The second approach is to extend the
LOB representation to show the non-critical activities on a mirrored grid as shown in
Figure 6.10.

Example 6.3

The activities involved in the construction of one kilometer of a pipeline are given
together with their estimated durations in the table below. The project consists of 10
similar kilometers. Calculate the number of crews needed for each activity if the deadline

Construction Management 131 Dr. Emad Elbeltagi


for completing the project is 40 days and draw the LOB schedule. Assume one day buffer
time between activities.

Table 6.2: Data for Example 6.3

Activity Duration Preceding


Activity name
no. (days) activities
1 Locate and clear 1 -
2 Excavate 3 1
3 String pipe 1 1
4 Lay pipe 4 2,3
5 Pressure test 1 4
6 Backfill 2 5

Solution

Figure 6.11 shows the CPM calculations for a single unit of the project. In this step, we
determine the duration of a single unit and identify the critical path.
Note that the one day buffer time is set as a lag between activities.

0 1 2 5 6 10 11 12 13 15
-1 2 (3) -1 4 (4) -1 5 (1) -1 6 (2)
1 (1)
0 1 2 5 6 10 11 12 13 15
-1 -1
2 3
3 (1)
4 5

Figure 6.11: CPM calculations for Example 6.3

T1 = 15 day TL= 40 day N = 10 units


Ri = (n-1) / TL - T1 + TFi = 9 / (25 + TFi)

Construction Management 132 Dr. Emad Elbeltagi


Table 6.3: LOB calculations for Example 6.3

Duration Total Cai =


Activity Ri = 4 / (25+TFi) Ci =Di x Ri Rai = Cai / Di
Di Float Round up Ci

1 1 0 0.36 0.36 1 1
2 3 0 0.36 1.08 2 0.667
3 1 2 0.333 0.333 1 1
4 4 0 0.36 1.44 2 0.5
5 1 0 0.36 0.36 1 1
6 2 0 0.36 0.72 1 0.5

No. of units
9 10 11 12 15.5 18.5 24 28 29 30 40 42
10

1 3 2 4 5 6

1
0 1 2 3 5 6 10 20 21 22 24 Time

Figure 6.12: LOB for Example 6.3

Construction Management 133 Dr. Emad Elbeltagi


6.5 Exercises

1. The construction of a housing project involves the activities given in the following
table. The contract is for the construction of twelve houses in 60 days. The man-
hours for each activity and the crew size/house are also given. Prepare an LOB
schedule for the contract. Assume a minimum buffer time of one day and six 8-
hour days per week. What is the overall project duration and when will the first
team of roof leave the site.

Activity Description Predecessors Man-hours Team Size / Section


10 Substructure - 100 4
20 Superstructure 10 200 8
30 Roof 20 60 4
40 Carpenter 30 90 6
50 Plumber 30 45 3
60 Electrician 30 40 3
70 Plaster 40, 50, 60 120 8
80 Final fix 70 350 24

2. The following network diagram represents the activities involved in a single


house. Each activity shows the man-hours needed and the number of crew
members. Assume 8 working hours per day and one day buffer time between
activities.

If you are to construct these tasks for 5 houses in 24 days, calculate the number of
crews that need to be involved in each activity. Draw the schedule and define the
day numbers in which each crew enters and leaves the site.

Sanit. Main Footing 2


48 Mhrs 64 Mhrs
3 Men 2 Man

Excavation Footing 1 Wall 1 Wall 2


48 Mhrs 64 Mhrs 72 Mhrs 72 Mhrs
3 Men 2 Man 3 Men 3 Men

Construction Management 134 Dr. Emad Elbeltagi


3. The construction plan for a house is as follows, with activities durations in days:

D (2)

A (5) B (5) C (7) E (3) F (5)

a. Calculate a weekly target rate to be used for scheduling a project of 30


repetitive houses, if all crews are working five 8-hour days per week and the
project has to be delivered in 85 days.
b. Given a desired target rate of four units per week, what is the number of
crews to be employed in activity B.
c. Using the same number of crews obtained in (b), activity B has to be speed-
up to a target rate of 5 units per week. Calculate how much time a crew needs
to cut from the duration of each unit. Work continuity and crew
synchronization has to be maintained.

4. The activities involved in the construction of one kilometer of a pipeline are given
together with their estimated durations in the table below. Each of the given
activities will be done using a separate gang. The project consists of 20 similar
kilometers. Construct the project summary diagram using the precedence
notations, and state the critical activities.

Activity Duration Preceding


Activity name
no. (weeks) activities
1 Locate and clear 1 -
2 Excavate 3 1
3 String pipe 1 1
4 Lay pipe 4 2,3
5 Pressure test 1 4
6 Backfill 2 5

Construction Management 135 Dr. Emad Elbeltagi


CHAPTER 7

RESOURCES MANAGEMENT

As we have seen in network scheduling, the basic inputs to critical-path analysis are the
individual project activities, their durations, and their dependency relationships.
Accordingly, the forward-path and backward-path calculations determine the start and
finish times of the activities. The CPM algorithm, therefore, is duration-driven.
Activities’ durations here are function of the resources that are required (rather than
available) to complete each activity. The CPM formulation, therefore, assumes that all
the resources needed for the schedule are available. This assumption, however, is not
always true for construction projects. Under resource constraints, the schedule becomes
impractical, cost and time are not accurate, and resources may not be available when
needed. In order to deal with such issue, a proper management of available resources is
required to adjust the schedule accordingly.

When a project plan is first devised it is likely that the plan will identify peaks of
resource requirements. However, given the finite nature of resource availability, it may
be impractical to meet such peak resource needs. Ideally, there should be an even
demand for resources over the entire project duration, with a smooth increase at the
beginning of a project and a smooth decrease at the end. Given the limited nature of
resources, consideration should be given to the project resource requirements; the project
plan should be refined when necessary so that it is practical.

7.1 Resource Definition

The first step in resource management is to decide exactly what resources are considered
important enough to be managed. While the most resource used is people or workers
(such as welders or carpenters), it may also include other resources such as machines

Construction Management 136 Dr. Emad Elbeltagi


(such as an excavator or loader), space on a project where space is restricted and where
this restriction limits the amount of other resources which can be deployed at any one
time, financial resources (money) that are needed to perform the required work, or
materials needed to accomplish different activities. Generally, a resource can be defined
as any thing (labor, equipment, material, money, etc.) that is needed to have the work
done.

Often resources are specified in terms of the number of units of resource required, e.g., 5
welders or 3 computer programmers. Alternatively, resources may be specified in terms
of the hours or days that a specific resource is required, e.g., 40 welder-hours or 24 man-
days.

Resources may be considered as consumable, such as materials that may be used once
and once only, or non-consumable, such as people, which may be used again and again.
The way in which consumable resources are used is not critical as long as they are used
efficiently. However, the way in which non-consumable resources are used can have a
significant impact on the project. Resource management is therefore mainly concerned
with non-consumable resources.

Also, resources may be classified according to their importance to key resources,


secondary resources and general resources. Key resources are the most important,
expensive and non-available resources in the project such as skilled labors, or equipment.
These types of resources will have a great attention in the resource scheduling process.
Secondary resources are those resources which have no constraints on their availability,
such as normal labor. General resources are defined as those resources that are used by all
or most of the activities on the project such as site overheads. General resources will not
be included in the resource management described later.

7.2 Resource Management

The most important resources that project managers have to plan and manage on day-to-
day basis are people, machines, materials, and money. Obviously, if these resources are
available in abundance then the project could be accelerated to achieve shorter project

Construction Management 137 Dr. Emad Elbeltagi


duration. On the other hand, if these resources are severely limited, then the result more
likely will be a delay in the project completion time. In general, from a scheduling
perspective, projects can be classified as either time constrained or resource constrained.

Resource leveling (smoothing)

A project is classified as time constrained in situations where the project completion time
can not be delayed even if additional resources are required. However, the additional
resource usage should be no more than what is absolutely necessary. Accordingly, the
primary focus, for purposes of scheduling, in time constrained projects is to improve
resource utilization. This process is called resource leveling or smoothing. It applies
when it is desired to reduce the hiring and firing of resources and to smooth the
fluctuation in the daily demand of a resource, as shown in Figure 7.1. In this case,
resources are not limited and project duration is not allowed to be delayed. The objective
in this case is to shift non-critical activities of the original schedule, within their float
times so that a better resource profile is achieved.

Resource Resource

Resource
limit

Time Time
Resource profile with high Resource profile with no
resource fluctuation and fluctuation (Ideal usage) and
exceeding limit below resource limit
- Project time: constrained
- Resources: unconstrained
- Objective: even resources usage

Figure 7.1: Resource leveling (smoothing)

Resource scheduling

On the other hand, a project is resource constrained if the level of resource availability
cannot be exceeded. In those situations where resources are inadequate, project delay is

Construction Management 138 Dr. Emad Elbeltagi


acceptable, but the delay should be minimal. The focus of scheduling in these situations
is to prioritize and allocate resources in such a manner that there is minimal project delay.
However, it is also important to ensure that the resource limit is not exceeded and the
technical relationships in the project network are respected.

7.3 Resource Allocation

Resource allocation, also called resource loading, is concerned with assigning the
required number of resources identified for each activity in the plan. More than one type
of resource may be assigned to a specific activity. For example, fixing steel plates on a
bridge deck may require different types of resources such as: welders, laborers and a
certain type of welding machine. From a practical view, resource allocation does not have
to follow a constant pattern; some activities may initially require fewer resources but may
require more of the same resources during the later stages of the project.

7.4 Resource Aggregation (Loading)

After each activity has been assigned its resources, the next step is to aggregate the
resources used by all activities. Resource aggregation is simply the summation, on a
period-by-period basis, of the resources required to complete all activities based on the
resource allocation carried out previously. The results are usually shown graphically as a
histogram. Such aggregation may be done on an hourly, daily, or weekly basis,
depending on the time unit used to allocate resources. When a bar chart is used, the
resource aggregation is fairly simple and straightforward. For a given bar chart, a
resource aggregation chart can be drawn underneath the bar chart. However, a separate
graph will be required for each resource type.

An example is shown in Figure 7.2, where, for a particular resource, the required resource
units for each time period are written on the bar chart. The total number of resource units
for each time period can then be summed and a resource aggregation or load chart can be
produced as presented underneath the bar chart. Thus, having a project scheduling is
necessary to facilitate the bar chart drawing.

Construction Management 139 Dr. Emad Elbeltagi


Figure 7.2: Resource aggregation

Figure 7.3: Resource aggregation chart showing resource requirements associated with
earliest and latest times along with highlighted resource requirements for critical
activities

Construction Management 140 Dr. Emad Elbeltagi


The non critical activities, activities which are not on the critical path, do not have fixed
starting and finishing times but are constrained by the earliest and latest starting and
finishing times. This situation offers the planner chance for adjusting the demand for
resources. Figure 7.3 illustrates such situation, which shows the resource aggregation
when the activities scheduled on their early times and late times. It can be seen that the
resource requirements that arise when both earliest and latest start times are considered
are different. The shaded area represents the resources required by the critical activities,
as these activities have a fixed position because their early times equal their late time.
Figure 7.3 shows, also, the accumulation of resources at the beginning of the project
when the activities scheduled on their early time. On the other hand, the resources
accumulate at the end of the project when the activities scheduled on their late times.

7.5 Resource Leveling (Smoothing)

As shown in Figure 7.3, the problem of resource fluctuation appears after the initial
scheduling of the project without considering the resources. The peaks and valleys in the
resource profile indicate high day-to-day variation in the resource demand. Resource
smoothing is the process that attempts to determine a resource requirement that is
"smooth" and where peaks and valleys are eliminated. For example, even if 7 units of a
given resource are available at any one time, utilizing 5 of these units each week is
preferable than using 4 units for the first week, 7 the next, 2 the next and so on. Even if
there is no limit to the amount of any one resource available, it is still desirable that
resource usage is as smooth as possible. Given that the resource requirements of those
activities on the critical path are fixed, some order or priority needs to be established for
selecting which activity and which particular resource associated with this activity should
be given priority in the smoothing process.

Resource leveling heuristics shift non-critical activities within their float times so as to
move resources from the peak periods (high usage) to the valley periods (low usage),
without delaying the project (i.e., area underneath the resource profile remains constant).
Usually, project managers may prefer having a desired resource profile in which the

Construction Management 141 Dr. Emad Elbeltagi


resource usage starts with low values and then the resources are build up till its maximum
values and starts to decrease as the project approaches its end as shown in Figure 7.4.

12

10
Resource units

0
1 2 3 4 5 6 7 8 9 10 11 12 13 14 15 16 17 18 19
Time

Figure 7.4: Preferred resource usage

7.5.1 Method of Moments for Resource Smoothing

The method of minimum moment is well known heuristic algorithm for smoothing
resource profiles. The Minimum Moment Algorithm has been used as a heuristic
approach to calculate a measure of the fluctuations in daily resource demands. This is
represented in Figure 7.5, where Histogram 1 and Histogram 2 are two alternative
resource histograms, both having a total area of 40 resource days (i.e., equal total
resource demands). Histogram 1 is an ideal one with a constant daily demand of 4
resource units, no day-to-day resource fluctuations, and the resource will be released after
day 10. Histogram 2, on the other hand, exhibits high resource fluctuation with daily
demand in the range of 2 to 6 resource units, and the resource will not be released until
the end of day 12. The moment (Mx) of both histograms around the horizontal axis (days)
are 160 and 166, respectively, representing a better resource leveling of Histogram 1.

The moment Mx is calculated by summing the daily moments, as follows:

n
 1 
M =
x ∑ ( 1 x Re source Demand j ) x 2 Re source Demand j  ( 7.1 )
j =1
Construction Management 142 Dr. Emad Elbeltagi
Where, n is the working-day number of the project’s finish date. Or, for comparison
reasons, equation (1) becomes:
n
M = ∑ ( Re source Demand )2 ( 7.2 )
x j
j =1

Histogram 1: Mx = 160
Histogram 2: Mx = 166

Figure 7.5: Moment calculations of resource histogram

Having the moment calculations defined, a project manager may use them as to minimize
the Mx to reduce daily resource fluctuations.

7.5.2 Heuristic Procedure for Resource Smoothing

This section describes another way to smooth resource profile using some heuristic rules.
This method can be summarized in the following steps:

- Prepare a complete activity schedule.


- Draw a bar chart of the project under study based on ES timing of the activities.
- Critical activities to be drawn first (as these activities will not be moved).
- Write the resource usage above each bar of the related activity.
- Draw the FF as dashed line beside the upper side of the bar and the TF beside the
lower side.

Construction Management 143 Dr. Emad Elbeltagi


- Aggregate (determine the resource sum) the resources in each time period.
- Calculate the total usage of resources = ∑ unit period usage.
- Calculate the average resource usage = ∑ usage / utilization period.
- Shift non-critical activities within their FF first, then their TF to decrease the peaks
and raise the valleys.
- Revise activities floats.
- Aggregate resources in each time period after shifting any activity.
- When shifting activities, it is preferred to start with the activities that have no
successors, as shifting these activities will not affect other activities. Also, by
shifting these activities, a float will be created for its predecessors.
- Shift activities only that will enhance the resource profile.

Example 7.1

The activities involved in the construction of a certain project are given in Table 7.1. One
resource type will be used during the contract. Determine minimum level of the resource
required to complete the project.

Table 7.1: Data for Example 7.1

Activity Duration (Weeks) Predecessors Resource (units/week)

A 0 - 0
B 2 1 0
C 5 1 2
D 3 1 2
E 2 2 1
F 6 2 2
G 6 3 3
H 6 4 1
I 4 4 0
J 2 5, 6 4
K 7 6, 7 2
L 3 2, 8 2
M 2 2, 8, 9 4
N 2 10, 11, 12, 13 0

Construction Management 144 Dr. Emad Elbeltagi


Solution

The project network is shown in Figure 7.6 with the activity timings and project
completion time of 20 weeks. Table 7.2 shows the activities timings and floats.

2 4
E (2)
14 16

0 2 2 8 8 10
B (2) F (6) J (2)
3 5 5 11 16 18

0 0 0 5 5 11 11 18 18 20
A (0) C (5) G (6) K (7) N (2)
0 0 0 5 5 11 11 18 18 20

0 3 3 9 9 12
D (3) H (6) L (3)
6 9 9 15 15 18

3 7 9 11
I (4) M (2)
12 16 16 18

Figure 7.6: Precedence network of Example 7.1

Figure 7.7 shows the bar chart and the resource histogram of the project and the weekly
usage of the resources and the total usage of 90 resource units. As shown in the resource
histogram, the peak resource usage is 13 units and the minimum usage is 2 units. The
total resource usage equals 90 units with utilization period of 18 weeks. Then, the
average resource usage equals 5 units (=90/18=5).

Construction Management 145 Dr. Emad Elbeltagi


Table 7.2: Activities times and floats of Example 7.1

Activity ES EF FF TF

A 0 0 0 0
B 0 2 0 3
C 0 5 0 0
D 0 3 0 6
E 2 4 4 12
F 2 8 0 3
G 5 11 0 0
H 3 9 0 6
I 3 7 2 9
J 8 10 8 8
K 11 18 0 0
L 9 12 6 6
M 9 11 7 7
N 18 20 0 0

The resource demand on weeks 9, 10, and 11 is high, while it is low in weeks 13 through
18. Accordingly, the solution process will try to sift the resources from that peak period
to the period of low usage. The following activities will be shifted:
- Activity M has a free float of 7 weeks. Shifting activity M by 7 weeks will reduce
the peak usage of the resource on weeks 10 and 11 and increase the usage on weeks
17 and 18. Also, shifting activity M will give chance for preceding activities to be
shifted.
- Activity J can be shifted by 6, however it has 8 weeks free float. By shifting
activity J, the free float of both activities E and F are changed.
- Shift activity L by 2 weeks to optimize the resource usage. The free float of activity
will be changed to 2 weeks.
- Next, shift activity E by 10 weeks to improve the resource usage.
- Shift activity H by 2 weeks.
- Finally, shift activity F by 1 week.

Construction Management 146 Dr. Emad Elbeltagi


0 1 2 3 4 5 6 7 8 9 10 11 12 13 14 15 16 17 18 19 20
C, 2
G, 3
K, 2
N, 0

B, 0
D, 2
E, 1
F, 2
H, 1
I, 0
J, 4
L, 2
M, 4

4 4 7 6 5 6 6 6 8 13 9 4 2 2 2 2 2 2 ∑= 90

Units
14
13
12
11
10
9
8
7
6
5
4
3
2
1
0 Time
1 2 3 4 5 6 7 8 9 10 11 12 13 14 15 16 17 18 19 20

Figure 7.7: Bar chart and resource histogram before leveling of Example 7.1

The heuristic procedure for leveling project resource is shown in Figure 7.8. In each step,
the resources are aggregated to ensure that shifting an activity improves the resource
utilization. The resource histogram for the leveled project is shown in Figure 7.9.

Construction Management 147 Dr. Emad Elbeltagi


0 1 2 3 4 5 6 7 8 9 10 11 12 13 14 15 16 17 18 19 20
C, 2
G, 3
K, 2
N, 0

B, 0
D, 2
E, 1
F, 2
H, 1
I, 0
J, 4
L, 2
M, 4

4 4 7 6 5 6 6 6 8 13 9 4 2 2 2 2 2 2 ∑= 90
M (7 weeks) -4 -4 +4 +4
4 4 7 6 5 6 6 6 8 9 5 4 2 2 2 2 6 6
J (6 weeks) -4 -4 +4 +4
4 4 7 6 5 6 6 6 4 5 5 4 2 2 6 6 6 6
L (2 weeks) -2 -2 +2 +2
4 4 7 6 5 6 6 6 4 3 3 4 4 4 6 6 6 6
E (10 weeks) -1 -1 +1 +1
4 4 6 5 5 6 6 6 4 3 3 4 5 5 6 6 6 6
H (2 weeks) -1 -1 +1 +1
4 4 6 4 4 6 6 6 4 4 4 4 5 5 6 6 6 6
F (1 week) -2 +2
4 4 4 4 4 6 6 6 6 4 4 4 5 5 6 6 6 6

Figure 7.8: Applying heuristic procedure for resource leveling


Units
7

0 Time
1 2 3 4 5 6 7 8 9 10 11 12 13 14 15 16 17 18 19 20
Figure 7.9: Resource histogram for Example 6.1 after leveling

Construction Management 148 Dr. Emad Elbeltagi


Example 7.2

The activities involved in the construction of a small project are given in Table 7.3. The
resource usage for each activity is shown in Table 7.3. Smooth the resource so that a
preferred resource usage is obtained.

Table 7.3: Data for Example 7.2

Activity Duration (Weeks) Predecessors Labors (units/week)

A 3 - 9
B 5 - 6
C 1 - 4
D 1 A 10
E 7 B 16
F 6 B 9
G 4 C 5
H 3 C 8
I 6 D, E 2
J 4 F, G 3
K 3 H 7

Solution

The precedence network of the project is shown in Figure 7.10 with the activity timings
and project completion time of 18 weeks.

To achieve a resource profile with less resource load at the beginning and build up
towards the middle of the project and decreases towards the end, the following activities
will be shifted:
- Shift activity K by 11 weeks, this activity has a free float 11 weeks.
- Shift activity H by 11 weeks (it has 11 weeks free float).
- Activity A will be shifted by one week, accordingly, activity D will e shifted by
one week. This is because activity A has no free float.
- Shift activity F by 3 weeks and accordingly, activity J will be shifted 3 weeks
because activity F has no free float.
- Finally, shift activity G by 3 weeks.

Construction Management 149 Dr. Emad Elbeltagi


3 4
D (1)
11 12

0 3 5 12 12 18
A (3) E (7) I (6)
8 11 5 12 12 18

0 0 0 5 5 11 11 15 18 18
A (0) B (5) F (6) J (4) End (0)
0 0 0 5 8 14 14 18 18 18

0 1 1 5 4 7
C (1) G (4) K (3)
9 10 10 14 15 18

1 4
H (3)
12 15

Figure 7.10: Precedence network of Example 7.2

Figure 7.11 shows the bar chart and the resource histogram of the project and the weekly
usage of the resources.

0 1 2 3 4 5 6 7 8 9 10 11 12 13 14 15 16 17 18
B, 6
E, 16
I, 2

A, 9
C, 4
D, 10
F, 9
G, 5

H, 8
J, 3
K, 7

19 28 28 29 18 32 32 25 25 25 25 19 5 5 5 2 2 2

Figure 7.11: Bar chart and resource aggregation of Example 6.2

Construction Management 150 Dr. Emad Elbeltagi


The heuristic procedure for leveling project resource is shown in Figure 7.12. In each
step, the resources are aggregated to ensure that shifting an activity improves the resource
utilization. The resource histogram for the leveled project is shown in Figure 7.13.

0 1 2 3 4 5 6 7 8 9 10 11 12 13 14 15 16 17 18
B, 6
E, 16
I, 2

A, 9
C, 4
D, 10
F, 9
G, 5

H, 8
J, 3
K, 7

19 28 28 29 18 32 32 25 25 25 25 19 5 5 5 2 2 2
K (11 weeks) -7 -7 -7 +7 +7 +7
19 28 28 29 11 25 25 25 25 25 25 19 5 5 5 9 9 9

H (11 weeks) -8 -8 -8 +8 +8 +8
19 20 20 21 11 25 25 25 25 25 25 19 13 13 13 9 9 9
A (1 week ) -9 +9
D (1 week) -10 +10
10 20 20 20 21 25 25 25 2525 25 19 13 13 13 9 9 9
F (3 weeks) -9 -9 -9 +9 +9 +9
J (3 weeks) -3 -3 -3 +3 +3 +3
10 20 20 20 21 16 16 16 25 25 25 25 19 19 13 12 12 12
G (3 weeks) -5 -5 -5 +5 +5 +5
10 15 15 15 21 21 21 21 25 25 25 25 19 19 13 12 12 12

Figure 7.11: Solution of Example 7.2

Construction Management 151 Dr. Emad Elbeltagi


30

25

20
Units

15

10

0
1 2 3 4 5 6 7 8 9 10 11 12 13 14 15 16 17 18
Time (weeks)
Figure 7.12: Resource histogram after leveling of Example 7.2

7.6 Scheduling with Limited Resource

Shortage of resources is a major challenge for construction projects. Often, the number of
skilled labor is limited, related equipment has to be returned as soon as possible, and / or
a limited require our special consideration. Scheduling under these resource constraints
becomes a complex problem, particularly when more than one resource is limited.
Activity
2 2 2
A
1 1 1
B
2 2 2 2
C
2 2 2 2 2
D
2 2 2
E

1 2 3 4 5 6 7 8 9 10 11 12 13 14 Time periods
2 2 5 3 3 2 2 2 2 2 2 2 2 2 Resource usage

6
5
5
Resource available 4 men /day 4
3
3
2 2
2
1

Figure 7.13: Resource needed exceed resource limit

Construction Management 152 Dr. Emad Elbeltagi


The technique that deals with limited resources has been referred to as "resource-
scheduling" or "resource-constrained scheduling”. The problem of resource-constrained
scheduling appears after the initial network analysis is conducted and a bar chart is
drawn. A resource conflict occurs when at any point in the schedule several activities are
in parallel and the total amount of required resource(s) exceeds the availability limit, for
any of the resources required in these parallel activities. The situation is illustrated in
Figure 7.13 with activities A, B, and C that, at time period 3, require 5, while 4 are only
available per day.

The simple solution to that situation is that we can prioritizing the parallel activities,
given the resource to higher priority activities and delay the others until the earliest time
the resource become available again. Notice that if we delay an activity at time period 3,
to solve the situation, we may end up with another resource conflict later in time.
Continuing with identifying next conflict points and resolving them, determines the new
schedule and the new project duration. Accordingly, the objective in such situation is to
delay some activities so that the resource conflict is resolved and the project delay is
minimized.

Various models were developed in an attempt to answer this question, and thus optimize
resource-scheduling decisions. Early efforts used mathematical optimization, dynamic
programming, and linear programming. These models, however, were applicable only to
very small size problems. On the other hand, heuristic solutions for this problem have
been developed. Heuristic solutions, in general, use simple rules of thumb to provide
approximate but good solutions that are usable for large scale problems. An example of
these rules of thumb is that the resource can be assigned to activity (ies) having smaller
total float values than others (indicating a desire not to delay the critical and close-to-
critical activities). Figure 7.14 show an example where priority was assigned to the
activities having least total float when conflict arises.

Construction Management 153 Dr. Emad Elbeltagi


Resource Activity
2 A
2 B
1 C
1 D
2 E
Resource limit = 2 2 2 4 4 1 1 2 2

Resource Activity
2 A
2 B
1 C
1 D
2 E
2 2 2 2 1 1 2 2 2 2

Figure 7.14: Resource scheduling using least TF rule

These heuristic rules are based mainly on activity characteristics. The two most effective
and commonly used heuristic rules are the least total-float (LTF) and the earliest late-start
(ELS). These two rules have been proven to provide identical results, with the ELS rule
being advantageous compared to the LTF rule. This is because the value of the late-start
derived from the original CPM calculations, unlike the total-float, need not to be changed
every time an activity is rescheduled due to insufficient resource availability. As such, the
ELS rule can be applied with much less computational effort than the LTF rule, and
accordingly has been used as a basis for the resource scheduling.

The resource scheduling procedure using the ELS is outlined in Figure 7.15. It starts from
the project start time and goes through cycles of identifying eligible activities according
to the network logic.

7.7 Case Study

The procedure described earlier will be demonstrated on a case study project having 20
activities and each activity uses 6 resources with their limits given in Table 6.4. It is
required to schedule the project so that the daily resource requirements do not exceed the
resource limits.

Construction Management 154 Dr. Emad Elbeltagi


Draw the CPM network and calculate the
late start (LS) values of all activities

Current Time = 0

Select eligible activities (activities having


their predecessors completed, in addition
to any continuing ones)

Sort eligible activities in an ascending


order according to their LS values and
pick the first activity

Available res. NO
>
Required res.?

Delay this
Current time = Lowest YES activity
finish time of the eligible
activities Select next Start this activity and
eligible activity adjust the resource pool

NO All eligible
activities are
scheduled?

YES

All
NO activities are
scheduled?

YES

End

Figure 7.15: Resource scheduling procedure using the ELS rule

Construction Management 155 Dr. Emad Elbeltagi


Table 7.4: Case study data

Daily Resource Requirements


Activity Duration
(days) Predecessors R1 R2 R3 R4 R5 R6
(1) (2) (3) (4) (5) (6) (7) (8) (9)
A 6 ---- 5 2 2 2 7 4
B 3 ---- 3 5 2 3 9 6
C 4 A 2 4 4 2 3 1
D 6 ---- 5 4 3 5 5 4
E 7 A, B 3 5 2 3 8 0
F 5 C 4 1 4 9 2 5
G 2 D 4 1 4 3 9 8
H 2 A, B 5 5 4 0 9 1
I 2 G, H 3 2 4 3 4 2
J 6 F 1 5 4 6 7 3
K 1 C, E 3 3 2 4 5 1
L 2 E, G, H 3 2 2 8 3 4
M 4 I, K 2 2 2 2 4 8
N 2 F, L 1 4 4 3 4 1
O 3 L 5 5 4 6 2 3
P 5 J, M, N 3 2 3 4 7 8
Q 8 O 4 5 4 2 3 4
R 2 D, O 5 3 3 3 7 8
S 6 P, R 2 4 6 2 3 4
T 2 Q 1 6 2 7 5 2

Daily Resource Limits 7 10 10 16 18 13

The CPM network of the case study is shown in Figure 7.16, indicating project duration
of 32 days, without considering the resource limits (constraints). Applying the heuristic
procedure to consider resource constraints resulted in the manual solution given in Table
7.5, with 49 days project duration. In Table 6.5, the first 10 columns represent the
activities’ data, while the last 2 columns are the scheduling decisions made at each cycle.

Construction Management 156 Dr. Emad Elbeltagi


Figure 7.16: Precedence network of the case study project

According to the project network of Figure 7.16, activities A, B, and D are at the start of
the project and thus they become eligible for scheduling at current time = 0 (beginning of
the project), as shown in the first cycle of Table 7.5. The eligible activities were sorted by
their late-start values (the criteria used for assigning resources, as shown in column 9).
Considering these three activities in their priority order, available resources were enough
to start activity A, but the remaining amounts of resources were not enough for either B
or D. As such, activity A was scheduled to start at time 0 and to end at time 6 (duration =
6 days), while activities B and D were delayed till the earliest time more resources
became available (day 6).

At day 6, activity A was finished, and as such, all its immediate successors become
eligible for scheduling (unless they have other unfinished predecessors), in addition to B
and D which were delayed in the previous cycle. After sorting and considering these
activities one-by-one, B and C could start, while D was delayed.

Construction Management 157 Dr. Emad Elbeltagi


Table 7.5: Solution of the case study project
Resources
Time Eligible Late Finish
Activities R1=7 R2=10 R3=10 R4=16 R5=18 R6=13 Start Duration Decision Time
(1) (2) (3) (4) (5) (6) (7) (8) (9) (10) (11) (12)
A 5 2 2 2 7 4 0 6 Start 6
B 3 5 2 3 9 6 6 3 Delay -
0
D 5 4 3 5 5 4 7 6 Delay -
B 3 5 2 3 9 6 6 3 Start 9
6 C 2 4 4 2 3 1 6 4 Start 10
D 5 4 3 5 5 4 7 6 Delay -

C 2 4 4 2 3 1 6 4 Continue 10
D 5 4 3 5 5 4 7 6 Start 15
9
E 3 5 2 3 8 0 9 7 Delay -
H 5 5 4 0 9 1 13 2 Delay -

D 5 4 3 5 5 4 7 6 Continue 15
E 3 5 2 3 8 0 9 7 Delay -
10
F 4 1 4 9 2 5 10 5 Delay -
H 5 5 4 0 9 1 13 2 Delay -

E 3 5 2 3 8 0 9 7 Start 22
F 4 1 4 9 2 5 10 5 Start 20
15
G 4 1 4 3 9 8 13 2 Delay -
H 5 5 4 0 9 1 13 2 Delay -

E 3 5 2 3 8 0 9 7 Continue 22
G 4 1 4 3 9 8 13 2 Start 22
20
H 5 5 4 0 9 1 13 2 Delay -
J 1 5 4 6 7 3 15 6 Delay -

H 5 5 4 0 9 1 13 2 Start 24
22 J 1 5 4 6 7 3 15 6 Start 28
K 3 3 2 4 5 1 16 1 Delay -
J 1 5 4 6 7 3 15 6 Continue 28
I 3 2 4 3 4 2 15 2 Start 26
24
K 3 3 2 4 5 1 16 1 Start 25
L 3 2 2 8 3 4 17 2 Delay -
I 3 2 4 3 4 2 15 2 Continue 26
25 J 1 5 4 6 7 3 15 6 Continue 28
L 3 2 2 8 3 4 17 2 Delay -

J 1 5 4 6 7 3 15 6 Continue 28
26 L 3 2 2 8 3 4 17 2 Start 28
M 2 2 2 2 4 8 17 4 Delay -

M 2 2 2 2 4 8 17 4 Start 32
28 N 1 4 4 3 4 1 19 2 Start 30
O 5 5 4 6 2 3 19 3 Delay -

M 2 2 2 2 4 8 17 4 Continue 32
30
O 5 5 4 6 2 3 19 3 Start 33

O 5 5 4 6 2 3 19 3 Continue 33
32
P 3 2 3 4 7 8 21 5 Delay -

P 3 2 3 4 7 8 21 5 Start 38
33 Q 4 5 4 2 3 4 22 8 Start 41
R 5 3 3 3 2 8 24 2 Delay -
Q 4 5 4 2 3 4 22 8 Continue 41
38
R 5 3 3 3 2 8 24 2 Delay -
R 5 3 3 3 7 8 24 2 Start 43
41
T 1 6 2 7 5 2 30 2 Start 43
43 S 2 4 6 2 3 4 26 6 Start 49

Construction Management 158 Dr. Emad Elbeltagi


The process, therefore, was continued at day 9 which is the finish time of activity B (C
was scheduled to finish at day 10). The third cycle at day 9, as such, included 4 eligible
activities: activity C (continuing till day 10); activity D (delayed from previous cycle);
and two more activities (E and H, which immediately follow B).

Example 7.3

The activities of a project along with their durations, predecessors and resource used are
given in Table 7.6. If resource 1 is limited to 8 units and resource is limited to one unit,
determine the activities schedule start and finish times so that the weekly resource usage
does not exceed the resource limits.

Table 7.6: Data of Example 7.3

Duration Resource (units/week)


Activity Predecessors
(Weeks) R1≤8 R2 ≤1
A 4 - 3 0
B 6 - 6 1
C 2 - 4 0
D 8 A 0 1
E 4 D 4 1
F 10 B 0 1
G 16 B 4 0
H 8 F 2 0
I 6 E, H 4 1
J 6 C 5 1
K 10 G, J 2 0

Solution

The project network is drawn and the activities timings are calculated giving a project
completion time of 32 weeks without considering the resource limits.

The solution will be arranged in the Table below (Table 7.7).

Construction Management 159 Dr. Emad Elbeltagi


Table 7.7: Solution of example 7.3
Resources
Current Eligible Finish
Duration ELS Decision
time activities R1 ≤8 R2 ≤1 time

B 6 1 6 0 Start 6
0 A 3 0 4 10 Delay -
C 4 0 2 14 Delay -

G 4 0 16 6 Start 22
F 0 1 10 8 Start 16
6
A 3 0 4 10 Start 10
C 4 0 2 14 Delay -

G 4 0 16 - Continue 22
F 0 1 10 - Continue 16
10
C 4 0 2 14 Start 12
D 0 1 8 14 Delay -

G 4 0 16 - Continue 22
F 0 1 10 - Continue 16
12
D 0 1 8 14 Delay -
J 5 0 6 16 Delay -

G 4 0 16 - Continue 22
D 0 1 8 14 Start 24
16
J 5 1 6 16 Delay -
H 2 0 8 18 Start 24

D 0 1 8 - Continue 24
22 H 2 0 8 - Continue 24
J 5 1 6 16 Delay -
J 5 1 6 14 Start 30
24
E 4 1 4 22 Delay -
E 4 1 4 22 Start 34
30
K 2 0 10 22 Start 40

K 2 0 10 - Continue 40
34
I 2 0 6 26 Start 40

Construction Management 160 Dr. Emad Elbeltagi


0 4 4 12 12 16
A (4) D (8) E (4)
10 14 14 22 22 26

6 16 16 24 24 30
F (10) H (8) I (6)
8 18 18 26 26 32

0 0 0 6 6 22 22 32 32 32
Start (0) B (6) G (16) K (10) End (0)
0 0 0 6 6 22 22 32 32 32

0 2 2 8
C (2) J (6)
14 16 16 22

Figure 7.17: Precedence diagram of Example 7.3

Then, the project completion time is 40 weeks with activities timing as given below:

Schedule Schedule Schedule Schedule


Activity Activity
start finish start finish
A 6 10 G 6 22
B 0 6 H 16 24
C 10 12 I 34 40
D 16 24 J 24 30
E 30 34 K 30 40
F 6 16

7.8 Exercises

1. Estimates and schedules when initially prepared assume unlimited resources:


a. True b. False

2. Float is a useful tool for project management to use in order to level resources:
a. True b. False

Construction Management 161 Dr. Emad Elbeltagi


3. Which would not be considered a scheduling resource:
a. People b. Money
c. Equipment d. Laydown space

4. A Precedence network for a project has the logic given in the following table. The
resource rates for each activity are also given. It is required to:

a- Level resource I and then draw resource histogram before and after leveling
on the same diagram.
b- Level resource II and then draw resource histogram before and after leveling
on the same diagram.
c- Level resources I and II simultaneously, and plot the histograms for both
resources. Compare the results with those obtained form (a) and (b).

Resource Rate
Activity Predecessors
Duration Resource I Resource II

A --- 2 3 1
B A 8 4 6
C A 6 8 6
D A 4 6 5
E A 3 4 1
F B 12 2 0
G C, D 4 7 9
H C, D, E 6 9 5
K F, G, H 3 2 0

5. The following project consisting of 10 activities is considered for multi-resource


scheduling:
a) Develop a resource schedule assuming that the availability of resources is
limited as below.
Resource A: 7
Resource B: 7
Resource C: 6

Construction Management 162 Dr. Emad Elbeltagi


b) If the project completion time is limited to 14 weeks. Estimate the minimum
resource level required for each resource type. Also, draw the resource
histograms before and after leveling.

Resources required
Activity (i – j) Description Duration (days)
A B C
1-2 A 3 4 4 2
1-3 B 4 3 4 1
1-5 C 5 1 3 2
2-4 D 2 1 0 0
2-6 E 3 2 1 0
3-4 F 4 2 2 1
4-7 G 3 3 1 2
5-6 H 6 4 4 4
5-7 I 4 3 2 1
6-7 J 3 1 4 5

6. Consider the following project.

Resources required
Activity Predecessor Duration (days)
Plumbers Labors
A - 4 2 3
B - 3 1 -
C - 6 1 3
D B 8 3 4
E B 7 - 1
F C 2 3 5
G A, D 9 1 2
H E 5 2 4
I E 4 - 2
J F, I 4 2 3

Assume eleven plumbers and nine labors are available for the project. Both
resources must work at the same time when assigned to the same activity. Prepare
an activity schedule which satisfies the resources constraints.

Construction Management 163 Dr. Emad Elbeltagi


CHAPTER 8

PROJECT TIME-COST TRADE-OFF

In the previous chapters, duration of activities discussed as either fixed or random


numbers with known characteristics. However, activity durations can often vary
depending upon the type and amount of resources that are applied. Assigning more
workers to a particular activity will normally result in a shorter duration. Greater speed
may result in higher costs and lower quality, however. In this section, we shall consider
the impacts of time and cost trade-offs in activities.

Reducing both construction projects’ cost and time is critical in today’s market-driven
economy. This relationship between construction projects’ time and cost is called time-
cost trade-off decisions, which has been investigated extensively in the construction
management literature. Time-cost trade-off decisions are complex and require selection
of appropriate construction method for each project task. Time-cost trade-off, in fact, is
an important management tool fo overcoming one of the critical path method limitations
of being unable to bring the project schedule to a specified duration.

8.1 Time-Cost Trade-Off

The objective of the time-cost trade-off analysis is to reduce the original project duration,
determined form the critical path analysis, to meet a specific deadline, with the least cost.
In addition to that it might be necessary to finish the project in a specific time to:

- Finish the project in a predefined deadline date.


- Recover early delays.
- Avoid liquidated damages.
- Free key resources early for other projects.

Construction Management 164 Dr. Emad Elbeltagi


- Avoid adverse weather conditions that might affect productivity.
- Receive an early completion-bonus.
- Improve project cash flow

Reducing project duration can be done by adjusting overlaps between activities or by


reducing activities’ duration. What is the reason for an increase in direct cost as the
activity duration is reduced? A simple case arises in the use of overtime work. By
scheduling weekend or evening work, the completion time for an activity as measured in
calendar days will be reduced. However, extra wages must be paid for such overtime
work, so the cost will increase. Also, overtime work is more prone to accidents and
quality problems that must be corrected, so costs may increase. The activity duration can
be reduced by one of the following actions:

- Applying multiple-shifts work.


- Working extended hours (over time).
- Offering incentive payments to increase the productivity.
- Working on week ends and holidays.
- Using additional resources.
- Using materials with faster installation methods.
- Using alternate construction methods or sequence.

8.2 Activity Time-Cost Relationship

In general, there is a trade-off between the time and the direct cost to complete an
activity; the less expensive the resources, the larger duration they take to complete an
activity. Shortening the duration on an activity will normally increase its direct cost
which comprises: the cost of labor, equipment, and material. It should never be assumed
that the quantity of resources deployed and the task duration are inversely related. Thus
one should never automatically assume that the work that can be done by one man in 16
weeks can actually be done by 16 men in one week.

Construction Management 165 Dr. Emad Elbeltagi


A simple representation of the possible relationship between the duration of an activity
and its direct costs appears in Figure 8.1. Considering only this activity in isolation and
without reference to the project completion deadline, a manager would choose a duration
which implies minimum direct cost, called the normal duration. At the other extreme, a
manager might choose to complete the activity in the minimum possible time, called
crashed duration, but at a maximum cost.

Cost Crash duration


&
Crash cost

Normal duration
&
Normal cost

Time

Figure 8.1: Illustration of linear time/cost trade-off for an activity

The linear relationship shown in the Figure 8.1 between these two points implies that any
intermediate duration could also be chosen. It is possible that some intermediate point
may represent the ideal or optimal trade-off between time and cost for this activity. The
slope of the line connecting the normal point (lower point) and the crash point (upper
point) is called the cost slope of the activity. The slope of this line can be calculated
mathematically by knowing the coordinates of the normal and crash points.

Cost slope = crash cost – normal cost / normal duration – crash duration

As shown in Figures 8.1, 8.2, and 8.3, the least direct cost required to complete an
activity is called the normal cost (minimum cost), and the corresponding duration is
called the normal duration. The shortest possible duration required for completing the
activity is called the crash duration, and the corresponding cost is called the crash cost.
Normally, a planner start his/her estimation and scheduling process by assuming the least
costly option

Construction Management 166 Dr. Emad Elbeltagi


Cost Crash duration
&
Crash cost

Normal duration
&
Normal cost

Time

Figure 8.2: Illustration of non-linear time/cost trade-off for an activity

Cost

Crash duration
&
Crash cost

Normal duration
&
Normal cost

Time

Figure 8.3: Illustration of discrete time/cost trade-off for an activity

Example 8.1

A subcontractor has the task of erecting 8400 square meter of metal scaffolds. The
contractor can use several crews with various costs. It is expected that the production will
vary with the crew size as given below:

Construction Management 167 Dr. Emad Elbeltagi


Crew
Estimated daily production
size Crew formation
(square meter)
(men)
166 5 1 scaffold set, 2 labors, 2 carpenter, 1 foreman
204 6 2 scaffold set, 3 labors, 2 carpenter, 1 foreman
230 7 2 scaffold set, 3 labors, 3 carpenter, 1 foreman

Consider the following rates: Labor LE96/day; carpenter LE128/day; foreman LE144/day
and scaffolding LE60/day. Determine the direct cost of this activity considering different
crews formation.

Solution

The duration for installing the metal scaffold can be determined by dividing the total
quantity by the estimated daily production. The cost can be determined by summing up
the daily cost of each crew and then multiply it by the duration of using that crew. The
calculations are shown in the following table.

Crew size Duration (days) Cost (LE)


5 50.6 (use 51) 51 x (1x60 + 2x96 + 2x128 + 1x144) = 33252
6 41.2 (use 42) 42 x (2x60 + 3x96 + 2x128 + 1x144) = 33936
7 36.5 (use 37) 37 x (2x60 + 3x96 + 3x128 + 1x144) = 34632

This example illustrates the options which the planner develops as he/she establishes the
normal duration for an activity by choosing the least cost alternative. The time-cost
relationship for this example is shown in Figure 8.4. The cost slop for this activity can be
calculates as follow:

Cost slope 1 (between points 1 and 2) = (33936 – 33252) / (51 – 42) = 76.22 LE/day
Cost slope 2 (between points 2 and 3) = (34632 – 33936) / (42 – 37) = 139.2 LE/day

Construction Management 168 Dr. Emad Elbeltagi


34800
34600 3
34400
34200
Cost (LE)

34000
2
33800
33600
33400
1
33200
33000
30 35 40 45 50 55

Duration (days)

Figure 8.4: Time-cost relationship of Example 8.1

8.3 Project Time-Cost Relationship

Total project costs include both direct costs and indirect costs of performing the activities
of the project. Direct costs for the project include the costs of materials, labor, equipment,
and subcontractors. Indirect costs, on the other hand, are the necessary costs of doing
work which can not be related to a particular activity, and in some cases can not be
related to a specific project.

If each activity was scheduled for the duration that resulted in the minimum direct cost in
this way, the time to complete the entire project might be too long and substantial
penalties associated with the late project completion might be incurred. Thus, planners
perform what is called time-cost trade-off analysis to shorten the project duration. This
can be done by selecting some activities on the critical path to shorten their duration.

As the direct cost for the project equals the sum of the direct costs of its activities, then
the project direct cost will increase by decreasing its duration. On the other hand, the
indirect cost will decrease by decreasing the project duration, as the indirect cost are
almost a linear function with the project duration. Figure 8.5 illustrates the direct and
indirect cost relationships with the project duration.

Construction Management 169 Dr. Emad Elbeltagi


Project cost

Project duration

Figure 8.5: Project time-cost relationship

The project total time-cost relationship can be determined by adding up the direct cost
and indirect cost values together as shown in Figure 8.5. The optimum project duration
can be determined as the project duration that results in the least project total cost.

8.4 Shortening Project Duration

The minimum time to complete a project is called the project-crash time. This minimum
completion time can be found by applying critical path scheduling with all activity
durations set to their minimum values. This minimum completion time for the project can
then be used to determine the project-crash cost. Since there are some activities not on the
critical path that can be assigned longer duration without delaying the project, it is
advantageous to change the all-crash schedule and thereby reduce costs.

Heuristic approaches are used to solve the time/cost tradeoff problem such as the cost-
lope method used in this chapter. In particular, a simple approach is to first apply critical
path scheduling with all activity durations assumed to be at minimum cost. Next, the
planner can examine activities on the critical path and reduce the scheduled duration of
activities which have the lowest resulting increase in costs. In essence, the planner
develops a list of activities on the critical path ranked with their cost slopes. The heuristic
solution proceeds by shortening activities in the order of their lowest cost slopes. As the

Construction Management 170 Dr. Emad Elbeltagi


duration of activities on the shortest path are shortened, the project duration is also
reduced. Eventually, another path becomes critical, and a new list of activities on the
critical path must be prepared. Using this way, good but not necessarily optimal
schedules can be identified.

The procedure for shortening project duration can be summarized in the following steps:
1. Draw the project network.

2. Perform CPM calculations and identify the critical path, use normal durations and
costs for all activities.

3. Compute the cost slope for each activity from the following equation:

cost slope = crash cost – normal cost / normal duration – crash duration

4. Start by shortening the activity duration on the critical path which has the least cost
slope and not been shortened to its crash duration.

5. Reduce the duration of the critical activities with least cost slope until its crash
duration is reached or until the critical path changes.

6. When multiple critical paths are involved, the activity(ies) to shorten is determined
by comparing the cost slope of the activity which lies on all critical paths (if any),
with the sum of cost slope for a group of activities, each one of them lies on one of
the critical paths.

7. Having shortened a critical path, you should adjust activities timings, and floats.

8. The cost increase due to activity shortening is calculated as the cost slope
multiplied by the time of time units shortened.

9. Continue until no further shortening is possible, and then the crash point is reached.

10. The results may be represented graphically by plotting project completion time
against cumulative cost increase. This is the project direct-cost / time relationship.
By adding the project indirect cost to this curve to obtain the project time / cost
curve. This curve gives the optimum duration and the corresponding minimum
cost.

Construction Management 171 Dr. Emad Elbeltagi


Example 8.2

Assume the following project data given in Table 8.1. It is required to crash the project
duration from its original duration to a final duration of 110 days. Assume daily indirect
cost of LE 100.

Table 8.1: Data for Example 8.2


Normal Crash
Activity Preceded by
Duration (day) Cost (LE) Duration (day) Cost (LE)

A - 120 12000 100 14000


B - 20 1800 15 2800
C B 40 16000 30 22000
D C 30 1400 20 2000
E D, F 50 3600 40 4800
F B 60 13500 45 18000

Solution

The cost slope of each activity is calculated. Both the crashability and the cost slope are
shown beneath each activity in the precedence diagram. The critical path is B-C-D-E and
the project duration in 140 days. Project total normal direct cost = sum of normal direct
costs of all activities = LE 48300.

0 120 140 140


A (120) End (0)
20 140 140 140
0 0 20@100 20 60 60 90 90 140
Start (0) C (40) D (30) E (50)
0 0 20 60 60 90 90 140
10@600 10@60 10@120
0 20 20 80
B (20) F (60)
0 20 30 90
5@200 15@300

1. The activity on the critical path with the lowest cost slope is of activity “D”, this
activity can be crashed by 10 days. Then adjust timing of the activities.

Construction Management 172 Dr. Emad Elbeltagi


0 120 130 130
A (120) End (0)
10 130 130 130
0 0 20@100 20 60 60 80 80 130
Start (0) C (40) D (20) E (50)
0 0 20 60 60 80 80 130
10@600 10@120
0 20 20 80
B (20) F (60)
0 20 20 80
5@200 15@300

A new critical path will be formed, B-F-E.


New Project duration is 130 days.
The project direct cost is increased by 10 x 60 = LE 600.
Project direct cost = 48300 + 600 = LE 48900

2. At this step activity “E” will be crashed, as this activity lies on both critical paths.
Activity “E” will be shortened by 10 days.

0 120 120 120


A (120) End (0)
0 120 120 120
0 0 20@100 20 60 60 80 80 120
Start (0) C (40) D (20) E (40)
0 0 20 60 60 80 80 120
10@600
0 20 20 80
B (20) F (60)
0 20 20 80
5@200 15@300

Accordingly, all activities will b turn to critical activities.


New Project duration is 120 days.
The project direct cost is increased by 10 x 120 = LE 1200.
Project direct cost = 48900 + 1200 = LE 50100

Construction Management 173 Dr. Emad Elbeltagi


3. In this step, it is difficult to decrease one activity’s duration and achieve decreasing in
the project duration. So, either to crash an activity on all critical paths (if any),
otherwise, choose several activities on different critical paths. As shown, activities
“A” and “B” can be crashed together which have the least cost slope (100 + 200).
Then, crash activities “A” and “B” by 5 days.

0 115 115 115


A (115) End (0)
0 115 115 115
0 0 15@100 15 55 55 75 75 115
Start (0) C (40) D (20) E (40)
0 0 15 55 55 75 75 115
10@600
0 15 15 75
B (15) F (60)
0 15 15 75
15@300

New Project duration is 115 days.


The project direct cost is increased by 5 x (100 + 200) = LE 1500.
Project direct cost = 50100 + 1500 = LE 51600

4. In this final step, it is required to decrease the duration of an activity from each path.
The duration of activity ”A” will be crashed to 110 days, “C” to 35 days, and “F” to
55 days. Thus, achieving decreasing project duration to 110 days. Also, increase in
the project direct cost by 5 x (100 + 600 + 300) = LE 5000

0 110 110 110


A (110) End (0)
0 110 110 110
0 0 10@100 15 50 50 70 70 110
Start (0) C (35) D (20) E (40)
0 0 15 50 50 70 70 110
5@600
0 15 15 70
B (15) F (55)
0 15 15 70
10@300
Construction Management 174 Dr. Emad Elbeltagi
Duration (days) Direct cost (LE) Indirect cost (LE) Total cost (LE)

140 48300 14000 62300


130 48900 13000 61900
120 50100 12000 62100
115 51600 11500 63100
110 56600 11000 67600

80000

70000

60000

50000
Cost (LE)

40000

30000

20000

10000

0
100 110 120 130 140 150

Project duration (days)


Example 8.3

The durations and direct costs for each activity in the network of a small construction
contract under both normal and crash conditions are given in the following table.
Establish the least cost for expediting the contract. Determine the optimum duration of
the contract assuming the indirect cost is LE 125/day.

Solution

The cost slope of each activity is calculated. Both the crashability and the cost slope are
shown beneath each activity in the precedence diagram. The critical path is A-C-G-I and
the contract duration in 59 days.

Construction Management 175 Dr. Emad Elbeltagi


Table 8.2: Data for Example 8.3
Normal Crash
Activity Preceded by
Duration (day) Cost (LE) Duration (day) Cost (LE)

A - 12 7000 10 7200
B A 8 5000 6 5300
C A 15 4000 12 4600
D B 23 5000 23 5000
E B 5 1000 4 1050
F C 5 3000 4 3300
G E, C 20 6000 15 6300
H F 13 2500 11 2580
I D, G, H 12 3000 10 3150

Solution

The cost slope of each activity is calculated. Both the crashability and the cost slope are
shown beneath each activity in the precedence diagram. The critical path is A-C-G-I and
the contract duration in 59 days.

20 43
D (23)
24 47

0 12 12 20 20 25 27 47 47 59
A (12) B (8) E (5) G (20) I (12)
0 12 14 22 22 27 27 47 47 59
2@100 2@150 1@50 5@60 2@75
12 27 27 32 32 45
C (15) F (5) H (13)
12 27 29 34 34 47
3@200 1@300 2@40

1. The activity on the critical path with the lowest cost slope is “G”, this activity can
be crashed by 5 days, but if it is crashed by more than 2 days another critical path
will be generated. Therefore, activity “G” will be crashed by 2 days only. Then
adjust timing of the activities.

Construction Management 176 Dr. Emad Elbeltagi


20 43
D (23)
22 45

0 12 12 20 20 25 27 45 45 57
A (12) B (8) E (5) G (18) I (12)
0 12 14 22 22 27 27 45 45 57
2@100 2@150 1@50 3@60 2@75
12 27 27 32 32 45
C (15) F (5) H (13)
12 27 27 32 32 45
3@200 1@300 2@40

A new critical path will be formed, A-C-F-H-I.


New contract duration is 57 days and the cost increase is 2 x 60 = LE 120.

2. At this step the activities that can be crashed are listed below:
Either “A” at cost LE 100/day
Or “C” at cost LE 200/day
Or “I” at cost LE 75/day
Or “F & G” at cost LE 360/day
Or “H & G” at cost LE 100/ day
Activity “I” is chosen because it has the least cost slope, and it can be crashed by 2
days. Because it is last activity in the network, it has no effect on other activities.

20 43
D (23)
22 45

0 12 12 20 20 25 27 45 45 55
A (12) B (8) E (5) G (18) I (10)
0 12 14 22 22 27 27 45 45 55
2@100 2@150 1@50 3@60

12 27 27 32 32 45
C (15) F (5) H (13)
12 27 27 32 32 45
3@200 1@300 2@40

New contract duration is 55 days and the cost increase is 2 x 75 = LE 150.


Cumulative cost increase = 120 + 150 = LE 270

Construction Management 177 Dr. Emad Elbeltagi


3. Now, we could select “A” or both “H & G”, because they have the same cost
slope. Activity “A” is chosen to be crashed. This will change the timings for all
activities, but no new critical path will be formed.

18 41
D (23)
20 43

0 10 10 18 18 23 25 43 43 53
A (10) B (8) E (5) G (18) I (10)
0 10 12 20 20 25 25 43 43 53
2@150 1@50 3@60

10 25 25 30 30 43
C (15) F (5) H (13)
10 25 25 30 30 43
3@200 1@300 2@40

New contract duration is 53 days and the cost increase is 2 x 100 = LE 200.
Accordingly, cumulative cost increase = 270 + 200 = LE 470

4. Now, activities “H & G” can be crashed by 2 days each. “A” new critical path A-
B-D-I will be formed.

18 41
D (23)
18 41

0 10 10 18 18 23 25 41 41 51
A (10) B (8) E (5) G (16) I (10)
0 10 10 18 20 25 25 41 41 51
2@150 1@50 1@60

10 25 25 30 30 41
C (15) F (5) H (11)
10 25 25 30 30 41
3@200 1@300

New contract duration is 51 day and the cost increase is 2 x 100 = LE 200.
Accordingly, cumulative cost increase = 470 + 200 = LE 670

Construction Management 178 Dr. Emad Elbeltagi


5. At this stage, the network has three critical paths. The activities that can be
crashed are listed below:
Either C & B at cost LE 350/day or F, G & B at cost LE 510/day
Activities C & B are chosen because they have the least cost slope.
16 39
D (23)
16 39

0 10 10 16 16 21 23 39 39 49
A (10) B (6) E (5) G (16) I (10)
0 10 10 16 18 23 23 39 39 49
1@50 1@60

10 23 23 28 28 39
C (13) F (5) H (11)
10 3 23 28 28 39
1@200 1@300

New contract duration is 49 days and the cost increase is 2 x 350 = LE 700.
Cumulative cost increase = 670 + 700 = LE 1370
Now, there is no further shortening is possible.
The contract duration and the corresponding cost are given in the table below.

Duration Direct cost X 1000 LE Indirect cost x 1000 LE Total cost x 1000 LE
59 36.50 7.375 43.875
57 36.62 7.125 43.745
55 36.77 6.875 43.645
53 36.97 6.625 43.595
51 37.17 6.375 43.545
49 37.87 6.125 43.995

50
Total cost
40 Direct cost
LE x 1000

30

20

10 Indirect cost

0
48 50 52 54 56 58 60
Time (days)

Construction Management 179 Dr. Emad Elbeltagi


8.5 Exercises

1. The following table gives the activities involved in a pipeline contract. The
duration and cost data are also given. The indirect cost for the contract is
LE120/day. Calculate the minimum cost of the work corresponding to contract
duration of 102 days.

Normal Crashability Cost


Act. Description Predes. Slope
Time Cost (days) (LE)
A Preparation --- 10 200 - -
B Move on to site A 20 200 - -
C Obtain pipes A 40 1800 - -
D Obtain valves A 28 500 8 10
E Locate pipeline B 8 150 - -
F Cut specials C 10 100 4 40
G Excavate trench E 30 3000 20 180
H Prepare valve chambers C, G 20 2800 12 50
I Layout joint pipes C, G 24 1000 10 65
J Fit valves D, F, H 10 200 4 80
K Concrete anchors I 8 520 1 80
L Backfill J, K 6 420 1 60
M Finish valve chambers J, K 6 200 3 40
N Test pipeline J, K 6 150 2 70
O Clean up site L, N 4 300 - -
P Leave site M, O 2 180 - -

2. Cost and schedule data for a small project are given below. Assume an indirect
cost of LE 200/day. Develop the time-cost curve for the project and determine the
minimum contract duration

Construction Management 180 Dr. Emad Elbeltagi


Cost (LE) Duration (days)
Activity Preceded by
Crash Normal Crash Normal

A - 3900 3600 6 7
B A 6500 5500 3 5
C B 7200 6350 7 9
D B 4900 4700 18 19
E B 2200 2050 9 10
F C 1700 1200 6 8
G F 7200 7200 5 5
H E 10000 9450 10 11
I D, G, H 4700 4500 6 7

3. Draw the precedence diagram for the following data.

Duration (days)
Cost slope
Activity Followed by
Minimum LE / day
Normal

A B, E, F 7 5 200
B K 9 5 450
C H, D 8 7 400
D I, N 11 4 100
E G, M 9 6 400
F L 8 7 500
G C 7 5 200
H I, N 6 2 200
I - 12 9 200
J E, F 10 8 600
K G 14 10 350
L M 18 16 700
M C 9 8 550
N - 12 9 200

It is required to compress the schedule to a 65-day. How much more would the
project cost?

Construction Management 181 Dr. Emad Elbeltagi


CHAPTER 9

PROJECT FINANCE AND CONTRACT PRICING

In the previous chapters, techniques for project planning, scheduling, resources


management, and time-cost trade off have been introduced. This chapter will deal with
project cash flow to predict the actual flow of money during the contract duration. Also,
this chapter will introduce the means for finalizing a contract price. A project's cash flow
is basically the difference between the project's income and its expense. The difference
between a company's total income and its total expense over a period of time is the
company cash flow.

9.1 Contract Cash Flow

At the project level, a project’s cash flow is the difference between the project’s expense
and income. At the construction company level, the difference between company’s total
expense and its total income over a period of time is the company’s cash flow.

Cash flow = Cash in – Cash out = Income - Expense

Forecasting cash flow is necessary for a construction company for the following
reasons:
- To ensure that sufficient cash is available to meet the demands.
- It shows the contractor the maximum amount of cash required and when it will be
required. Thus, the contractor can made arrangements to secure the required cash.
- It provides a reliable indicator to lending institutions that loans made can be repaid
according to an agreed program.
- It ensures that cash resources are fully utilized to the benefit of the owner and
investors in the company.

Construction Management 182 Dr. Emad Elbeltagi


The three main ingredients in determination of cash flow are:
- Expenses (cash out) which represents the aggregate of the payments which the
contractor will make over a period of time for all resources used in the project such
as labor, equipment, material, and subcontractors.
- Income (cash in) that represents the receipts a contractor will receive over a period
of time for the work he/she has completed.
- Timing of payments: in cash flow analysis, we are interested in the timing of
payments related to the work done by the contractor.

9.1.1 Construction Project Costs

In preparing the cash flow for a project, it is necessary to compute the costs that must be
expended in executing the works using activities durations and their direct and indirect
costs. The principal components of a contractor's costs and expenses result from the use
of labors, materials, equipment, and subcontractors. Additional general overhead cost
components include taxes, premiums on bonds and insurance, and interest on loans. The
sum of a project's direct costs and its allocated indirect costs is termed the project cost.

The costs that spent on a specific activity or project can be classified as;
- Fixed cost: costs that spent once at specific point of time (e.g., the cost of
purchasing equipment, etc.)
- Time-related cost: costs spent along the activity duration (e.g., labor wages,
equipment rental costs, etc.)
- Quantity-proportional cost: costs changes with the quantities (e.g., material cost)

Project direct costs

The costs and expenses that are incurred for a specific activity are termed direct costs.
These costs are estimates based on detailed analysis of contract activities, the site
conditions, resources productivity data, and the method of construction being used for
each activity. A breakdown of direct costs includes labor costs, material costs, equipment
costs, and subcontractor costs. Activities’ direct costs are estimated as presented
previously in chapter 3.

Construction Management 183 Dr. Emad Elbeltagi


Project indirect costs

Other costs such as the overhead costs are termed indirect costs. Part of the company’s
indirect costs is allocated to each of the company's projects. The indirect costs always
classified to: project (site) overhead; and General (head-office) overhead.

Project overhead
Project overhead are site-related costs and includes the cost of items that can not be
directly charged to a specific work element and it can be a fixed or time-related
costs. These include the costs of site utilities, supervisors, housing and feeding of
project staff, parking facilities, offices, workshops, stores, and first aid facility. Also,
it includes plants required to support working crews in different activities.

A detailed analysis of the particular elements of site-related costs is required to


arrive at an accurate estimate of these costs. However, companies used to develop
their own forms and checklists for estimating these costs. Sit overhead costs are
estimated to be between 5% - 15% of project total direct cost.

General overhead
The costs that cannot be directly attributed a specific project called general
overhead. These are the costs that used to support the overall company activities.
They represent the cost of the head-office expenses, mangers, directors, design
engineers, schedulers, etc. Continuous observations of the company expenses will
give a good idea of estimating reasonable values for the general overhead expenses.
Generally, the general overhead for a specific contract can be estimated to be
between 2% - 5% of the contract direct cost. The amount of the general overhead
that should be allocated to a specific project equals:

Project direct cost x general overhead of the company in a year


Expected sum of direct costs of all projects during the year

Having identified the direct costs, indirect costs, then the project total cost equals the sum
of both direct and indirect costs.

Construction Management 184 Dr. Emad Elbeltagi


When studying cash flow, it is very important to determine the actual dates when the
expenditures (cost) will take place. At that time, the expenditures will renamed as the
expenses. Figure 9.1 illustrate the difference between the costs and the expenses. As
shown in the figure, they are the same except the expenses are shifted (delayed) tan the
costs.

LE x 1000
700
600
500 Cost

400
300 Expense

200
100
0 Time
0 2 4 6 8 10

Figure 9.1: Project cost and expense curves

Example 9.1

Consider the construction of 8-week foundation activity with operation cost of LE8800.
The operation cost is broken down into the following elements:
- Labor LE1600 paid weekly
- Plant LE4000 paid weekly after 4 weeks credit facility
- Materials LE800 paid weekly after 5 weeks credit facility
- Subcontractors LE2400 paid weekly after 3 weeks credit facility
Determine the expenses (cash out) of this activity.

Solution

A time-scaled plan is developed for this activity for the payments for labor, plant,
material, and subcontractors. The cot will be plotted weekly with the delay specified in
Example 9.1.

Construction Management 185 Dr. Emad Elbeltagi


Weeks
Operation
1 2 3 4 5 6 7 8 9 10 11 12 13
Labor - 200 200 200 200 200 200 200 200
Plant - - - - 500 500 500 500 500 500 500 500
Material - - - - - 100 100 100 100 100 100 100 100
Subcontractors - - - 300 300 300 300 300 300 300 300
Total payment
- 200 200 500 1000 1100 1100 1100 1100 900 900 600 100
(Expense)

9.1.2 The S-Curve

The curve represents the cumulative expenditures of a project direct and indirect costs
over time is called the S-curve as it take the S-shape as shown in Figure 9.2. In many
contracts, the owner requires the contractor to provide an S-curve of his estimated
progress and costs across the life of the project. This S-shaped of the curve results
because early in the project, activities are mobilizing and the expenditure curve is
relatively flat. As many other activities come on-line, the level of expenditures increases
and the curve has a steeper middle section. Toward the end of a project, activities are
winding down and expenditures flatten again (Figure 9.2). The S-Curve is one of the
most commonly techniques to control the project costs.

100
85
Cost

50

15
Time
0 5 10 15 20

Figure 9.2: A sample S-curve

Construction Management 186 Dr. Emad Elbeltagi


An S-curve for a project can be developed using the following steps:
- Constructing a simple bar chart for all the tasks of the project.
- Assigning costs to each task using task duration.
- Plotting the cumulative amounts of expenditures versus time by smoothly
connecting the projected amounts of expenditures over time.

Example 9.2

Consider the project shown in Figure 9.3. The costs of activities are assumed as shown in
Table 9.1. The indirect costs of tasks are calculated considering a daily cost of LE500. It
is required to draw the S-curve of the total cost of the project.

4 14 12 22
D(8)
2 3
A(4) E(4)
6 6 16 18 24 26 32 32
B(6) F(10) H(8) I(6)
0 0 1 4 5 6 9
C(2) G(16)
K(10)
J(6) 8
7

2 16 22 22

Figure 9.3: Project network of Example 9.2

Table 9.1: Cost data of Example 9.2


Direct Indirect Total
Activity Duration Cost Cost Cost

A 4 2,000 2,000 4,000


B 6 9,000 3,000 12,000
C 2 3,000 1,000 4,000
D 8 12,000 4,000 16,000
E 4 18,000 2,000 20,000
F 10 15,000 5,000 20,000
G 16 8,000 8,000 16,000
H 8 20,000 4,000 24,000
I 6 9,000 3,000 12,000
J 6 9,000 3,000 12,000
K 10 5,000 5,000 10,000

Construction Management 187 Dr. Emad Elbeltagi


Solution

The S-curve is calculated based on the project's bar chart and the expenditures of each
activity. As illustrated in Figure 9.3, the eleven activities of this project are scheduled
across a 32-day time span. A bar chart representation of these activities is drawn in
Figure 9.4 showing the total costs associated with each activity above each activity's bar.
The figure shows the total expenditures and the cumulative bi-daily expenditures across
the life of the project. The S-curve of the cumulative expenditures over time is plotted in
Figure 9.5.
Time (days)

12000

4000
16000
20000

16000
24000
12000
12000

10000

Cost (x LE000) 10 10 12 14 10 10 16 16 8 8 8 8 6 6 6 2
Cumulative cost 10 20 32 46 56 66 82 98 106 114 122 130 136 142 148 150
(x LE1000)

Figure 9.4: Project bar chart of Example 9.2

9.1.3 Project Income (Cash-in)

The flow of money from the owner to the contractor is in the form of progress payments.
Estimates of work completed are made by the contractors periodically (usually monthly),
and are verified by the owner's representative. Depending on the type of contract (e.g.,
lump sum, unit price, etc.), these estimates are based on evaluations of the percentage of
total contract completion or actual field measurements of quantities placed. Owners
usually retain 10% of all validated progress payment claims submitted by contractors.

Construction Management 188 Dr. Emad Elbeltagi


The accumulated retainage payments are usually paid to the contractor with the last
payment. As opposed the expenses presented in Figure 9.1 with smooth profile, the
revenue will be a stepped curve. Also, when the contractor collects his/her money it is
named project income (cash in) as shown in Figure 9.6.

160

140
Cumulative Cost (X $1000)

120

100

80

60

40

20

0
10 34 58 12
7 16
9 20 24 28 32

Time (days)

Figure 9.5: The S-Curve for the Example Project

LE x1000
800
700
600
500 Revenue
400 Income

300
200
100
0 Time
0 2 4 6 8 10

Figure 9.6: Project revenue and income curves

Construction Management 189 Dr. Emad Elbeltagi


The time period shown in Figure 8.6 represents the time intervals at which changes in
income occur. When calculating contract income it is necessary to pay attention to the
retention and/or the advanced payment to the contractor if any.

Retention
Retention is the amount of money retained by the owner from every invoice, before
a payment is made to the contractor. This is to ensure that the contractor will
continue the work and that no problems will arise after completion. This retainage
amount ranges from 5% to 10% and hold by the owner from every invoice till the
end of the contract. The whole amount will be paid to the contractor at the end of the
contract.

Advanced payment
This is amount of money paid to the contractor for mobilization purposes. Then, it is
deducted from contract progress payment. Applying this strategy improves the
contractor cash flow and prevents him/her from loading the prices at the beginning
of the contract. This strategy, however, may be used only in projects that require
expensive site preparation, temporary facilities on site, and storage of expensive
materials at the beginning of the project.

9.1.4 Calculating Contract Cash Flow

Having determined the contract expenses and income as presented in the previous
section, it is possible to calculate the contract cash flow. If we plotted the contract
expense and income curves against each other, then the cash flow is the difference
between the points of both curves. Figure 9.7 shows the cash flow of a specific contract.
The hatched area represents the difference between the contractor’s expense and income
curves, i.e., the amount that the contractor will need to finance. The larger this area, the
more money to be financed and the more interest charges are expected to cost the
contractor.

Construction Management 190 Dr. Emad Elbeltagi


Cumulative cost (LE)
Expense

Overdraft

Income

Time
1 2 3 4 5 6 7 8

Figure 9.7: Cash flow based on monthly payments

The contractor may request an advanced or mobilization payment from the owner. This
shifts the position of the income profile so that no overdraft occurs as shown in Figure
9.8.
Cumulative Cost (LE)

Expense
Income

Advanced
payment
Time
0 1 2 3 4 5 6 7 8

Figure 9.8: Effect of advanced payment on improving cash flow

In case of less number of payments (two or three payments) along the contract period,
this will lead to increase the overdraft as shown in Figure 9.9. From the previous study,
the factors that affect the project finance (cash flow) should be considered when
calculating the cash flow:

- The project bar chart (project schedule).


- Activities’ direct and indirect cost.

Construction Management 191 Dr. Emad Elbeltagi


- Contractor method of paying his/her expenses.
- Contractor’s markup (mainly the profit margin).
- Retention amount and its payback time.
- Time of payment delay by owner.
- Advanced or mobilization payment.
Cumulative Cost (LE)

Expense

Income

Time
0 1 2 3 4 5 6 7 8

Figure 9.9: Effect of receiving two payments on cash flow

The cash flow calculations are made as described in the following steps:
- Perform project schedule and determine project and activities timing.
- Draw bar chart based on early or late timings.
- Calculate the cost per time period.
- Calculate the cumulative cost.
- Adjust the cost according the method of paying it to produce the expenses.
- Calculate the cumulative revenue (revenue = cost x (1 + markup)).
- Adjust the revenue based on the retention and delay of owner payment to determine
the income.
- Calculate the cash flow (cash flow = income – expense) at the contract different
times.

Construction Management 192 Dr. Emad Elbeltagi


Example 9.3

To illustrate the steps of cash flow calculations, consider the same project presented in
Figure 8.3. The total cost of the activities is presented in Table 9.1.

In this project, the markup equals 5% and the contractor will pay his expenses
immediately. Retention is 10% and will be paid back with the last payment. The
calculations will be made every 8 days, i.e., the contractor will receive his/her payment
every 8-days (time period). Owner’s payment is delayed one period, while the contractor
will submit the first invoice after the first period. No advanced payment is given to the
contractor.

Solution

The project revenue of each activity is calculated as revenue = cost (1 + markup) as


shown in Table 9.2. The activities timing is presented in Example 8.2.

Table 9.2 Project cost and revenue


Total Revenue (LE x
Activity Duration (day) Total Cost (LE x 1000)
1000)
A 4 04.00 04.20
B 6 12.00 12.60
C 2 04.00 04.20
D 8 16.00 16.80
E 4 20.00 21.00
F 10 20.00 21.00
G 16 16.00 16.80
H 8 24.00 25.20
I 6 12.00 12.60
J 6 12.00 12.60
K 10 10.00 10.50

By summing up the activities cost and revenue, then the contract total cost equals LE
150,000 and the total revenue equals LE 157,500. By considering that both the cost and

Construction Management 193 Dr. Emad Elbeltagi


the revenue are evenly distributed over the activities durations. The calculations are
presented as shown in Figure 9.10. The calculations will be made every 8-days period.

As shown in Figure 9.10, the project duration is divided into four periods each one equals
8 days. In addition, one period is added after project completion. Simple calculations are
then performed with the top four rows showing the project expenses. The next five rows
for income, and the last row for cash flow. As shown, after summing up the costs it
became direct expenses to the contractor as there is no delay in paying them.

The expected owner payments are then added up to from the project revenue. The
retention is subtracted from the owner payment and will be paid back to the contractor
with the last payment (row 7 in Figure 9.10). Then, the revenue is delayed by one period
to form the contractor income. The calculations in the last row are the difference between
the project income and project expense. Having two values in some periods shows the
sudden change of the cash flow as the contractor receives more payments from the
owner. For example, in the second period, just before the contractor receive his/her
payment the cash flow was (0 – 98,000 = - 98,000 LE). As the contractor receives a
payment of LE 43,470, the cash flow improves and becomes -54,530 (43,470 – 98,000).

As seen from Figure 9.10, the maximum overdraft money (maximum cash) is LE 98,000
and will be needed at the 16th day of the project. Thus shows the importance of studying
the contract cash flow. Accordingly, the contractor can made his arrangements to secure
the availability of this fund on the specified time.

Figure 9.11 shows the contract expense and income curves. These curves will be needed
to calculate the contractor cost of borrowing or investment of the overdraft money (area
between expense and income). Figure 9.12 shows the contact net cash flow.

Construction Management 194 Dr. Emad Elbeltagi


Time (days)
1000/day

2000/day
000/day

2000/day
2000/day
5000/day

2000/day

1000/day

3000/day
2000/day
2000/day

1000/day

1. Cost/2 days x LE1000 10 10 12 14 10 10 16 16 8 8 8 8 6 6 6 2 - - - -


2. Cost each period x
46 52 32 20 -
LE1000
3. Cumulative cost x
46 98 130 150 150
LE1000
4. Cumulative Expense
46 98 130 150 150
x 1000
5. Revenue = row 3 x
48.3 54.6 33.6 21 -
1.05
6. Revenue - retention =
43.47 49.14 30.24 18.90 -
row 5 x 0.9
7. Retention x LE1000 - - - - 15.75
8. Cumulative revenue x
43.47 92.61 122.85 141.75 157.50
LE1000
9. Cumulative income x
- 43.47 92.61 122.85 157.50
LE1000
10. Cumulative cash
flow x LE1000 = row 9 -46 -98/-54.53 -86.53/-37.39 -57.39/-27.15 -27.15/+7.5
– row 4

Figure 9.10: Cash flow calculations of Example 9.3

9.1.5 Minimizing Contractor Negative Cash Flow

It is very essential to the contractor to minimize his/her negative cash flow because this
may hinder him/her during performing the contract due to lack of financial resources.
Among the procedures the contractor may follow to minimize negative cash flow is:

Construction Management 195 Dr. Emad Elbeltagi


160
150 Area = LE 10,000
140 x 1 period (8-days)
130
120
110
100
LE x1000

90
80
70
60
50
40
30
20
10
0
0 1 2 3 4 5 6
Time (period)
Figure 9.11: Expense and income curves for Example 9.3

0 1 2 3 4 5 6
30

10

-10
LE x 1000

-30

-50

-70

-90

-110
Time (period)

Figure 9.12: Contract net cash flow

- Loading of rates, in which the contractor increases the prices of the earlier items in
the bill of quantities. This ensures more income at the early stages of the project.
However, this technique might represent a risk to the contractor or the owner.
- Adjustment of work schedule to late start timing in order to delay payments. In this
case, the contractor should be aware that in this case in delay might happen will
affect the project completion time and may subject him/her to liquidated damages.

Construction Management 196 Dr. Emad Elbeltagi


- Reduction of delays in receiving revenues.
- Asking for advanced or mobilization payment.
- Achievement of maximum production in the field to increase the monthly
payments.
- Increasing the mark up and reducing the retention.
- Adjust the timing of delivery of large material orders to be with the submittal of the
monthly invoice.
- Delay in paying labor wages, equipment rentals, material suppliers and
subcontractors.

9.1.6 Cost of Borrowing (Return on Investment)

Cash requirements (negative cash flows) during a project result in a contractor either
having to borrow money to meet his/her obligation or using funds from the company
reserves, which my have been more profitably if employed elsewhere. Accordingly, there
should be a charge against the project for the use of these funds.

One of the methods to determine the amount of interest to be charged during a contract is
to calculate the area between the expenses and income curves. To simplify the
calculations, the area is calculated in terms of units of LE x time period (money x time).
The time may be in days, weeks, months, etc. The underneath the expense curve is
considered as negative area (negative cash), while the area above the expense curve is
considered positive area (positive cash). The total net number of area units is calculated
and multiplied by the value of the unit and the result is multiplied by the interest rate or
rate of investment.

Cost of borrowing = net area x interest rate (9.1)

Note that, the interest rate should be calculated in the same time period as the time period
of the unit areas. For example, if the units’ areas are calculated in LE.month, then the
interest arte should be in months.

Construction Management 197 Dr. Emad Elbeltagi


Example 9.4

Consider example 8.3, it is required to calculate the cost of borrowing if the interest rate
is 1% every time period (8-days).

Solution

Referring to Figure 9.11, the approximate number of unit areas between the expense and
the income curves equals 24 units. Each unit equals LE 10,000 time period. Then, the
cost of borrowing = 24 x 100000 x 0.01 = LE 2400. This value must be added to the
contract price.

Example 9.5

The expense and income curves for a specific contract are shown in Figure 9.13. During
construction, money will be borrowed from the bank as required at an interest rate of
15% per year. Income from project earns an interest of 12% per year. Calculate the net
interest to be charged to the project.
90

80

70 2
60
LE x1000

50

40

30 1
20

10

0
0 1 2 3 4 5 6 7 8 9 10
Time (month)

Figure 9.13: Cash flow diagram for Example 9.5

Construction Management 198 Dr. Emad Elbeltagi


Solution

- Each square represents LE 10000 month


- Note that the interest rate is given per year and the square area is measured in
month, then, it is required to calculate the interest per month by dividing by 12.
Negative area, area 1 (income curve below expense curve)
- No. of negative squares = 5.7
- Interest charge = 5.7 x 10000 x 0.15 / 12 = LE 712.5
Positive area, area 2 (income curve above expense curve)
- No. of positive squares = 0.6
- Interest charge = 0.6 x 10000 x o.12 / 12 = LE 60
Net interest to be charged to the project = 712.5 – 60 = LE 652.5

Example 9.6

Table 9.3 shows a contractor’s project budget and profit distribution for a newly awarded
contract. The contractor will receive monthly payment less 10% retention and will be
paid to the contractor one month later. Half the retention is released on project
completion and the other half is released six months later. To reduce administrative costs,
the owner proposed to the contractor that measurements and payments be made every
two months with a delay of one month before the contractor receives payment. It is
required:

- Prepare graphs of cumulative cash out and expenses for both monthly and bi-
monthly measurements. Assume an average payment delay of one month of the
contractor’s cost.
- Calculate the maximum amount of capital needed to execute the project with
monthly and bi-monthly measurements.
- Calculate the cost of borrowing for extra funding needed, if the measurement is
made bi-monthly. Given that the investment rate is 15% per annum.

Construction Management 199 Dr. Emad Elbeltagi


Table 9.3: Budgeted value and profit distribution of Example 9.6

Month 1 2 3 4 5 6 7 8 9 10
Value of work each month (LE
3 4 5 8 8 8 7 6 5 2
x1000)
Profit (% of value) 15 15 10 10 10 10 10 10 5 5

Solution

The calculations of the project’s cash in and cash out passed on monthly and bi-monthly
measurements are presented in Table 9.4. As shown, the time scale of Table 9.4 is 16
months. As given in the example, the project duration is 10 months, and half of the
retention will be paid after six month of project completion. The total project value is LE
56,000. Then the total retention is LE 5,600 (0.10 x 56,000).

The cumulative expense and income curves are shown in Figure 9.14.
- The maximum cash needed in case of monthly measurement is LE 6850 at month 6
and 7 immediately before payment is received as shown in row k of Table 9.4.
- The maximum cash needed in case of bi-monthly measurement is LE 14050 at
month 7 immediately before payment is received as shown in row l of Table 9.4.

The extra fund required to finance the project if measurements and payments are made
every two months is represented by the shaded area on Figure 9.14.

Construction Management 200 Dr. Emad Elbeltagi


Table 9.4: Cash in and cash out calculations of Example 9.6

a. Month 1 2 3 4 5 6 7 8 9 10 11 12 13 14 15 16
b. Monthly value of work (LE
3 4 5 8 8 8 7 6 5 2 - - - - - -
x1000)
c. Monthly value – retention = 0.9b
2.7 3.6 4.5 7.2 7.2 7.2 6.3 5.4 4.5 1.8 - - - - - -
(LE x1000)
d. Retention (LE x 1000) - - - - - - - - - - 2.8 - - - - 2.8

d. Cumulative value (LE x 1000) 2.7 6.3 10.8 18 25.2 32.4 38.7 44.1 48.6 50.4 50.4 50.4 50.4 50.4 50.4 50.4

e. Cumulative income on monthly


- 2.7 6.3 10.8 18 25.2 32.4 38.7 44.1 48.6 53.2 53.2 53.2 53.2 53.2 56
measurements (LE x1000)
f. Cumulative income on bi-
monthly measurements (LE - - 6.3 6.3 18 18 32.4 32.4 44.1 44.1 53.2 53.2 53.2 53.2 53.2 56
x1000)
g. Profit (% of value) 15% 15% 10% 10% 10% 10% 10% 10% 5% 5% - - - - - -

h. Cost = b(1-g) (LE x1000) 2.55 3.4 4.5 7.2 7.2 7.2 6.3 5.4 4.75 1.9 - - - - - -

i. Cumulative cost (LE x1000) 2.55 5.95 10.45 17.65 24.85 32.05 38.35 43.75 48.5 50.4 - - - - - -

j. Cumulative expense (LE x1000) - 2.55 5.95 10.45 17.65 24.85 32.05 38.35 43.75 48.5 50.4 50.4 50.4 50.4 50.4 50.4

k. Cash flow monthly -2.55 -32.5 -4.15 -6.85 -6.85 -6.85 -5.95 -5.05 -4.4 -1.8
0 2.8 2.8 2.8 2.8 5.6
measurements = e - j (LE x 1000) 0.15 0.35 0.35 0.35 0.35 0.35 0.35 0.35 0.1 2.8

l. Cash flow bi-monthly -5.95 -11.35 -14.05 -11.35 -6.3


0 -2.55 -4.15 -6.85 -5.95 -4.4 2.8 2.8 2.8 2.8 5.6
measurements = f - j (LE x 1000) 0.35 0.35 0.35 0.35 2.8

Construction Management 201 Dr. Emad Elbeltagi


60

55

50

45

40

35
Cash out Cash in (bi-monthly)
LE x1000

30

25

20

15 Cash in (monthly)
10

0
0 1 2 3 4 5 6 7 8 9 10 11 12 13 14 15 16 17

Time (month)

Figure 9.14: Cash out and cash in based on monthly and bi-monthly measurement
intervals
The extra financed area (shaded area on Figure 8.14) = 2.7 x 1
+ (10.8 – 6.3) x 1
+ (25.2 -18.0) x 1
+ (38.7 – 32.4) x 1
+ (48.6 – 44.1) x 1
= 2.7 + 4.5 + 7.2 + 6.3 + 4.5
= 25.2 x 1000 LE.month
Interest charge of extra funding = 25.2 x 1000 x 0.15 / 12 = LE 315.

9.2 Project Cash Flow

The project cash flow deals with the whole life of the project not the construction period
only. Thus, project cash flow studies the project finance from the feasibility studies phase
till the operation phase. In this case, the time is much longer than that of the contract. At
the early stage of a project, the project experience negative cash flow as there is no

Construction Management 202 Dr. Emad Elbeltagi


income in these stages. In the operation stage, the revenue will increase than the
expenses. Atypical project cash flow is shown in Figure 9.15. When comparing the
economics of projects, the cumulative cash flow provides indicators for such comparison
as payback period, profit, and the maximum capital. These indicators called the
profitability indicators.

Cumulative
cash flow

Payback period Profit

Project duration
Maximum
capital

Figure 9.15: Typical project cash flow

9.2.1 Project Profitability Indicators

Profit

It is the difference between total payments and total revenue without the effect of
time on the value of money. When comparing alternatives, the project with the
maximum profit is ranked the best.

Maximum capital

It is the maximum demand of money, i.e., the summation of all negative cash
(expenditures). The project with minimum capital required is ranked the best.

Payback period

It is the length of time that it takes for a capital budgeting project to recover its initial
cost, where the summation of both cash out and cash in equals zero. When
comparing alternatives, the project with the shortest payback period is ranked the
best.

Construction Management 203 Dr. Emad Elbeltagi


Example 9.7

Two projects A and B have annual net cash flows as show in Table 9.5. Assume all cash
flows occur at the year-end. Establish the ranking of the projects in order of attractiveness
to the company using:
a. Maximum capital needed b. Profit c. Payback period

Table 9.5: Net cash flow of Example 9.7


Year 1 2 3 4 5 6 7 8
Project A (LE x 1000) -10 -40 -30 20 60 20 15 30
Project B (LE x1000) -30 -80 30 50 10 20 40 40

Solution

The cumulative cash flow is first calculated as shown in Table 9.6.

Table 9.6: Cumulative cash flow of Example 9.7


Year 1 2 3 4 5 6 7 8

Project A (LE x 1000) -10 -50 -80 -60 0 20 35 65


Project B (LE x1000) -30 -110 -80 -30 -20 0 40 80

The cumulative cash flow of projects A and B are shown in Figure 9.16. From the figure
the following indicators are drawn:

100
80
65
50
Project A
0
0 2 4 6 8
-50
-80
Project B
-100
-110

-150

Figure 9.16: Cumulative net cash flow of Example 9.7

Construction Management 204 Dr. Emad Elbeltagi


- Maximum capital: project A (LE 80,000) is better than project B (LE 110,000)
because it needs less capital.
- Profit: Project B (LE 80,000) is more profitable than project A (LE 65,000).
- Payback period: Project A (5 years) is better than project B (6 years) because is has
shorter payback period.

9.3 Discounted Cash Flow

The value of money is dependent on the time at which it is received. A sum of money on
hand today is worth more than the same sum of money to be received in the future
because the money on hand today can be invested to earn interest to gain more than the
same money in the future. Thus, studying the present value of money (or the discounted
value) that will be received in the future is very important. This concept will be
demonstrated in the following subsections.

9.3.1 Present Value

Present value (PV) describes the process of determining what a cash flow to be received
in the future is worth in today's pounds. Therefore, the Present Value of a future cash
flow represents the amount of money today which, if invested at a particular interest rate,
will grow to the amount of the future cash flow at that time in the future. The process of
finding present values is called Discounting and the interest rate used to calculate present
values is called the discount rate.

To illustrate this concept, if you were to invest LE 100 today with an interest rate of 10%
compounded annually, this investment will grow to LE 110 [100 x (1 + 0.1)] in one year.
The investment earned LE 10. At the end of year two, the current balance LE 110 will be
invested and this investment will grow to LE 121 [110 x (1 + 0.1)]. Accordingly,
investing a current amount of money, P, for one year, with interest rate (r) will result in a
future amount, C using the following equation.

C = P. (1 + r) (9.2)

Construction Management 205 Dr. Emad Elbeltagi


If P is invested for n years then the future amount C will equals.

C = P. (1 + r )n (9.3)

In contrary to the Equation 9.3, the present value (the discounted value), P, of a future
some of money, C, that will be received after n years if the discount rate is r is calculated
as follow:

P = C / (1 + r )n (9.4)

For example, the present value of $100 to be received three years from now is $75.13 if
the discount rate is 10% compounded annually.

Example 9.8

Find the present value of the following cash flow stream given that the interest rate is
10%.

Solution

9.3.2 Net Present Value (NPV)

Net present value (NPV) is the summation of all PV of cash flows of the project, where
expenses are considered negative and incomes are considered positive. A project will be
considered profitable and acceptable if it gives a positive NPV. When comparing
projects, the project with the largest (positive) NPV should be selected.

Example 9.9

The Table below illustrates the net cash flow of two projects over 5 years. Using the
NPV, which project would you prefer if the discount rate 10%.

Construction Management 206 Dr. Emad Elbeltagi


Year 0 1 2 3 4 5

Project A (LE ) -1000 500 400 200 200 100


Project B (LE ) -1000 100 200 200 400 700

Solution

Project A:

Project B:

From the results of the NPV, project A should be chosen since it has the larger NPV.

9.3.3 Internal Rate of Return (IRR)

The internal rate of return (IRR) of a capital budgeting project is the discount rate (r) at
which the NPV of a project equals zero. The IRR decision rule specifies that a project
with an IRR greater than the minimum return on capital should be accepted. When
choosing among alternative projects, the project with the highest IRR should be selected
(as long as the IRR is greater than the minimum acceptable return of capital). The IRR is
assumed to be constant over the project life.

Example 9.10

Calculate the IRR for both projects presented in Example 8.9, and compare among them
using the resulted IRR. Assume the return on capital equals 10%.

Solution

Project A:

Construction Management 207 Dr. Emad Elbeltagi


Project B:

Both projects are acceptable as they produce IRR grater than the return (cost) on capital.
However, when comparing them, Project A should be chosen since it has the higher IRR.

9.4 Finalizing a Tender Price

The total price of a tender comprises the cost and the markup. The cost includes direct
and indirect costs. The markup, on the other hand, includes profit margin, financial
charges (cost of borrowing), and a risk allowance margin (Figure 9.17). Estimating
activities direct costs presented previously in chapter 3. Estimating indirect costs
presented earlier in this chapter.

Price

Markup Cost

Risk allowance Financial charge Profit Indirect cost Direct cost

Figure 9.17: Components of a tender price

If you are much involved in the construction business, you must have experienced how
difficult it is to decide on a suitable margin to make your bid competitive against other
contractors. We need to decide on the markup percentage that makes the bid low enough
to win and, at the same time, high enough to make reasonable profit. Generally,
contractors often have to main methods of assessing a specific contract markup:

- Estimating a single percentage markup to be added to the total cost. It is assumed


that this percentage will cover all the components of markup as shown in Figure
9.17; and

Construction Management 208 Dr. Emad Elbeltagi


- Detailed analysis of the risky components in the project and their impact on the
project in terms of increased time and cost. Also, cash flow analysis to estimate
the financial charge and estimating a reasonable profit margin.

Calculations of the financial charges (cost of borrowing) were, also, presented previously
in this chapter based on the cash flow analysis of the contract. Estimating profit and risk
allowance margins will be presented in the next subsection.

9.4.1 Estimating Profit Margin

Profit is the reward the contractor expecting to gain form performing a specific contract
in retune of his efforts and skills. Also, profit is the part of money that the contractor will
retain after paying every thing including the taxes, the insurance, etc. Estimating a value
for the profit margin is usually depends on the market conditions. However, the factors
that might affect choosing a profit margin values are summarized as follows:

- The contactors’ competition to win a project;


- A contractor’s desirability for work;
- Volume of work that the contractor has at a certain time;
- Size of the project and it complexity;
- Location of the project; and
- Identity of the client and the engineer.

9.4.2 Risk Management

Uncertainty and risks usually leads to project completion delays and cost overruns.
Uncertainty is the gap between the information required to estimate an outcome and the
information already possessed by the decision maker. Thus, the early assessment of the
risks and uncertainties which would affect the construction of a project may improve the
performance in terms of time and money. Risk management is a major step in project
planning; however, it is a complex process since the variables are dynamic and dependent
on variety of conditions such as: project size, project complexity, location, time of the

Construction Management 209 Dr. Emad Elbeltagi


year, etc. In order to offset the effect of risks time and/or cost contingencies should be
added to cover unforeseen occurrences.

We need risk management to minimize management by crisis; minimize surprises and


problems; increase probability of project success; and better handle on true costs and
schedules by properly estimating contingencies.

Risk management is defined as the process for systematically identifying, analyzing, and
responding to risk events throughout the life of a project to obtain the optimum or
acceptable degree of risk elimination or control. Accordingly, the major steps of risk
management are:
- Identification of risks;
- Responses to avoid, reduce, or transfer risk;
- Analysis and assessment of residual risks after the risk responses; and
- Adding time and /or cost contingency for residual risks in the project estimates.

In general, in risk allocation, the risk should be carried out by the party (client or
contractor) who is best able to make the assessment of the risk or uncertainty. If there is
any doubt, it should be carried out by the client. This is because, it is better for the client
to pay for what does happen rather than for what the contractor thought might happen in
these risks.

Risk Identification

Construction risk is defined as the possibility of undesirable extra cost or delay due to
factors having uncertain future outcome. Or it the possibility of suffering loss and the
impact that loss has on the involved party. The purpose is to identify all risks to the
project/contract and provide a preliminary assessment of their consequences. Identify
every factor that may harm the project as potential risk. For example, one may state “If
the lay-down area is not optimized then productivity will be too low;” “segmental liners
may not be available prior to construction thus delaying project”. In identifying risks, a

Construction Management 210 Dr. Emad Elbeltagi


number of approaches can be used including: standard checklists; comparison to other
projects; expert interviews; and brainstorming sessions.

Main categories of sources of risks are listed along with some examples of each category
as follow:
- Administrative: Delay in possesses of site;
Limited working hours;
Limited access to the site; and
Troubles with public services

- Logistical: Shortage or late supply of different resources;


Site remoteness problems; and
Difficulties in communications with different parties involved.

- Construction: limited work space;


Changes in soil condition than the soil report;
Construction method used;
Availability of skilled labor;
Equipment breakdown; and
Effect of varied weather and environmental conditions on
consstruction.

- Physical: Periods of high tides, temperature, etc;


Placing fill in dry season; and
Diverting water canals in time of low flow.

- Design: Design incomplete;


Design changes; and
Design errors.

Construction Management 211 Dr. Emad Elbeltagi


- Financial: Inflation which results in reducing the purchasing power of the
currency;
New restrictions applied on importing materials and equipments;
Exchange rate fluctuation;
Changes in taxes;
Availability of funds; and
Delay payments by client.

- Political: Change of local laws and regulations;


Inflation which result in reducing the purchasing power of the
currency;
Effect of wars and revolutions; and
Necessity to use local resources.

- Management: Scheduling errors;


Space congestion;
Errors in bill of quantities; and
Estimating of cost and duration based on standard figures.

- Contractual: Contract type and its suitability for undertaken work;


Co-ordination of work; and
Liability towards others.

- Disasters: Floods and storms;


Fires;
Earthquakes;
Accidents;
Diseases; and
Other acts of GOD.

Construction Management 212 Dr. Emad Elbeltagi


Response to Risk and Uncertainties

Having identified a list of possible risks and uncertainties that a project may face,
management should develop responses to avoid, reduce or transfer these risks. The
following list of actions may be taken to reduce or transfer risks:

- Using construction methods which have high degree of success;


- Using extra resources to enhance the construction program to absorb possible
delay;
- Securing alternative suppliers and advanced delivery dates for materials;
- Providing temporary roads to give flexibility of operations;
- Allowing free housing near construction site for labors to reduce problems arising
from remoteness of job site;
- Locating site facilities away of the working space to give sufficient area for
construction works;
- Assuming realistic reduced resources output;
- Maintaining good roads to provide assumed production rates for hauling
equipment;
- Using equipments for which spare parts are easily available to reduce shortage of
spare parts;
- Providing facilities for mechanical maintenance of equipment; and
- Purchasing insurance to cover the risk of site injuries.

The previously mentioned items are some examples of the actions that may be taken to
reduce or transfer the effect of risks. However, some risks will not be eliminated. To deal
with residual risks, a detailed risk analysis may be required.

Time contingency

In addition to the above-mentioned responses to risks, time contingency is one of the


contractor’s responses to risks and uncertainties. Time contingency is an extra time
that added to the contract time to offset the effect of some risks that are known to the

Construction Management 213 Dr. Emad Elbeltagi


contractor in advance such as late delivery of materials. This extra time may be
added in two ways:

- A general allowance is added to the overall contract duration when most the
activities will be affected by the risk. For example, effect of bad weather which
will affect all running activities.
- Allowance is added to a particular activity affected by the risk.

Cost contingency

Also, the contractor has to assess the risks he/she is going to retain and include
appropriate cost contingency allowance to the contract estimate. This allowance can
be added as a fixed percentage of money from the direct cost based on the contractor
experience. However, this allowance might not be appropriate for the specific risks.
Also, it results in a single figure estimate. This method can be used when there is no
means for performing risk analysis. The second method is to make a detailed
analysis of risks as presented in the next subsection.

Example 9.11

In a specific, the following risks were identified.


- Client’s delays;
- Troubles encountered with public services;
- Late supply of materials; and
- Equipment breakdown.
As a contractor, give your views on the possible responses to deal with them.

Solution

Contractor’s response to the risks:


- Client’s delay: the contractor should supply an activity schedule to warn the client
and to be an evidence for the delay.

Construction Management 214 Dr. Emad Elbeltagi


- Troubles encountered with public services: the contractor should use maps of new
tools to locate public services. Also, he may use trial pits.
- Late supply of materials: the contractor should secure advanced delivery dates and
alternative suppliers.
- Equipment breakdown: the contractor should supply the site with a complete
workshop for maintenance of equipment.

Risk Analysis

After applying the responses to risks mention in the previous section, there are still some
residual risks that need risk analysis to assess their impact on the project time and cost.
This risk analysis is the process which incorporates uncertainty in a quantitative manner,
using probability theory, to evaluate the potential impact of risk. The basic steps of risk
analysis are:

- Estimate range of risk variables;


- Choose the appropriate probability distribution which best fit risk variables;
- Define the affected activities by these risk variables; and
- Use a simulation model to evaluate the impact of risks (Monte Carlo Simulation).

This, risk analysis usually includes: sensitivity analysis; and probability analysis.

Sensitivity analysis

Sensitivity analysis is used to identify those variables which contribute most to the
risk of the contract (time and/or cost). The purpose of this analysis is to eliminate
those risk variables which have minor impact on the performance criteria and hence
reduce problem size and effort. The following procedure for risk sensitivity analysis
will be followed:

- Three values of each risk variables occurrence are to be specified: a most likely,
an optimistic, and a pessimistic;

Construction Management 215 Dr. Emad Elbeltagi


- To assess the effect of each risk variable:
• Set all other risk variables at their most likely value;

• Determine a value (A) for the performance (cost and/or time) criteria when

risk variable under consideration is set at its optimistic value;


• Determine another value (B) for the performance criteria (cost and/or time)

when risk variable under consideration is set at its pessimistic value;


• The difference between the obtained two values (A – B) of the performance

criteria is checked (subjectively).

- Two cases can be encountered:


• The difference corresponding to a risk variable has a little effect on the

performance criteria (time and/or cost). Then, this risk variable can be
eliminated from the probability analysis;
• A significant difference is found, and consequently this risk variable should

be included in the probability analysis.

Probability analysis

The purpose of the probability analysis is to determine the effect of those risk
variables which have a significant impact on the performance criteria of the project.
The following procedure for risk probability analysis will be followed

- Consider the risk variables as random variables;


- Specify the suitable probability distribution for each risk variable;
- Use a suitable simulation technique to determine the probability distribution of the
performance criteria (Monte Carlo Simulation);
- Cost contingency can be simply calculated after specifying the probability
distribution of the performance criteria (Cumulative Probability Distribution as
shown in Figure 9.18) as follows:
• Choose p; acceptable probability of project cost (time) overrun;

• Allocate p on the Cumulative Probability Distribution curve, and determine

the corresponding value (target cost); D

Construction Management 216 Dr. Emad Elbeltagi


• Calculate the cost contingency (D- C), where C is the base contract estimate

(time and cost).

Cumulative probability distribution % p

50%

Cost
Mean Cost
Base Cost = C Contingency
Target Cost = D

Figure 9.18: Cost contingency

9.5 Pricing Policy

Having all contract costs (direct and indirect), and markup components (profit margin,
risk allowance and financial charge), it is time to finalize the bid price. While, the direct
cost are associated directly to the contract activities, indirect cost and markup are not
associated with specific activities but with the whole contract. Accordingly, pricing
policy is the method by which the indirect costs and markup will be distributed among
the items of the bill of quantities, so that the bid price is ready to be submitted to the
client.

9.5.1 Balanced bid (straight forward method)

In this method the indirect cost and the markup will be distributed among different items
based on their direct cost; i.e., the more the direct cost of an item, the more its share from
indirect cost and markup. The resulting bid price is called a balance d bid.

Construction Management 217 Dr. Emad Elbeltagi


The share of specific itemDirect
= cost of this item x (total indirect cost + markup)
Total contract direct cost

Example 9.12

Assume that the direct cost for an item (a) is LE 400,000 and that item is included in a
contract whose price is LE 3,500,000 and its total direct cost is LE 2,800,000. Calculate
the price for item (a) considering a balanced bid.

Solution

Bid price = direct cost + indirect cost + markup


Indirect cost + markup (for the whole contract) = Bid price - direct cost
= 3,500,000 - 2,800,000 = LE 700,000
Then, Indirect cost + markup for activity (a)

= 400,000 x 700,000 = LE 100,000


2,800,000
Then, price of activity a = its direct cost + indirect cost
= 400,000 + 100,000 = LE 500,000

9.5.2 Unbalanced bid (Loading of Rates)

The contract price is said to be unbalanced if the contractor raises the prices on certain
bid items (usually the early items on the bill of quantities) and decreases the prices on
other items so that the tender price remain the same. This process is also called the
loading of rates. The contractor usually loads the prices of the first items to ensure more
cash at the beginning of the contract and to reduce the negative cash flow and
accordingly reduces borrowing of money.

Loading of rates may be risky to both the contractor and the owner. If the contractor
raised the price for an item and the quantity of this item increased than that was estimated
in the bill of quantities then, this situation is more risky to the owner as it will cost the

Construction Management 218 Dr. Emad Elbeltagi


owner more money. On the other hand, if the contractor reduced the price of a specific
item and the quantity of that item increased, thus situation will be more risky to the
contractor. So, it is better to follow a balanced way of distributing the indirect costs and
markup among contract items.

Example 9.13

Consider a small contract comprises of five sequential activities of equal duration. The
quantity of work in each activity, the direct cost rate, and total cost rate for balanced and
unbalanced bid are given in Table 9.6.

- Compare the cash flow curves for both balanced and unbalanced bids;
- Determine the effect of unbalanced bid on the contractors profit if:
• Quantity of activity (B) is increased by 50%.

• Quantity of activity (C) is increased by 50%.

Table 9.6: Data for Example 9.13

Balanced bid Unbalanced bid


Activity Quantity Direct
Rate Price Rate Price
cost rate

A 100 4 5 500 6 600


B 100 8 10 1000 14 1400
C 100 16 20 2000 18 1800
D 100 16 20 2000 18 1800
E 100 8 10 1000 9 900

Tender price 6500 6500

Solution

Assume each activity with one time unit duration then, the cash flow will be as given in
Table 9.7. Also, cash flow curves for both balanced and unbalanced curves are shown In
Figure 9.19. It shows that in the unbalanced bid, the contractor will receive more money
in the early stages of the contract.

Construction Management 219 Dr. Emad Elbeltagi


Table 9.7: Cash flow calculations

Time 1 2 3 4 5
Cumulative cash flow
500 1500 3500 5500 6500
(balanced bid)
Cumulative cash flow
600 2000 3800 5600 6500
(unbalanced bid)

- contract total direct cost = 100 (4 + 8 + 16 + 16 + 8) = 5200


- Contract price = 6500
- Contract markup and profit = 6500 – 5200 = 1300 = 25% of direct cost

7000

6000

5000

4000
Price

3000

2000 Balanced bid

1000

0
0 1 2 3 4 5 6
Time

Figure 9.19: Cash flow for balanced and unbalanced bids

- Table 9.8 shows the effect of tender price if the quantity of activity “B” increased
by 50%.
- The price of the unbalanced bid (7200) is grater than that of the balanced bid
(7000) which means more profit to the contractor and more risk to the owner.

In another way:
- Total direct cost = 5200 + 50 x 8 = 5600

Construction Management 220 Dr. Emad Elbeltagi


- Indirect cost & markup for balanced bid = 7000 – 5600 = 1400 = 25% of direct
cost
- Indirect cost & markup for unbalanced bid = 7200 – 5600 = 1600 = 29% of direct
cost
This increase means that the profit of the contractor has been increased and thus
represents risk to the owner.

Table 9.8: Effect of change in quantity of activity B

Balanced bid Unbalanced bid


Activity Quantity Direct
Rate Price Rate Price
cost rate
A 100 4 5 500 6 600
B 150 8 10 1500 14 2100
C 100 16 20 2000 18 1800
D 100 16 20 2000 18 1800
E 100 8 10 1000 9 900

Tender price 7000 7200

- Table 8.9 shows the effect of tender price if the quantity of activity “C” increased
by 50%.
- The price of the unbalanced bid (7400) is less than that of the balanced bid (7500)
which means less profit and more risk to the contractor.

In another way:
- Total direct cost = 5200 + 50 x 16 = 6000
- Indirect cost & markup for balanced bid = 7500 – 6000 = 1500 = 25% of direct
cost
- Indirect cost & markup for unbalanced bid = 7400 – 6000 = 1400 = 23% of direct
cost

Construction Management 221 Dr. Emad Elbeltagi


This decrease means that the profit of the contractor has been decreased and thus
represents risk to the contractor.

Table 9.9: Effect of change in quantity of activity C

Balanced bid Unbalanced bid


Activity Quantity Direct
Rate Price Rate Price
cost rate
A 100 4 5 500 6 600
B 100 8 10 1000 14 1400
C 150 16 20 3000 18 2700
D 100 16 20 2000 18 1800
E 100 8 10 1000 9 900

Tender price 7500 7400

9.5.3 Method Related Charge

The prices entered in the conventional bill of quantities might not represent the real cost
of the work defined in the individual items. This is because not all costs are directly
related to the quantity of work completed. Therefore, adjustment of the price due to a
change in quantity of a particular item may not represent the real variation in cost. This is
usually produces unnecessary amount of uncertainty and financial problems in many
contracts.

For example, site overheads are mainly time related charge. In the conventional bill of
quantities, the cost of site overheads is recovered by spreading it over the quantity
proportional rates. If variations occur and the site facilities are required for a longer
period of time, there is no systematic way to adjust the contract price.

If the time-related site overhead costs could be entered in the bills of quantities as a time-
related charge, then the cash flow pattern would be realistic and the price of this item
could be adjusted in case of any variations happened.

Construction Management 222 Dr. Emad Elbeltagi


Thus, the method related charge is proposed to enable contractors to enter any operation
whose cost is time related and not directly linked to the quantities of work being done. It
allows the contractor to define fixed and time-related charges that cove charges which are
independent of the quantity of work completed. These charges are called method-related
charges. Table 9.10 gives an example of these items.

Table 9.10: Method related chare items example


Fixed Time-related
Activity / Resource Unit Price
charge charge
Establish site Fixed - Sum
Site overheads - Time-related Month
Bulldozers - Time-related Day
Excavators - Time-related Day
General overheads - Time-related Month
----------------- - - -
----------------- - - -

The main advantages of using the method related charges are:

- Allows a system evaluation of changes.


- It realistically reflects the cost of construction which reduces the effect of inflation.
- No need for loading of rates (unbalanced bidding).
- Improves cash flow.

Figure 9.20 shows an actual bill of quantities for a project where safety and health
equipment, mobilization, and scaffolds used are listed as separate items in the bill of
quantities.

Construction Management 223 Dr. Emad Elbeltagi


SCHEDULE NO. 1 STEEL STRUCTURE, WALLS AND CONCRETE FLOOR
REMEDIATION

UNIT TOTAL
ITEM DESCRIPTION QUANTITY UNIT
PRICE PRICE

Heath Safety Equipment,


Lump Sum
Monitoring, and Implementation.

Project Site Mobilization and


1
Construction of Decontamination Lump Sum
Station.
Subtotal

2 Steel Structure Decontamination.

Square
a) Decontaminate steel structure 1100
Meters
b) Load and haul debris to
5 Ton
Landfill in Alexandria
Subtotal
Walls Decontamination and
3
Coating.

a) Support walls 1200 Square


Meters
Square
b) Rolling Scaffold 1200
Meters
c) Remove existing paint on Square
2950
interior and exterior walls Meters

d) Clean all surfaces for painting 2950 Square


Meters
e) Place a coat of smooth plaster Square
2950
paint Meters
f) Load and haul debris to
15 Ton
Landfill in Alexandria
Subtotal

Figure 9.20: Bill of quantities showing some items such as scaffold and safety equipment

Construction Management 224 Dr. Emad Elbeltagi


9.6 Exercises

1. The activities involved in the construction of a small building are given below.
The price of the work contained in each activity is listed in the table. The
contractor undertaking this project would like you to prepare graphs of
cumulative expense and income to date against time for activities starting as early
as possible. The mark-up is 10% of tender value and retention is 5%.
Measurement is made monthly with a payment delay of one month. The retention
is paid at the end of the contract. To simplify the calculations you may assume
that all costs must be met by the end of the month in which they are incurred.
What is the maximum amount of cash the contractor needs to execute this
contract and when does he require this amount?

Duration Value
No. Activity Predecessors Overlap
(months) (LE)
10 Excavation 2 - - 9000
20 Concrete bases 3 10 1 12000
30 Erect frames 1.5 20 1 18000
40 Concrete floor slab 1 20 1 15000
50 Fix cladding 1.5 30 1 6000
60 Install plant 1 40, 50 - 20000

2. A simplified project shown in the following figure. The direct costs associated
with the individual activities are shown above the bars. It is assumed that project
indirect cost will amount to 5000 LE monthly. The contractor included a profit
mark-up of 10000 LE to his bid so that the total bid price was 210000 LE. The
owner retains 10% of all validated progress payments until one half of the
contract value (i.e. 105000 LE). The progress payments will be billed at the end
of the month and the owner will transfer the billed amount minus any retains to
the contractor’s account 30 day later. Determine the expenses and income profile
of this project.

Construction Management 225 Dr. Emad Elbeltagi


Activity
A
40000 LE
B
60000 LE
C
30000 LE Time (month)
D

3. The table below lists the cumulative monthly expenses incurred by a contractor
and the corresponding monthly payments which are received from the client of a
project. Calculate the cost to the contractor of providing the working capital
necessary to finance the project if the chosen annual investment rate is 10%. If the
client makes his payments one-month later than anticipated in the table, by what
percentage will the financial charge increase?

Cumulative expense Cumulative income


End of month
(LE x 1000) (LE x 1000)
0 0 0
1 12 0
2 20 0
3 54 0
4 90 14
5 130 40
6 180 100
7 220 130
8 240 190
9 260 210
10 290 300
11 290 320
12 290 340

4. Two projects A and B have annual net cash flows as shown below. The company
discount rate is 10% per year. Assume all cash flows occur at the year-end.

Construction Management 226 Dr. Emad Elbeltagi


Establish the ranking of the projects in order of attractiveness to the company
using:
a. N.P.V. b. I.R.R. c. Payback period

Year 1 2 3 4 5
Project A (LE) -100 100 30 20 10
Project B (LE) -100 10 30 60 100

5. The following figure shows the Bar Chart for a small project, durations, schedule,
Bid Prices, and logical relationships among activities are all shown. Use the
following additional information to calculate the maximum amount of cash the
contractor needs and when dose him/her requires this amount.
ƒ Indirect cost is $ 1000 per day
ƒ Contractor markup is 5%
ƒ Time period is 8 days with interest rate of 1% per period
ƒ Retainage amount is 10%, and all withheld retainage money will be paid
back 2 periods after the last payment
ƒ Owner’s payment delay of any invoice is one period.

Construction Management 227 Dr. Emad Elbeltagi


6. Given the project below, and the minimum attractive annual rate of return of 30%,
how much would you mark up the project based on cash flows? Lag factors for all
costs incurred are zero. No office overhead is considered. Income is received one
period after expense incurred. Assume retainage equals 10%.

Mobilization Site
Month Subcontractors Equipment Materials Payroll
Demobilization overhead
0 LE 40,000 0 0 0 0 0
1 - 10,000 20,000 10,000 10,000 1,000
2 - 30,000 10,000 20,000 15,000 5,000
3 - 30,000 20,000 30,000 20,000 6,000

Construction Management 228 Dr. Emad Elbeltagi


CHAPTER 10

PROJECT CONTROL

The limited objective of project control deserves emphasis. Project control procedures are
primarily intended to identify deviations from the project plan rather than to suggest
possible areas for cost savings. This characteristic reflects the advanced stage at which
project control becomes important. The time at which major cost savings can be achieved
is during planning and design for the project. During the actual construction, changes are
likely to delay the project and lead to inordinate cost increases. As a result, the focus of
project control is on fulfilling the original design plans or indicating deviations from
these plans, rather than on searching for significant improvements and cost savings. It is
only when a rescue operation is required that major changes will normally occurring in
the construction plan.

10.1 Problems that may Arise During Construction

In construction, no project, almost, is executed as planned. Control needs to be carried-


out due to the dynamic nature of the construction process. Controlling after project finish
is trivial and updates are usually done periodically. Controlling can be done for project
schedule and/or project cost. As the construction stage of project starts, the project mostly
will face delays and/or cost overruns. The following is a list of the factors that may cause
such problems:
- Change in activity durations and quantities.
- Sudden changes of the availability of resources.
- Change orders.
- Accidents.
- Procurement delays.

Construction Management 229 Dr. Emad Elbeltagi


10.2 Schedule Updating

Construction typically involves a deadline for work completion, so contractual


agreements will force attention to schedules. More generally, delays in construction
represent additional costs due to late facility occupancy or other factors. Just as costs
incurred are compared to budgeted costs, actual activity durations may be compared to
expected durations. In this process, forecasting the time to complete particular activities
may be required.

It is important to devise efficient and cost effective methods for gathering information on
actual project accomplishments. Generally, observations of work completed are made by
inspectors and project managers and then work completed is estimated. Once estimates of
work complete and time expended on particular activities are available, deviations from
the original duration estimate can be estimated.

For example, Figure 10.1 shows the originally scheduled project progress versus the
actual progress on a project. This figure is constructed by summing up the percentage of
each activity which is complete at different points in time; this summation can be
weighted by the magnitude of effort associated with each activity. In Figure 10.1, the
project was ahead of the original schedule for a period including point A, but is now late
at point B by an amount equal to the horizontal distance between the planned progress
and the actual progress observed to date.

In evaluating schedule progress, it is important to bear in mind that some activities


possess float, whereas delays in activities on the critical path will cause project delays. In
particular, the delay in planned progress at time t may be soaked up in activities' float
(thereby causing no overall delay in the project completion) or may cause a project delay.
As a result of this ambiguity, it is preferable to update the project schedule to devise an
accurate portrayal of the schedule adherence. After applying a scheduling algorithm, a
new project schedule can be obtained.

Construction Management 230 Dr. Emad Elbeltagi


B

Figure 10.1: Planned versus actual progress over time on a project

It is common that actual durations of activities differ from those estimated. Furthermore,
there may be additions or deletions to the scope of the contract that will affect the time at
which activities can be started or completed. Schedule updating is a procedure for
introducing the latest progress information into the schedule.

Data have to be collected on the actual progress of completed activities and those under
execution. A completely new estimate of the amount of work remaining to be done
should be made for each activity. The probable output of various resources should be
assessed. If the job is found to be behind schedule, corrective actions must be made to
retrieve position. A procedure for manual schedule updating can be summarized in the
following steps:
-Change the duration of all completed activities to zero.
-Identify all activities on which work is currently processing as Live Activities
-Put early start time of live activities equals the updating date and their durations equal
remaining duration.
-Change duration of future activities as given in the update report.
-Carry-out network analysis in the normal way and prepare a new activity schedule.

Construction Management 231 Dr. Emad Elbeltagi


To illustrate the above hand procedure for schedule updating, consider the following
example with the planning data given in Table 10.1. The corresponding precedence
network along with time analysis is shown in Figure 10.2. It is evident that initial project
duration is 20 weeks and the activities A, B, F, and K comprises the critical path.

Table 10.1: Planning data of the example problem

Activity Predecessors Duration (wks) Overlap (wks)


A --- 2
B A 2
C A 5
D A 3
E B 2
F B 6 -3
G C 6
H D 6
I D 4 -2
J E, F 2
K F, G 8 3 with G
L H 3
M H, I 2

At the end of the 7th week, new filed data are collected and the project status activities is
as follows:
-Activities A, B, D, and E have been completed.
-Remaining Duration of activity C is one week.
-Remaining Duration of activity H is 4 weeks.
-Activity G will not start until beginning of week 10.
-Overlap between activities K and G is one week only
-Volume of work of activity L has been increased by 33%.
-Activity J has been omitted.
The updated precedence network and the corresponding updated schedule are shown in
Figure 10.3. It is shown that a new critical path is developed. The new project completion
time is 21 weeks which indicates that a delay of one week is encountered. Corrective
actions should be taken to improve project performance during the remaining portion.

Construction Management 232 Dr. Emad Elbeltagi


4 6
E (2)
16 18
(12, 12)
2 4 7 13 13 15
B (2) -3 F (6) J (2)
2 4 7 13 18 20
(0, 0) (0, 0) (5, 5)
0 2 2 7 7 13 13 20 20 20
A (2) C (5) G (6) 3 K (7) Finish (0)
0 2 5 10 10 16 13 20 20 20
(0, 0) (0, 0) (3, 3) (0, 0) (0, 0)
2 5 5 11 11 14
D (3) H (6) L (3)
8 11 11 17 17 20
ES EF
(6, 0) (3, 0) (6, 6)
D (3) -2
7 11 11 13
LS LF
I (4) M (2)
(TF, FF)
14 18 18 20
(7, 0) (7, 7)
Figure 10.2: Scheduling data of the example problem

E (0)

7 13
B (0) F (6) J (0)
8 14
(0, 0)
7 8 9 15 14 21 21 21
A (0) C (1) G (6) 1 K (7) Finish (0)
8 9 9 15 14 21 21 21
(0, 0) (3, 3) (0, 0) (0, 0)
7 11 11 15
D (0) H (4) L (4)
11 17 17 21
(3, 0) (6, 6)
ES EF 7 11 11 13
D (3) I (4) M (2)
LS LF 15 19 19 21
(TF, FF) (7, 0) (7, 7)
Figure 10.3: Updated network of the example problem

Construction Management 233 Dr. Emad Elbeltagi


10.3 Delays Analysis

Work changes mean changes in the volume and duration of work to be performed from
that envisaged at the start of the contract. Variation in the form of addition and deduction
result in more or less cost and time to execute the varied item. On the other hand,
omissions mean less cost but not necessarily less time. It might result in wasting
resources. For instance, if the quantity of work in a critical activity is increased by x%
then the duration of the activity will be extended by x%. The direct cost of the activity
should be increased by the same ratio while the indirect cost of the contract might be
increased for the extended period.

It is typical for construction contracts to be delayed. A delay that occurred on a non-


critical activity does not participate to the delaying completion date of the contract.
Therefore, delays on non-critical paths are not considered.

10.3.1 Types of Delays

Delays can be divided into the following categories:


- Those over which the client has control; compensable delays;
- Those over which the contractor has control; non-excusable delays;
- Those over which the neither party has any control; excusable delays; and
- Concurrent delays.
A brief description of each category is given below.

Compensable delays

A delay is deemed compensable to the contractor when it’s within the control of, is
the fault of, or is due to the negligence of the client. Examples include:
- late possession of site;
- faulty design;
- incomplete drawings and specification;
- changes in scope;
- suspension of work;

Construction Management 234 Dr. Emad Elbeltagi


- differing site conditions;
- late delivery of client-supplied materials; and
- client’s failure to disclose information vital to the contractor.

For this type of delays, the conditions of contract should allow the contractor to be
entitled to a time extension and to monetary recompense for extra costs associated
with the delay.

Non-excusable delays

In this category, the contractor’s own actions or inactions have caused the delay.
The contractor is entitled neither time extensions nor monetary recompense from
the client. He/she may pay liquidated damages according to the contract.

Excusable delays

These are occurrences over which neither the client nor the contractor has any
control. Example includes:
- unforeseen future events which the contractor has not been aware;
- impracticable things which the contractor can only do at an excessive cost;
- events in which the contractor is blameless, such as material shortage beyond
what was expected at the time of bidding.

The contractor should declare the excusable delays. The sole relief for these delays
is a time extension.

Concurrent delays

Concurrent delays are two or more delays that occur at the same time, either of
which, if it occurred alone, would have affected contract completion date. They can
be classified as follows:
- excusable delays and non-excusable delays;
- excusable delays and compensable delays;

Construction Management 235 Dr. Emad Elbeltagi


- excusable delays and compensable delays and non-excusable delays; and
- compensable delays and non-excusable delays.

Concurrent delays with an excusable delay will generally be considered as


excusable delays. For these delays, the contractor is entitled to time extension if the
delays are on the critical path. This protects him from any resulting liquidated
damages. For concurrent compensable and non-excusable delays, the contractor is
allowed a time extension for completion with each party suffering his/her own
losses. The terms of the contract should declare the method of evaluation of such
claims.

10.3.2 The As-Built Schedule

The as-planned schedule of a contract is its initial schedule. The as-built schedule will
show the time status of the contract and the causes of all the time changes that happen.
Both schedules will be drawn as time-scaled diagrams.

The as-built schedule provides a complete record of the work as-built. It shows all delays
encountered and the actual starting and finishing dates of every activity. When compared
with the initial schedule, it gives the date for the evaluation of each time delays
encountered during construction. The following legend will be used to draw the as-built
schedule; “o” to represent compensable delays; “c” to represent non-excusable delays and
“n” to represent excusable delays. This schedule will now become the basis for analysis
of the effect of different types of delays of the contractor’s progress.

Analysis of The As-Built Schedule

If the as-built schedule contains more than one equally delayed critical path, each of
them will be examined in turn to determine its net working duration. This is the
actual time in which all the activities along a path could have been completed if
there had been no work changes or delays affecting the path. This can be found as
follows:

Construction Management 236 Dr. Emad Elbeltagi


A path net working duration = its total duration – all delay times lying on it

The net working duration may be less than the estimated contract duration given by
the as-planned schedule. This means that the contractor has performed the contract
within the estimated time. On the other hand, it may exceed the contract duration.
Then the contractor’s original estimates were incorrect. Having examined all
apparent critical paths, the scheduler can determine the primary path(s) as that
(those) with the longest net working duration. The work could not have been
completed in less time than this, even if the delays has not occurred, other parallel
apparent critical paths may be classified as secondary as they do not control the
contract duration. They have float with respect to the primary path.

If the as-built schedule contains one primary critical path, then the overall effects of
all eventualities on the contract will be the difference between the path actual
duration and its net working duration. The responsibility of each party for the
contract delayed completion is then determined by inspection. If the schedule
contains more than one primary critical path with the same net working duration,
then it may have concurrent delays. A brief discussion of these delays is provided
next.

10.3.3 Analysis of Concurrent Delays

The difference between the as-planned schedule duration and the as-built primary critical
path duration can be portioned out as follows:

1. The number of days in which the contractor’s and the client’s delays are
concurrent are those days where the two type of delays occurred; one delay
affected a primary critical path and the other affected the other primary critical
path on the same day(s).

Construction Management 237 Dr. Emad Elbeltagi


2. The number of days of concurrent delays with an excusable delay is those days
where an excusable delay occurred on any primary critical path and a contractor’s
delay, a client’s delay, or both delays occurred on other primary critical path(s) on
the same day(s).

3. Excluding delays number 1 and 2 above, the number of days a contractor should
be assessed for liquidate damages is the smallest number of days of inexcusable
delays on all primary critical paths.

4. Excluding delays number 1 and 2 above, the number of days a contractor should
be reimbursed for additional overhead expense plus a time extension is the
smallest number of days of compensable delays on all primary critical paths.

5. Finally, the number of days a contractor should be given a time extension is the
difference between the total delay duration and the summation of all the above
four delays duration.

Example 10.1

Consider the contract given in the Table 10.2. The delay report given in Table 10.3 was
recorded for this contract. Determine how each party is responsible for the contract
delayed completion.
Table 10.2: Data for Example 10.1

Activity Predecessor Duration (days)


A - 7
B - 5
C A 7
D B 9
E B 6
F C 4
G D 3
H E 9
I F 5
J H 3

Construction Management 238 Dr. Emad Elbeltagi


Table 10.3: Recorded delays for Example 10.1

Delay no. Category Activity affected Effective dates Delay time

1 Neither A 1 1
2 Contractor A 2-3 2
3 Contractor A 7 1
4 Contractor B 3 1
5 Owner B 4 1
6 Neither B 5-7 3
7 Contractor C 12-14 3
8 Owner C 15-16 2
9 Owner E 13-15 3
10 Contractor E 16 1
11 Neither E 19-23 5
12 Owner F 24-25 2
13 Owner G 22 1
14 Contractor G 23 1
15 Neither H 30 1
16 Owner H 33 1
17 Owner I 32 1
18 Neither I 33-34 2
19 Contractor I 35-36 2
20 Contractor I 39 1
21 Owner I 40 1
22 Neither J 37-38 2

Solution
The as-planned and as-built schedules are drawn in Fig. 10.4. Apparent critical paths
are B E H J and A C F I. Each of them has a net working duration = 41 – 18 = 23
days. So, the two apparent critical paths are primary critical paths. The total delay of
18 days can then be divided as follows:
1. Concurrent compensable 7 non-executable = 3 days
2. Concurrent with excusable = 2 days
3. Inexcusable = 1 day

Construction Management 239 Dr. Emad Elbeltagi


4. Compensable = 2 days
5. Excusable = 18 - (3 + 2 +1 +1) = 10 days

Accordingly, the contractor should be given a time extension of 17 days. He will pay
liquidated damages for 4 days and will be reimbursed for overheads of 5 days.

As-planned schedule
B E H J
5 6 9 3
D G
9 3 Legend:
A C F I c = contractor
7 7 4 5 o = owner
n = neither

As-built schedule
0 2 4 6 8 10 12 14 16 18 20 22 24 26 28 30 32 34 36 38 40

B E H J
c on nn o oo c n nn nn n o n n
2 1 1 3 3 2 3 1 2 5 2 4 1 2 1 3 2 3

D G
o c
9 2 1 1

A C F I
n cc c c cc o o o o o nn cc c o
1 2 3 1 4 3 2 7 2 4 2 1 2 2 2 1 1 1

Figure 10.4: Delays analysis for Example 10.1

10.4 Earned Value Management

For cost control on a project, the construction plan and the associated cash flow estimates
can provide the baseline reference for subsequent project monitoring and control. The
final or detailed cost estimate provides a baseline for the assessment of financial
performance during the project. To the extent that costs are within the detailed cost
estimate, then the project is thought to be under financial control. Overruns in particular

Construction Management 240 Dr. Emad Elbeltagi


cost categories signal the possibility of problems and give an indication of exactly what
problems are being encountered.
The key to a profitable project is to keep construction costs within the budget and to
know when and where job costs are deviating. The budget determines the amount of cash
that will be required over the various periods of the project. Various techniques are
usually used for cost control such as S-curve method and earned value technique.
Because it is the most widely used method, only earned value technique will be described
next. Earned value technique involves a combination of three measures that are needed
for the analysis. These measures include:

Budgeted Cost of Work Scheduled (BCWS)

BCWS measures what is planned in terms of budget cost of the work that should to
place (i.e., according to the baseline schedule of the project). BCWS curve can be
plotted by accumulating the budget cost of the initial schedule.

Budgeted Cost of Work Performed (BCWP) (Earned value – EV)

BCWP measures what is done in terms of the budget cost of work that has actually
had been accomplished to date. BCWP curve can be plotted point by point after each
reporting period. Here we accumulate the budget cost on the schedule that shows the
actual percent complete.

Actual Cost of Work Performed (ACWP)


ACWP measures what is paid in terms of the actual cost of work that has actually
been accomplished to date. BCWS curve can also be plotted point by point after each
reporting period. Here we accumulate the actual expenditures on the schedule that
shows the actual percent complete.

The significance of these three measures is that they directly indicate schedule and
cost performances of the project at different reporting periods. This illustrated as

Construction Management 241 Dr. Emad Elbeltagi


shown in Figure 10.5. Using these three measures, different project performance
indicators can be calculated. Among these indicators are:

Schedule Variance (SV)


It is the difference between the earned value (BCWP) and the planned budget cost
(BCWS).
SV = BCWP – BCWS;
SV > 0 indicates ahead of schedule while SV < 0 indicates behind schedule

Cost Variance (CV)


It is the difference between the actual cost (ACWP) and the earned value or the
budget cost (BCWP).
CV = BCWP – ACWP;
CV > 0 indicates cost saving while CV < 0 indicates cost overruns

Money
BCWS

BCWP (EV)
Variance
Cost

ACWP Schedule Variance

Time

Figure 10.5: Earned value measures and indictors

Schedule Performance Index (SPI)


SPI = BCWP / BCWS;
with SPI > 1 indicates ahead of schedule, and SPI < 1 indicates behind schedule.

Construction Management 242 Dr. Emad Elbeltagi


Cost Performance Index (CPI)
CPI = BCWP / ACWP;
with CV > 1 indicates cost saving, and CV < 1 indicates cost overruns.
Cost Schedule Index (CPI) = CPI × SPI
Estimate at Completion (EAC): What do we currently expect the total project to
accomplish? EAC = BAC / CPI
Estimate to Completion (ETC): From this point on, how much more do we expect to
be at end of the project? ETC = EAC – BCWP
Variance at Completion (VAC): How much over or under budget do we expect to be
at end of the project? VAC = BAC - EAC

These concepts are best illustrated by the example problem with the planning data given
in Table 10.1. The price of each activity is given Table 10.4.The schedule of the example
problem is given in Figure 10.2. At the end of week number 5, the project status is
recorded as given in Table 10.4. Actual costs are also recorded as given in Table 10.5.

Table 10.4: Cost data of the example problem


Activity A B C D E F G H I J K L M
Price (1000) 36 24 145 84 126 168 126 78 80 216 70 228 120

Actual Start 0 2 2 2 4
Projected
2 5 7 6 7
Completion

Table 10.5: Actual cost at the end of week 5

Week 1 2 3 4 5 Total
Cost 18 18 72 72 125 305
Figure 10.6 shows the project schedule in bar chart format. The planned expenditure of
each activity is assumed to be uniformly distributed over activity duration. The BCWP
are calculated as shown in Figure 10.7. The weekly budgets are plotted on a cumulative
basis as the BCWS are as shown in Figure 10.8. In Figure 9.8, BCWS as well as BCWP

Construction Management 243 Dr. Emad Elbeltagi


and ACWP to the end of week 5 are plotted on the same graph. It now possible to
calculate schedule and cost control indicators as:
SV = 262 – 294 = -ve which indicates that the project is behind schedule.
CV = 262 – 305 = -ve which indicates that the cost is overrun

Example 10.2

A cost control report of a certain contract gives the following figures as percentages
of the contract a-completion budgeted cost:

BCWP = 25% BCWS = 47% ACWP = 72%

Calculate, cost and schedule variances and comment on the status of the contract.

Solution

Cost variance (CV) = BCWP – ACWP = 25 – 72 = -47%


Schedule variance (SV) = BCWP – BCWS = 25 – 47 = -22%
The contract is behind schedule with over cost.

Act. Budget/w 1 2 3 4 5 6 7 8 9 10 11 12 13 14 15 16 17 18 19 20
A 18 18 18
B 12 12 12
C 29 29 29 29 29 29
D 28 28 28 28
E 63 63 63
F 28 28 28 28 28 28 28
G 21 21 21 21 21 21 21
H 13 13 13 13 13 13 13
I 20 20 20 20 20
J 108 108 108
K 10 10 10 10 10 10 10 10
L 76 76 76 76
M 60 60 60

Weekly
120
105

185

185

194
118
18

18
69
69

42
82
82
82
82

10
10
10
10
10

Cumulative
105
174
294
399
441
523
605
687
769
954
1139
1333
1451
1461
1471
1481
1491
1501
18

36

Figure 10.6: Bar chart showing budgeted expenditures (BCWS)

Construction Management 244 Dr. Emad Elbeltagi


Act. Budget/w 1 2 3 4 5 6 7 8 9 10 11 12 13 14 15 16 17 18 19 20
A 18 18 18
B 18 8 8 8
C 29 29 29 29 29 29
D 21 21 21 21 21
E 42 42 42 42

Weekly
58
58
100
18
18

Cumulative
18
36
94
152
262

Figure 10.7: Bar chart updated at end of week 5 (BCWP)

350
BCSW
300
BCWP
250 ACWP

200

150

100

50

Figure 10.8: Cost curves

Construction Management 245 Dr. Emad Elbeltagi


Exercises
1. The activities involved in the construction of one kilometer of a pipeline are given
together with their estimated durations in the table below. Each of the activities
will be done using a separate gang. All of them are sequential except “Excavate
trench” and “String pipe” are done concurrently. The project consists of 3 similar
kilometers.
Activity name Duration (days)
(LC) Locate and clear 1
(ET) Excavate trench 5
(SP) String pipe 2
(LP) Lay pipe 6
(PT) Pressure test 1
(BF) Backfill 2

a. Prepare complete plan of the works and determine the activity schedule. Mark
the critical path.

b. Assume now that the project is being constructed according to ES timings


determined in “a” above. At the end of day 10, the following information is
obtained from the project site:
- Activities LC were completed on schedule.
- Duration of the first part of activity ET was prolonged by 1 day but that of
the second part will be the same as the original duration.
- Activities SP were completed although ES of the third part had been
delayed by 3 days.
- Remaining duration of the first part of activity LP is 3 days.
- The first part of activity PT will not start until beginning of day 16.
Update your network and mark the critical path.

c. If the activities direct cost is listed as below:


Activity Direct Cost (LE)
LC 400
ET 3000
SP 600
LP 1800
PT 200
BF 1000

Construction Management 246 Dr. Emad Elbeltagi


The contract site overhead amounts to LE77.777/day and distributed among
activities according to their relative direct costs. If the actual cumulative cost at
the end of day 10 is LE9270, calculate schedule and cost performance ratios and
comment on the progress of the contract.

2. The indirect cost for this contract is LE 250/week. The contract conditions state
that the contractor will pay LE 200/week for delays by his own fault. Each of the
activities will be done using a separate gang.

Activity A B C D E F G H

Predecessors - - A B C, D B F E, G
Duration 7 9 8 12 4 9 7 7

Now, consider the project is under construction according to the original schedule
and the following table gives the delay report for this contract. Determine how
each party is responsible for the contract delayed completion.

Activities Effective Delay time


Number Category
affected dates (week)
1 Owner A 2–5 4
2 Contractor C 14 – 16 3
3 Owner C 21 – 23 3
4 Owner D 15 – 18 4
5 Contractor E 27 – 28 2
6 Neither F 18 – 19 2
7 Neither G 26 – 27 2
8 owner G 28 – 29 2

3. A project with LE200,000 planned value and 12 month duration. At the end of the
fourth month, it is found the only 40% of the project is performed with LE75,000
actual cost. Using the Earned Value method, calculate the following: CV, SV,
CPI, SPI, ETC, EAC and VAC.

Construction Management 247 Dr. Emad Elbeltagi


REFERENCES
Awani, Alfred O. (1983). “Project Management Techniques.” Petrocelli Books Inc.

Clough, Richard H. & Sears, Gelen A. (1979). “Construction Project Management.”


John Wiley & Sons Inc., NY.

Cormican, David. (1985). “Construction Management: Planning and Finance.”


Construction Press, London.

Eldosouky, Adel I. (1996). “Principles of Construction Project Management.” Mansoura


University Press, Mansoura, Egypt.

Gould, Frederick E. (1997). “Managing the Construction Process: Estimating,


Scheduling, and Project Control.” Prentice-Hall Inc., New Gersy.

Harris, Frank & McCaffer, Ronald. (1983). “Modern Construction Management.”


Granada Publishing, Great Britain.

Harris, Robert. (1978). “Precedence and Arrow Networking Techniques for


Construction.” John Wiley & Sons Inc., NY.

Hegazy, T. (2002). “Computer-Based Construction Project Management.” Prentice Hall,


Upper Saddle River, NJ, USA.

Pilcher, Roy. (1992). “Principles of Construction Management.” Mc-Graw Hill Book


company, 3rd ed.

ElgareAllah, Mohamed Ibrahim & Nawara, Jamal Mohamed. (1984). “Edarat Almsharee’
Alhandaseah.” John Wiley & Sons Inc., NY. (This book is available in Arabic).

.2006 ،‫ دار اﻟﻨﺸﺮ ﻟﻠﺠﺎﻣﻌﺎت‬، "‫اﺑﺮاھﯿﻢ ﻋﺒﺪ اﻟﺮﺷﯿﺪ ﻧﺼﯿﺮ "إدارة ﻣﺸﺮوﻋﺎت اﻟﺘﺸﯿﯿﺪ‬

Construction Management 248 Dr. Emad Elbeltagi

You might also like